TestPrep

Réussis tes devoirs et examens dès maintenant avec Quizwiz!

What is the correct dose, concentration, and route of epinephrine via EpiPen for anaphylactic shock in an adult? A) 0.3 mg; 1:1,000; intramuscular B) 0.15 mg; 1:1,000; intravascular C) 0.3 mg; 1:10,000; intramuscular D) 0.15 mg; 1:10,000; intravascular

A) 0.3 mg; 1:1,000; intramuscular In some EMS systems, EM Ts are trained to draw up and administer epinephrine from an ampule or a vial. In this case, the dose is 0.2 to 0.5 mg of a n ,000 solution (1 mg/mL) for adults and 0.01 mg/kg for children. Other systems carry epinephrine auto-injectors, or assist patients who have a prescription. The adult EpiPen delivers 0.3 mg of a J:1 ,000 solution, and the pediatric EpiPen delivers 0.15 mg of a J:2 ,000 solution. Whether drawn from an ampule or vial or delivered via EpiPen, the drug is administered via the intramuscular route. The J:1 ,000 concentration is very concentrated compared to J:10 ,000, and is therefore NOT administered by the intravascular route.

Following an injury, a 5-year-old child moans and makes incomprehensible sounds. According to the modified Glasgow Coma Scale for children, what score should be assigned for verbal response? A) 1 B) 2 C) 3 D) 4

A) 1 The Glasgow Coma Scale (GCS) is modified for children (older than 1 year of age) to accommodate their age- related behavior. Under the verbal component of the GCS, a score of 5 is assigned if the child engages in oriented conversation; a score of 4 is assigned if the child is confused, but still converses; a score of 3 is assigned if the child cries in response to pain; a score of 2 is assigned if the child uses inappropriate words; and a score of 1 is assigned if the child moans, is using is incomprehensible words or making incomprehensible sounds, or if there is no response at all.

A 5-year-old child has burns to his head, anterior chest, and both upper extremities. What percentage of his total body surface area has been burned? A) 45% B) 54% C) 63% D) 72%

A) 45% According to the pediatric Rule of Nines, the child's head represents 18% of his or her total body surface area (TBSA), the anterior chest represents 9% (the entire anterior torso [chest and abdomen] represents 18%), and each entire upper extremity represents 9%. Therefore, burns to the head, anterior chest, and both upper extremities cover 45% of the child's TBSA. The Rule of Nines is modified for infants and children. The head accounts for 18% of the child's TBSA (9% in adults) because the head is proportionately larger than an adult's. The lower extremities account for 13.5% (some references cite 14%) of the child's TBSA (18% in adults) because the child's lower extremities are proportionately smaller than an adult's.

Which of the following patients is the BEST candidate for fibrinolytic therapy? A) 50-year-old man who has slurred speech and left arm weakness that started 45 minutes ago; history of a heart attack 8 months ago B) 53-year-old woman who experienced a sudden, severe headache and is now unresponsive; history of a hemorrhagic stroke 3 years ago C) 59-year-old man whose mental status has decl ined over the past 12 hours; history of renal fa ilure, hypertension, and type 2 diabetes D) 63-year-old woman had a syncopal episode and is now confused; she takes blood thinners and has a history of deep vein thrombosis

A) 50-year-old man who has slurred speech and left arm weakness that started 45 minutes ago; history of a heart attack 8 months ago To be the most effective, fibrinolytic medications for stroke should be administered within the first 3 hours following the onset of signs and symptoms. In some patients, this time frame may be extended to 4.5 hours. Any history of intracranial hemorrhage, regardless of how long ago it was, is a contraindication for fibrinolytic therapy. Other contraindications include bleeding disorders (ie, hemophilia), use of blood thinning medications (ie, Coumadin, Eliquis, Pradaxa), and any active hemorrhage. The ultimate decision to initiate fibrinolytic therapy is the physician at the emergency department, in consultation with the neurologist. However, EMS can, through a targeted assessment, identify those who would most likely benefit from it.

A 65-year-old woman reports severe chest pain that radiates to her back, in between her shoulder blades. She tells you that this is the most severe pain that she has ever experienced, and that it has been intense since it began. Her past medical history includes hypertension. When you palpate her radial pulses, you note that they are unequal in strength. What should you suspect? A) Acute aortic dissection B) Unstable angina pectoris C) Acute hypertensive crisis D) Acute myocardial infarction

A) Acute aortic dissection The patient's clinical presentation is classic for acute dissection of the aorta. Dissection occurs when high pressure causes damage (dissection) through Lhe layers of the artery until an aneurysm eventually forms. Hypertension is a major risk factor for aortic dissection. The patient wilh acute aortic dissection typically presents with a sudden onset of severe chest pain, which is often described as the worst pain they have ever felt. In many cases, the patient radiates to the back, in between the scapulae. In contrast to the pain associated with acute cardiac ischemia (ie, unstable angina, myocardial infarction), which often begins acutely and progressively worsens, the pain from aortic dissection is of maximal intensity from the onset. Depending on the degree of dissection, blood pressure changes (greater than 15 mm Hg) may be noted between arms, and pulses in the upper extremities may be unequal in strength. Since there is no blood pressure provided, and the patient's symptoms are not consistent with acute hypertensive crisi (ie, headache, photophobia, nausea, vomiting), there is no evidence that this is what is causing her symptoms.

Which of the following is the definitive sign of imminent delivery of a baby? A) Crowning B) Urge to defecate C) Lengthy contractions D) Contractions less than 3 minutes apart

A) Crowning The operative word in this question is definitive. While the urge to defecate, contractions that are less than 2 to 3 minutes apart, and contractions that are lengthy, are all strong indicators that delivery is very close, nothing is more definitive than visualization of the presenting part of the baby at the vaginal opening (crowning).

A 3-year-old girl presents with respiratory distress. She is crying and is clinging to her mother. Her heart rate is 150 beats/min and her oxygen saturation is 89%. What should you do? A) Administer blow-by oxygen via nonrebreathing mask. B) Administer an inhaled bronchodilator via face mask. C) Ventilate with a bag-mask device at 20 breaths/min. D) Ventilate with a bag-mask device at 30 breaths/min.

A) Administer blow-by oxygen via nonrebreathing mask. If a child presents with respiratory difficulty, the method of oxygen delivery depends on his or her mental status, respiratory effort, and heart rate. A child with respiratory distress has an increased work of breathing, is agitated and tachycardic, and is clinging to his or her parent. Oxygen for a child with respiratory distress should be given by the least threatening method. You should avoid further agitation of the child, which may cause deterioration of his or her condition. Give the child oxygen via the blow-by technique; allow the parent to hold the oxygen mask near the child's face. In this particular child, you have not assessed enough to determine if wheezing is present; therefore, an inhaled bronchodilator is not indicated at this time. If the child develops signs of respiratory failure, such as a decreased level of consciousness, signs of physical exhaustion, reduced tidal volume (shallow breathing), cyanosis, and bradycardia, you should ventilate with a bag-mask device attached to supplemental oxygen.

When is it appropriate to clamp and cut the umbilical cord? A) As soon as the cord has stopped pulsating B) After the placenta has completely delivered C) Before the newborn has taken its first breath D) Immediately following delivery of the newborn

A) As soon as the cord has stopped pulsating Generally, it is safe to clamp and cut the umbilical cord once it has stopped pulsating and the baby is breathing adequately. There is evidence to suggest that delayed clamping of the umbilical cord is beneficial. When blood flow through the umbilical cord ceases, it will stop pulsating. If the cord does not stop pulsating and/or the baby is not breathing adequately, the cord should not be clamped and cut and the baby should be kept at the level of the mother's perineum and managed appropriately while en route to the hospital.

While triaging patients at the scene of a mass-casualty incident, you encounter a 3-year-old boy who is unresponsive and apneic. After opening his airway, you determine that he remains apneic. According to the JumpSTART triage system, what should you do next? A) Assess for a palpable pulse B) Deliver 5 rescue breaths C) Assign him as immediate D) Assign him as deceased

A) Assess for a palpable pulse The JumpSTART triage system was developed for infants and children in order to accommodate the developmental and physiological differences in children. The system is intended for children younger than 8 years of age or who appear to weigh less than 100 pounds (45 kg). If the child is unresponsive and apneic, you should open the airway. If the child begins to breathe, assign an immediate category. If the child remains apneic, assess for the presence of a pulse. If there is no palpable pulse, a deceased category is assigned. If a palpable pulse is present, deliver 5 rescue breaths. If breathing resumes after 5 rescue breaths, assign an immediate category. If the child remains apneic, assign a deceased category.

Which of the following signs is LEAST suggestive of a diabetic emergency? A) Bradycardia B) Tachycardia C) Combativeness D) Fruity breath odor

A) Bradycardia Bradycardia is not commonly associated with either hyperglycemia or hypoglycemia. Tachycardia and combativeness can occur in patients with hyperglycemia or hypoglycemia. A fruity or acetone breath odor, however, is a specific clinical finding in patients with hyperglycemic ketoacidosis (diabetic coma).

A 60-year-old woman presents with chest discomfort, confusion, and weakness. The patient's husband t ells you that she vomited once before EMS arrival. The patient's BP is 70/40 mm Hg, her pulse is 45 beats/min and weak, and her respirat ions are 14 breaths/min and unlabored. Which of the following is the MOST likely cause of her hypotension? A) Bradycardia B) Hypovolemia C) Myocardial isch emia D) Respiratory compromise

A) Bradycardia Given the patient's presentation, her slow heart rate (bradycardia) is the most likely cause of her hypo tension. When the heart rate is too slow or too fast, cardiac output can fall, resulting in hypotension. Hypovolemia would be expected to cause tachycardia, not bradycardia. There is no evidence of respiratory compromise in this patient; her breathing is unlabored and at a normal rate. Myocardial ischemia, which would explain her chest discomfort, may be caused by her bradycardia, but would not explain her blood pressure.

Which of the fo llowing signs is the MOST ominous in a child with respiratory distress? A) Bradypnea B) Tachycardia C) Nasal flaring D) Retractions

A) Bradypnea Retractions, nasal flaring, and tachycardia are all common findings in infants and children with respiratory distress. As the child begins to tire, however, retractions often become weak and ineffective and the accessory muscles become less prominent during breathing. Bradypnea, a decrease in the respiratory rate, is an ominous sign and indicates impending respiratory arrest. Do NOT mistake bradypnea for a sign of improvement; it usually indicates that the child's condition has deteriorated. Therefore, you should be prepared to provide ventilatory assistance.

Which of the following describes the MOST appropriate method of performing chest compressions on an adult patient in cardiac arrest? A) Compress the chest at least 2 inches, allow full recoil of the chest after each compression, minimize interruptions in chest compressions B) Allow full recoil of the chest after each compression, compress the chest to a depth of 2 inches, deliver compressions at a rate of at least 80/min C) Do not interrupt chest compressions for any reason, compress the chest no more than 1½ inches, allow partial recoil of the chest after each compression D) Minimize interruptions in chest compressions, provide 70% compression time and 30% relaxation time, deliver compressions at a rate of 100/min

A) Compress the chest at least 2 inches, allow full recoil of the chest after each compression, minimize interruptions in chest compressions Effective chest compressions are essential for providing forward blood flow during CPR. To perform adequate chest compressions, the EMT should compress the chest of an adult at a rate of 100 to 120/min to a depth of at least 2 inches. A compression depth that is greater than 2.4 inches should be avoided , although this is not possible to accomplish without a CPR device the provides immediate feedback. When performing chest compressions on an infant or child, compress the chest at least one-third the depth of the chest (about 1.5 inches for infants, about 2 inches for children). Allow the chest to fully recoil after each compression, avoid leaning on the chest, and allow equal time for compression and relaxation. Minimize interruptions in CPR to 10 seconds or less. Obviously, chest compressions must be paused when using the AED to analyze the patient's cardiac rhythm or defibril late and when assessing for a spontaneous pulse.

Which of the following assessment findings would you expect to encounter in a newly born infant? A) Cyanosis to the hands and feet B) Shallow, irregular, gasping breaths C) Delayed apnea for up to 2 minutes D) Heart rate between 80 and 100 beats/min

A) Cyanosis to the hands and feet Immediately following birth, and possibly for up to 12 to 24 hours, cyanosis to the hands and feet (acrocyanosis) is a common finding in newborns and usually resolves on its own. Cyanosis to the face and trunk, however, is not normal. Irregular breathing is normal in the newborn; shallow or gasping breaths, however, are not. A healthy newborn should have a heart rate greater than 100 beats/min. A heart rate less than 100 beats/min indicates hypoxia. Most newborns begin to breathe immediately or within the first 30 seconds following birth; apnea beyond 30 seconds is not normal.

The EMT is treating a man with chest pain and has assisted him with his nitroglycerin. Which of the fo llowing should the EMT anticipate during reassessment of this patient? A) Decreased blood pressure B) Increased level of anxiety C) Increased oxygen saturation D) Burning sensation in the chest

A) Decreased blood pressure Because nitroglycerin is a vasodilator, you should expect that the patient's blood pressure will be lower when you reassess it. Some patients experience only a minor decrease in blood pressure; other patients may experience a more significant decrease (one of the many reasons to reassess your patient). Nitroglycerin typically does not increase anxiety; the fact that it relieves their chest pain, however, may actually decrease their anxiety. Nitroglycerin has no effect on oxygen saturation. Some patients may experience a burning sensation under their tongue; a burning sensation in the chest is not typical following nitroglycerin administration.

Shortly after being stung by a scorpion, a 20-year-old woman develops hives to her chest and arms. She is conscious and alert, but states that it is difficult to swallow. Her heart rate is 100 beats/min and her oxygen saturation is 94%. Which of her signs or symptoms should concern the EMT the MOST? A) Difficulty swallowing B) Hives to her chest and arms C) Heart rate of 100 beats/min D) Oxygen saturation of 94%

A) Difficulty swallowing The patient is experiencing an allergic reaction. Of the signs and symptoms listed, the fact that she is experiencing difficulty swallowing (dysphagia) should be the most concerning because it indicates swelling of the throat and upper airway; this is clearly the most life-threatening of her signs and symptoms. Urticaria (hives) are a clear indication of an allergic reaction; however, by themselves, they are harmless. The patient's oxygen saturation and heart rate are not grossly abnormal; however, you should closely monitor them.

Which of the following injury mechanisms is associated with hangings? A) Distraction B) Subluxation C) Axial loading D) Hyperextension

A) Distraction Injury to the cervical spine following a hanging occurs via distraction, or stretching, of the vertebrae and spinal cord. A subluxation is a partial or incomplete dislocation; it is an injury, not an injury mechanism. Injuries related to hyperextension mechanisms are common in patients who strike their head on the windshield during a motor vehicle crash. Axial loading is a mechanism of injury in which the spinal column is compressed vertically. Injuries caused by axial loading include cervical spine injuries after diving head first into shallow water and lumbar spine injuries after a fall from a significant height in which the patient lands feet first.

Which of the following statements regarding sudden cardiac arrest and ventricular fibrillation is correct? A) For each minute that defibrillation is delayed, the chance of survival decreases by as much as 10%. B) High-quality CPR often reverses ventricular fibrillation if it is initiated within 2 minutes of the onset. C) Most patients develop ventricular fibrillation within 10 minutes after the onset of sudden cardiac arrest. D) Patients with ventricular fibrillation are typically unconscious, are apneic, and have a weak and irregular pulse.

A) For each minute that defibrillation is delayed, the chance of survival decreases by as much as 10%. Ventricular fibrillation (V-Fib), a chaotic quivering of the heart muscle, is the most common dysrhythmia that results in sudden cardiac arrest (SCA). In V-Fib, the heart is not beating effectively and is not pumping blood; therefore, the patient will not have a pulse. The single most important treatment for V-Fib is early defibrillation. Even if CPR is begun right at the time of the patient's collapse, the likelihood of survival decreases by as much as 10% for each minute that defibrillation is delayed. CPR alone rarely, if ever, converts V-Fib to a cardiac rhythm with a pulse. V-Fib is often a transient dysrhythmia, and the window of opportunity for successful defibrillation is very narrow. After 10 minutes of cardiac arrest, most patients deteriorate to asystole (absence of electrical and mechanical activity in the heart).

A young woman reports significant weight loss over the last month, persistent fever, and night sweats. When you assess her, you note the presence of dark purple lesions covering her trunk and upper extremities. You should suspect: A) HIV infection. B) tuberculosis. C) rheumatic fever. D) end-stage cancer.

A) HIV infection. Weight loss, fever, and night sweats could indicate tuberculosis or HIV infection that has progressed to acquired immunodeficiency syndrome (AIDS); however, the dark purple lesions on the skin, which are called Kaposi's sarcoma, are malignant skin tumors and are a classic finding in patients in the later stages of AIDS.

Which of the following is an abnormal finding? A) Heart rate of 80 beats/min in a 3-month-old infant B) Rapid, irregular breathing in a newly born infant C) Systolic BP of 100 mm Hg in a 10-year-old child D) Respiratory rate of 26 breaths/min in a 2-year-old child

A) Heart rate of 80 beats/min in a 3-month-old infant The heart rate for an infant up to 3 months of age varies, depending on whether they are awake or asleep, but averages 140 beats/min. Therefore, a heart rate of 80 beats/min is grossly abnormal in this same age group and indicates bradycardia. Newborn infants normally have irregular breathing that ranges between 40 and 60 breaths/min. The systolic BP for a child between 6 and 12 years of age typically ranges between 90 and 115 mm Hg. The respiratory rate in a child between 1 and 3 years of age typically ranges between 24 and 40 breaths/min.

You are transporting a 30-year-old man who is experiencing an emotional crisis. The patient does not speak when you ask him questions. How should you respond to his unwillingness to speak? A) Remain sient until the pateint speaks to you. B) Continually encourage the patient to talk to you. C) Tell the patient that you cannot help if he will not talk. D) Do not speak to the patient, even if he begins to speak to you.

A) Remain sient until the pateint speaks to you. When dealing with a psychiatric patient who is silent and unwilling to speak to you, do not fear the silence. The patient simply does not wish to speak. You should not press the issue, because doing so may upset the patient. You should remain calm until the patient speaks to you, and then respond accordingly.

A 72-year-old woman is found unresponsive in her poorly ventilated home. Her skin is flushed, hot, and dry, and her respirations are rapid and shallow. She is wearing a medical alert bracelet that states she is a diabetic and is allergic to sulfa drugs. What should you suspect? A) Heatstroke B) Anaphylactic shock C) Acute ischemic stroke D) Diabetic complications

A) Heatstroke The patient's signs and symptoms are consistent with classic heatstroke. Unlike exertional heatstroke, which usually affects otherwise healthy people who exert themselves in the heat for long periods of time, classic heatstroke commonly affects children and older adults, and it typically occurs when the patient is in a hot, poorly ventilated space for a prolonged period of time. Significant underlying medical problems (eg, hypertension, diabetes, heart disease) increase the patient's risk for classic heatstroke. Both exertional and classic heatstroke present with hot, flushed skin; however, patients with exertional heatstroke may have moist skin, usually from residual perspiration, whereas patients with classic heatstroke typically have dry skin.

Which of the following clinical findings are consistent with epiglottitis? A) High fever and drooling B) Low-grade fever and stridor C) Low-grade fever and wheezing D) High fever and crackling breath sounds

A) High fever and drooling Epiglottitis is a bacterial infection of the soft tissue in the area above the vocal cords. In preschool- and school-aged children especially, the epiglottis can swell to two to three times its normal size, potentially causing an airway obstruction. Signs and symptoms include a high fever, sore throat, and respiratory distress. Because it hurts to swallow, drooling is very common. In many cases, you will find the child in the tripod position. Wheezing indicates a lower airway obstruction, such as asthma or bronchiolitis; it is not commonly seen in children with epiglottitis. Fever, crackles heard in the lungs, and respiratory distress are common findings with pneumonia, not epigiottitis.

Which of the following is the MOST significant finding in a patient experiencing an allergic reaction? A) Hoarseness B) Headache C) Abdominal cramps D) Widespread rash

A) Hoarseness A rash (urticaria), headache, abdominal cramps, and hoarseness are all signs of an allergic reaction. However, the presence of hoarseness is the most significant finding because it indicates upper airway swelling. Other significant findings include swelling of the face, neck, or tongue; wheezing and/or stridor; difficulty breathing; tachycardia; and hypotension. Death due to a severe allergic reaction (anaphylaxis) is caused by vascular collapse (caused by massive vasodilation) and respiratory failure (caused by upper and/or lower airway swelling).

A 29-year-old pregnant woman reports a severe headache, blurred vision, and swelling of her hands and feet. Which of the following additional signs or symptoms would MOST likely be present? A) Hypertension B) Hyperglycemia C) Abdominal pain D) Vaginal bleeding

A) Hypertension The patient's signs and symptoms are consistent with preeclampsia, a condition that could ultimately lead to seizures (eclampsia). Signs and symptoms of preeclampsia include hypertension, headache, visual disturbances, and edema. Hyperglycemia during pregnancy is a possible sign of gestational diabetes, not preeclampsia. Abdominal pain and vaginal bleeding should make you suspicious for another underlying problem (ie, abruptio placenta, spontaneous abortion) as they are not commonly observed in patients with preeclampsia.

Which of the following is the MOST common cause of shock in infants and children? A) Hypovolemia B) Cardiac failure C) Accidental poisoning D) Severe allergic reaction

A) Hypovolemia Of the causes of shock listed, hypovolemia is clearly the most common in children. Hypovolemia can occur as a result of dehydration (even from a few episodes of vomiting and/or diarrhea), prolonged and excessive fever, and blood loss from trauma. Less common causes include severe allergic reactions (anaphylaxis) cardiac failure, and poisonings.

A man's arm was amputated during a car crash. The patient is unresponsive; has rapid, shallow breathing; and has a rapid, weak pulse. As you and your partner are treating him, other responders are attempting to locate his severed arm. Which of the following statements regarding this scenario is correct? A) If the patient's arm has not been recovered by the time you are ready to transport, you should transport without delay. B) Quickly move the patient to the ambulance, continue treatment, and wait for the other responders to recover his arm. C) You should transport the patient immediately, even if the other responders recover his arm before you depart the scene. D) Your priority should be to recover the man's arm because a vascular surgeon may be able to successfully reattach it.

A) If the patient's arm has not been recovered by the time you are ready to transport, you should transport without delay. Life over limb! The patient is in shock and requires transport without delay. If the severed arm is recovered before you depart, take it with you. Otherwise, your priorities are to provide aggressive shock treatment and transport the patient to a trauma center. If the limb is recovered after you depart the scene, another provider can transport it to the trauma center, ensuring that it is wrapped in sterile dressings and kept cool.

Following blunt force trauma to the anterior chest, a man presents with difficulty breathing, distended jugular veins, absent breath soundsover the left side of the chest, and hypotension. Which of the following BEST describes the pathophysiology of this patient's injury? A) Increased pressure in the pleural space is compressing the great vessels B) Blood is filling the pleural space and is collapsing the lung on the left side C) Blood is filling the pericardia! sac and is restricting cardiac relaxation D) The aorta has been injured and blood is rapidly filling the thoracic cavity

A) Increased pressure in the pleural space is compressing the great vessels The patient is experiencing a tension pneumothorax. This type of injury occurs when air fills the pleural space and progressively collapses the lung. In the process, the vena cavae are compressed and blood return to the heart is reduced; clinically, this manifests as jugular vein distention because blood is backing up into the systemic venous system. If blood return to the heart is reduced, the amount of blood that leaves the heart will also be reduced; as a result, cardiac output falls and the patient becomes hypotensive. Breath sounds are markedly decreased or absent on the affected side of the chest because the lung is being collapsed. In a hemothorax, blood fills the pleural space instead of air. Breath sounds are decreased or absent on the affected side; however, because the patient is losing blood volume into the chest, the jugular veins would be collapsed, not distended as they are with a tension pneumothorax. Pericardia! tamponade also causes jugular vein distention; however, the patient's breath sounds are equal bilaterally (unless a pneumothorax is also present). Aortic injury would be expected to cause collapsed jugular veins; like the hemothorax, the patient is losing blood volume into the chest cavity. By itself, aortic injury does not cause unequal breath sounds.

Upon assessing a newborn, you note that the infant is breathing spontaneously and has a heart rate of 80 beats/min. What should you do? A) Initiate positive pressure ventilations. B) Provide blow-by oxygen with oxygen tubing. C) Assess the newbom's skin condition and color. D) Perform chest compressions at 120 per minute.

A) Initiate positive pressure ventilations. Positive pressure ventilations are indicated in the newborn if he or she is apneic or has gasping respirations, if the heart rate is less than 100 beats/min, or if central cyanosis persists despite the delivery of blow-by oxygen. Chest compressions are indicated if the heart rate is less than 60 beats/min, despite adequate positive pressure ventilation. In many cases, the newborn's heart rate will increase to greater than 100 beats/min with adequately performed positive pressure ventilation. If the newborn is breathing adequately and has a heart rate greater than 100 beats/min, you should proceed to assess skin color.

A known woman with diabetes is found unresponsive. Her respirations are rapid and shallow; her skin is cool, clammy, and pale; and her pulse is rapid and weak. Which of the following would BEST explain the likely cause of her condition? A) Insulin overdose B) Excessive eating C) High blood sugar D) Failure to take insulin

A) Insulin overdose The patient has classic signs of insulin shock, a condition caused by a low blood glucose level (hypoglycemia). Common causes of insulin shock include insulin overdose (accidental or intentional), failure to eat (or not eating enough), and excessive exertion. The patient's symptoms are not consistent with diabetic ketoacidosis (OKA), a condition that results from a high blood glucose level (hyperglycemia). Signs of OKA include deep, rapid breathing (Kussmaul respirations) with a fruity or acetone breath odor, and warm, dry skin. Unlike insulin shock, which can result from an insulin overdose, OKA can occur if a patient fails to take his or her insulin or takes too little.

Injury to which of the following organs would MOST likely cause hemorrhagic shock? A) Liver B) Kidney C) Stomach D) Bladder

A) Liver The liver is a large vascular organ that holds a significant amount of blood at any given time. Following abdominal trauma, liver injuries can be a source of significant blood loss and hemorrhagic shock. Although the kidneys are also solid organs, they are not as vascular as the liver; kidney injuries can contribute to hemorrhagic shock but are less commonly the sole cause. The stomach and bladder are hollow organs. Hollow organs, when injured, spill their contents into the abdominal cavity, resulting in intense peritoneal irritation and inflammation (peritonitis). The most significant complication associated with peritonitis is infection.

A man was stabbed in the lower right ribcage. He is diaphoretic; his pulse is rapid and weak; and his respirations are regular and unlabored at 24 breaths/min. Which of the following injuries should the EMT suspect? A) Liver laceration B) Ruptured diaphragm C) Massive hemothorax D) Tension pneumothorax

A) Liver laceration Anatomically speaking, the liver is protected by the lower right ribcage. The lungs are not as low in the thoracic cavity as some illustrations would suggest. The patient's unlabored breathing tends to point away from a pneumothorax or hemothorax. Although a diaphragmatic injury cannot be ruled out, one would expect a certain degree of respiratory distress. Based on the anatomic location of the injury, in addition to the patient's clinical presentation, a liver laceration is the most likely injury.

A hiker fell 25 feet from a ledge. There is obvious deformity to his thoracic spine and he has a large laceration on his forehead. His BP is 60/40 mm Hg, pulse is 50 beats/min, and respirations are 26 breaths/min. His face and chest are pale and cool, but his abdomen and lower extremities are pink and warm. Which of the following BEST describes the pathophysiology of these findings? A) Loss of nervous system control over the systemic vasculature B) Severe bleeding into the thoracic cavity from a ruptured aorta C) Systemic vasoconstriction due to nervous system hyperactivity D) Increased intracranial pressure due to bleeding within the brain

A) Loss of nervous system control over the systemic vasculature On the basis of the mechanism of injury and assessment findings, the EMT should suspect that the patient is experiencing neurogenic shock. Neurogenic shock occurs when an injury or condition (in this case, a spinal injury) interrupts the nervous system's control over the diameter of the blood vessels. As a result, the blood vessels dilate and the patient's blood pressure falls. The nervous system releases epinephrine and norepinephrine when a patient is in shock, which results in tachycardia and vasoconstriction. However, if the nervous system is impaired, as with neurogenic shock, these catecholamines do not get released. Therefore, the patient with neurogenic shock is bradycardic, not tachycardic as you would expect with other types of shock that do not involve nervous system impairment (ie, hypovolemic, septic, anaphylactic). The blood vessels above the level of the injury are still able to constrict, so the skin is pale and cool; however, the blood vessels below the level of the injury are dilated, so the skin is pink and warm. If this patient had a head injury with increased intracranial pressure, you would expect him to be hypertensive, not hypotensive.

A 5-year-old child has had severe diarrhea and vomiting for 3 days and is now showing signs of shock. Supplemental oxygen has been given, and you have elevated his lower extremities. En route to the hospital, you note that his work of breathing has increased. What should you do? A) Lower the extremities and reassess the child. B) Begin positive pressure ventilations and reassess the child. C) Insert a nasopharyngeal airway and increase the oxygen flow. D) Listen to the lungs with a stethoscope for abnormal breath sounds.

A) Lower the extremities and reassess the child. Because infants and small children rely heavily on their diaphragm for breathing (as evidenced by belly breathing), elevating their lower extremities can cause the diaphragm to shift into the thoracic cavity and decrease the effectiveness of breathing. Therefore, in the case of this child, you should lower the lower extremities and reassess. A supine position is recommended for any patient in shock, regardless of age.

After covering a large open chest wound with an occlusive dressing, it becomes necessary to ventilate the patient with a bag-mask device. What should you do? A) Remove the occlusive dressing. B) Ventilate with greater volume. C) Ventilate at 24 breaths/min. D) Request a paramedic intercept.

A) Remove the occlusive dressing. If it becomes necessary to ventilate a patient after covering an open chest wound, you should remove the occlusive dressing. With the wound closed, positive pressure ventilation will quickly increase intrathoracic pressure, resulting in a tension pneumothorax. Ventilating the patient with greater volume and/or a faster rate would only cause a more rapid increase in pleural tension and should be avoided. Excessive ventilation can also reduce venous return to the heart, causing a decrease in perfusion. Consider requesting a paramedic intercept, as long as it does not cause a delay in transporting the patient to a trauma center.

A 23-year-old woman with a history of bulimia is vomiting large amounts of bright red blood and complains of pain in her upper abdominal area. What should you suspect? A) Mallory-Weiss tear B) Esophageal varices C) Gastroesophageal reflux disease D) Lower gastrointestinal bleeding

A) Mallory-Weiss tear Based on her history and symptoms, a Mallory-Weiss tear is the most likely underlying etiology. A Mallory-Weiss tear is a tear in the junction between the esophagus and the stomach, causing severe bleeding and potentially death. Primary risk factors include alcoholism and eating disorders. The patient has a history of bulimia, an eating disorder characterized by binging on large amounts of food, followed by self-induction of vomiting. Vomiting is the principle symptom of a Mallory-Weiss tear. In extreme cases, patients may experience upper abdominal pain and hemorrhagic shock. Esophageal varices also present with hematemesis; however, it is generally accompanied by difficulty swallowing and is more common in patients with alcoholism. Upper abdominal pain is uncommon with ruptured esophageal varices. Lower gastrointestinal bleeding would be expected to cause blood passage in the stool, not hematemesis. Gastroesophageal reflux disease (GERO) is condition in which gastric acids regurgitate from the stomach and into the esophagus. The principle symptom of GERO is heartburn.

Which of the following injury mechanisms and clinical findings would MOST likely warrant transport to a facility that provides the highest level of trauma care? A) Motorcycle crash; pelvic instability; systolic BP of 80 mm Hg B) Fall from a standing position; no loss of consciousness; GCS of 14 C) Small-caliber gunshot wound to the calf; heart rate of110 beats/min D) Rollerblade accident; humeral fracture; heart rate of 100 beats/min

A) Motorcycle crash; pelvic instability; systolic BP of 80 mm Hg Transport destinations for trauma patients are based on the 2011 Centers for Disease Control and Prevention (CDC) guidelines for the field triage of injured patients. Regional protocols may be developed; however, they generally follow the CDC guidelines. Injuries or clinical findings that warrant transport to a facility that provides the highest level of trauma care include a GCS that is equal to or less than 13 following trauma; a systolic BP less than 90 mm Hg; a respiratory rate less than 10 or greater than 29 breaths/min, or the need for ventilatory support; all penetrating injuries to the head, neck, torso, or extremities proximal to the to the knee or elbow; chest wall instability or deformity (eg, flail chest); two or more proximal long bone fractures; a crushed, degloved, mangled, or pulseless extremity; amputation proximal to the ankle or wrist; pelvic fractures; open or depressed skull fracture; and paralysis. The motorcycle rider meets two of these criteria; he has pelvic instability and a systolic BP less than 90 mm Hg. The EMT should be familiar with the trauma triage criteria in his or her jurisdiction.

Unresponsiveness, shallow breathing, and constricted pupils are indicative of which type of drug overdose? A) Narcotic B) Marijuana C) Barbiturate D) Amphetamine

A) Narcotic Signs of a narcotic (opiate/opioid) overdose from drugs such as heroin, morphine (Astromorph, Duramorph), meperidine (Demerol), and codeine include altered mental status; slow, shallow breathing; pupillary constriction (miosis), hypotension; and bradycardia. Narcotics are central nervous system depressants that, when taken in excess, suppress the vital functions necessary for life, such as breathing, heart rate, and blood pressure. Barbiturates produce the same effects; however, the pupils are typically dilated (mydriasis), not constricted. Marijuana and amphetamine drugs are central nervous system stimulants and, therefore, would cause the patient to become restless or even combative.

Which of the following signs or symptoms are more common in children than in adults following an isolated head injury? A) Nausea and vomiting B) Altered mental status C) Tachycardia and diaphoresis D) Changes in pupillary reaction

A) Nausea and vomiting It is relatively common for children to vomit following a head injury such as a concussion. In adults, vomiting--though less common--is an ominous sign and indicates increased intracranial pressure. You must always be prepared for vomiting and have suctioning equipment readily available when managing the patient with a head injury. Altered mental status and pupillary changes following a head injury are equally as common in children and adults. Tachycardia and diaphoresis are signs of shock and are not commonly observed in patients with an isolated head injury.

Which of the following statements regarding heatstroke is correct? A) Not all patients experiencing heatstroke have dry skin. B) Heatstroke is caused by a hyperactive sweating mechanism. C) Heatstroke is more likely to occur when the humidity is low. D) As core body temperature rises, the patient becomes more agitated.

A) Not all patients experiencing heatstroke have dry skin. Heatstroke occurs when the body is exposed to more heat than it can eliminate and normal mechanisms for eliminating heat, such as sweating, are overwhelmed. The core body temperature then rises rapidly to the point where tissues are destroyed. Heatstroke can develop in patients during prolonged vigorous physical activity or when they are in a closed, poorly ventilated, hot and humid space. High humidity impairs a person's ability to eliminate heat via the sweating mechanism. Many patients with heatstroke have hot, dry, flushed skin; however, early in the course of heatstroke, the skin may be moist due to residual perspiration, as with exertional heatstroke. As the core body temperature rises, the patient's level of consciousness decreases. Untreated heatstroke will result in death.

Which of the following is an abnormal finding when using the Cincinnati Stroke Scale to assess a patient who presents with signs of a stroke? A) One arm drifts down compared with the other side. B) One of the pupils is dilated and does not react to light. C) The patient's face is symmetrical when he or she smiles. D) Both arms drift slowly and equally down to the patient's side.

A) One arm drifts down compared with the other side. The Cincinnati Stroke Scale is used to assess patients suspected of experiencing a stroke. It consists of three tests: speech, facial droop, and arm drift. Abnormality in any one of these areas indicates a high probability of stroke. To test arm drift, ask the patient to hold both arms in front of his or her body, palms facing upward, with eyes closed and without moving. Over the next 10 seconds, observe the patient's arms. If one a1·m drifts down toward the ground, you know that side is weak; this is an abnormal finding. To test for facial droop, have the patient smile, showing his or her teeth. The face should be symmetrical (both sides of the face should move equally). If only one side of the face moves well, you know that something is wrong with the part of the brain that controls the facial muscles. You should assess the pupils of a patient with a suspected stroke; however, this is not a component of the Cincinnati Stroke Scale.

Which of the following signs and symptoms should the EMT expect to encounter in a patient with severe anemia? A) Pallor and shortness of breath B) Flushed skin and a slow pulse C) Jaundice and abdominal pain D) Hematemesis and nosebleed

A) Pallor and shortness of breath Anemia is a deficiency of red blood cells (erythrocytes). Recall that red blood cells carry hemoglobin, which binds to oxygen, in order to deliver oxygen to the body's cells and tissues. You would expect the severely anemic patient to be pale (because of fewer red blood cells). In addition, because the patient's oxygencarrying capacity is reduced, shortness of breath is also a common finding. In response to hypoxia, the patient would become tachycardic. Flushed (red) skin is not expected in anemic patients; you would expect flushing to occur if the patient had excessive red blood cell production (polycythemia). Jaundice (yellow skin) is observed in patients with liver disease or dysfunction, not anemia. Hematemesis and nosebleed may be observed in patients with low platelet levels (thrombocytopenia), not anemia.

Which of the following injuries has the potential to produce the greatest amount of internal blood loss? A) Pelvic fracture B) Femur fracture C) Tibial fracture D) Humeral fracture

A) Pelvic fracture The pelvis is an incredibly vascular bone. In addition to the fact that numerous major blood vessels lie nearby, a patient can lose multiple liters of blood into the pelvic cavity. A single femur fracture is capable of producing approximately 1 to 1.5 liters of blood loss. An isolated humeral fracture could cause a loss of blood of up to 750 mL. Tibial fractures typically do not produce significant internal blood loss.

A 29-year-old woman, who is 38 weeks' pregnant, presents with heavy vaginal bleeding, a blood pressure of 70/50 mm Hg, and a heart rate of 130 beats/min. She is pale and diaphoretic, and denies abdominal cramping or pain. What should you suspect? A) Placenta previa B) Abruptio placenta C) Ruptured ovarian cyst D) Ruptured ectopic pregnancy

A) Placenta previa Of the conditions listed, placenta previa would be the least likely to present with abdominal pain, although some patients may have pain or cramping. Placenta previa is a condition in which the placenta develops over and covers some or all of the cervix. As the cervix dilates, the vasculature that attaches the placenta to the uterine wall tears, resulting in vaginal bleeding that is often severe enough to cause shock. By contrast, abruptio placenta is a condition in which the placenta prematurely separates from the uterine wall; it is characterized by tearing abdominal pain, heavy vaginal bleeding, and shock. Placenta previa and abruptio placenta occur during the later stages of pregnancy. A ruptured ovarian cyst typically causes lower abdominal pain, often unilateral. Ectopic pregnancy, a condition in which the egg implants and grows outside the uterus (usually in a fallopian tube), is a first-trimester condition; it is typically discovered between 8 and 10 weeks of pregnancy. If the ectopic pregnancy ruptures, the patient often presents with a sudden stabbing pain in the lower abdomen and shock due to intra-abdominal hemorrhage.

A 30-year-old woman is 22 weeks' pregnant with her first child. She tells you that her rings are not fitting as loosely as they usually do and that her ankles are swollen. Her blood pressure is 150/86 mm Hg. What should you suspect? A) Preeclampsia B) Gestational diabetes C) A hypertensive emergency D) A condition unrelated to pregnancy

A) Preeclampsia Preeclampsia typically develops after the 20th week of gestation; it most commonly occurs in primigravida (first-time pregnancy) women. Preeclampsia is characterized by a headache, visual disturbances, edema to the hands and feet, anxiety, and persistent hypertension. Left untreated, preeclampsia can lead to seizures (eclampsia). Gestational diabetes, a condition in which the pregnancy hormones estrogen and progesterone impair the effects of insulin (insulin resistance), is characterized by an increase in the patient's blood glucose level (BGL); there is no mention of the patient's BGL in this scenario. A hypertensive emergency usually occurs when the systolic blood pressure acutely rises above 180 mm Hg.

A 33-year-old factory worker was crushed between two pieces of machinery. You find him lying supine on the ground reporting severe pain to his pelvis. He is restless, diaphoretic, and tachycardic. What should you do? A) Prepare for immediate transport. B) Perform a detailed secondary exam. C) Carefully log roll him to check his back. D) Palpate his pelvis to assess for crepitus.

A) Prepare for immediate transport. Based on the mechanism of injury and the presence of signs of shock (eg, restlessness, tachycardia, diaphoresis), you should suspect that the patient has a fractured pelvis and is bleeding internally. Therefore, after completing your primary assessment and initiating shock treatment (eg, high-flow oxygen, applying blankets), you should perform a rapid head-to-toe assessment to assess for other injuries and then prepare for immediate transport. Spinal precautions should be considered. Do not log roll the patient; doing so compresses the pelvis and may cause further injury. You should also avoid palpating his pelvis; this will only cause further pain and may cause additional injury. Palpation of the pelvis is performed to assess its stability, not to elicit crepitus. Consider applying a pelvic binder device or tying a sheet around his hips in order to reduce the space within the pelvis; doing so may help slow internal bleeding. A detailed secondary exam of a critically injured patient at the scene is not appropriate; it takes too long to perform and should be done en route to the hospital if time permits.

A 45-year-old man reports shortness of breath that began suddenly. He tells the EMT that he has a condition called thrombophilia. What should you suspect? A) Pulmonary embolism B) Acute bronchospasm C) Congestive heart failure D) Intrapulmonary bleeding

A) Pulmonary embolism It is reasonable to believe that any one of the listed conditions could explain the patient's sudden shortness of breath. However, his history clearly leans toward a pulmonary embolism. Thrombophilia is an abnormality of blood coagulation that increases the risk of blood clots. Patients with this condition are often referred to as being hypercoagulable or prothrombotic. This case is a prime example of how knowing the patient's medical history can assist you in your differential diagnosis.

A young man fell and landed on his outstretched hand, resulting in pain and deformity to the left midshaft forearm. Distal circulation should be assessed at which of the following pulse locations? A) Radial B) Brachia! C) Pedal D) Popliteal

A) Radial The radius and ulna are the bones of the forearm. The radial pulse can be palpated on the lateral aspect (thumb side) of the wrist and is the most distal pulse site relative to the injury. The brachia! pulse is located on the medial aspect of the arm. The popliteal pulse is located behind the knee. The pedal (dorsalis pedis) pulse is located on top of the foot.

A patient has an open chest wound, which has been covered with an occlusive dressing. He is receiving oxygen at 12 L/min by nonrebreathing mask. During transport, the patient's heart rate increases, he becomes pale and diaphoretic, and his oxygen saturation falls. What should you do? A) Remove the occlusive dressing. B) Ventilate with a bag-mask device. C) Increase the oxygen to 15 L/min. D) Encourage him to take deep breaths.

A) Remove the occlusive dressing. When placing an occlusive dressing over a sucking chest wound (open pneumothorax), it is important to remember that you have converted the injury to a closed pneumothorax. With no way for the air in the pleural space to escape, and depending on the size of the pneumothorax, the patient could develop excessive pleural tension that interferes with circulation (tension pneumothorax). You should suspect that this is what is happening to the patient in this scenario. Your most immediate action should be to remove the occlusive dressing and allow air to escape from the pleural space. Positive pressure ventilation {ie, bag-mask ventilation) in a patient whose open chest injury has been covered with an occlusive dressing may rapidly lead to a tension pneumothorax; therefore, if the patient must be ventilated, the occlusive dressing should be removed.

A 20-year-old man was stung on the arm by a bee and is experiencing local redness, pain, and swelling. You can see that the stinger is still in the man's arm. What should you do? A) Remove the stinger by gently scraping the skin with the edge of a rigid object B) Administer 0.3 mg epinephrine via EpiPen and then contact medical control C) Carefully grab the stinger with a pair of tweezers and gently lift it straight up D) Apply a pressure bandage to the site and elevate it above the level of the heart

A) Remove the stinger by gently scraping the skin with the edge of a rigid object If the stinger from an insect is still in place, attempt to remove the stinger by scraping the skin with the edge of a sharp, stiff object such as a credit card. Do not use tweezers or forceps to remove the stinger because this may squeeze more venom into the wound. After the stinger has been removed, gently clean the area with soap and water. Position the injection site slightly below the level of the heart, and apply an icepack to the area. Applying ice over the site may slow absorption of the venom, diminish swelling, and relieve pain. The patient in this scenario is experiencing a local reaction; therefore, epinephrine is not indicated at this time.

A patient experienced blunt trauma to the left upper abdominal quadrant. When he is positioned supine, he experiences severe pain in his left shoulder. Which of the following should the EMT suspect? A) Ruptured spleen B) Lacerated bowel C) Injury to the kidney D) Perforated stomach.

A) Ruptured spleen The site of intra-abdominal bleeding can be difficult to determine, even by experienced surgeons. However, if you know your human anatomy, you will know that the spleen is located in the left upper abdominal quadrant. Furthermore, splenic injuries can present with referred pain to the left shoulder (Kehr sign). Injury to the kidney would be expected to produce flank pain and possibly hematuria. Injury to the gastrointestinal organs (ie, stomach, bowel) typically do not present with referred pain. When managing a patient with abdominal trauma, especially one who is in shock, your main focus should be initiating appropriate treatment and providing prompt transport to the hospital.

While assessing a patient who was ejected from his truck, the EMT notices that his chest collapses and his abdomen rises during inhalation. What should the EMT suspect? A) Spinal cord injury B) Fractured sternum C) Ruptured diaphragm D) Intraabdominal bleeding

A) Spinal cord injury The patient is exhibiting diaphragmatic breathing, which explains why his abdomen moves but his chest does not. This indicates a spinal cord injury below the CS level. The phrenic nerves, which innervate the diaphragm, arise from C3-CS. However, the intercostal nerves, which arise from below the CS level, have been interrupted; this would explain the absence of chest wall movement. A fractured sternum, depending on the severity of the fracture, would be expected to cause the chest to collapse during inhalation and bulge during exhalation, similar to a flail chest. A ruptured diaphragm would be expected to present with a scaphoid (concave) abdomen and decreased abdominal movement; in some cases of diaphragmatic rupture, bowel sounds may be auscultated over the lung fields (usually the left side). Intraabdominal bleeding would present with a rigid, distended abdomen; it would not explain this patient's abnormal chest and abdominal movement.

What type of injury occurs when a joint is twisted or stretched beyond its normal range of motion, resulting in swelling but no deformity? A) Sprain B) Strain C) Fracture D) Dislocation

A) Sprain A sprain occurs when a joint is twisted or stretched beyond its normal range of motion. As a result, the supporting capsule and ligaments are stretched or torn, resulting in injury to the ligaments, articular cartilage, synovial membrane, and tendons crossing the joint. Signs of a sprain include pain, swelling, ecchymosis, and increased motion of the joint. While a sprain is considered to be a partial dislocation, it is not associated with deformity. A dislocation is a disruption of a joint in which the bond ends are no longer in contact. The supporting ligaments are often torn, usually completely, allowing the bone ends to separate from each other; this results in deformity of the joint. A strain (pulled muscle) is a stretching or tearing of the muscle and/or tendon, causing pain, swelling, and bruising of the soft tissues in the area; deformity does not occur with a strain. A fracture is a break in the continuity of the bone. Non-displaced fractures may not be associated with deformity, whereas displaced fractures typically are. It takes radiographic evaluation to definitely determine the type of musculoskeletal injury a patient has; therefore, the EMT should treat any musculoskeletal injury as though an underlying fracture is present.

A 32-year-old man who was stung by a bee has diffuse hives, facial swelling, and difficulty breathing. When he breathes, you hear audible stridor. What does this indicate? A) Swelling of the upper airway structures B) Swelling of the lower airway structures C) Narrowing of the two mainstem bronchi D) Narrowing of the bronchioles in the lungs

A) Swelling of the upper airway structures This patient is experiencing a severe allergic reaction (anaphylaxis). Strid or, which is a high-pitched sound heard on inhalation, indicates swelling of the structures and tissues of the upper airway. If not promptly treated, the patient's airway may close completely, resulting in respiratory arrest. Narrowing of the bronchioles in the lungs causes wheezing, a whistling sound that may be heard during inhalation, exhalation, or both.

Which of the following clinical signs would indicate increased sympathetic nervous system activity? A) Tachycardia B) Hypotension C) Constricted pupils D) Slow respiratory rate

A) Tachycardia The autonomic nervous system is comprised of the sympathetic and parasympathetic nervous systems. The sympathetic nervous system, also called the "fight or flight" system, is responsible for making adjustments that allow the body to compensate for increased metabolic demand (ie, fear, shock, fever, etc.). The sympathetic nervous system increases the heart rate, increases respiratory rate and depth, dilates blood vessels in the muscles, constricts blood vessels in the digestive system, and causes pupillary dilation. Therefore, increased sympathetic activity would be expected to cause hypertension, tachycardia, and tachypnea. By contrast, the parasympathetic nervous system, also called the "rest and digest" system, is responsible for slowing the respiratory and heart rates, constricting blood vessels in the muscles, dilating the blood vessels in the digestive system, and constricting the pupils.

Which of the following assessment findings should alert the EMT that a patient with a closed lower extremity fracture is developing compartment syndrome? A) The pain is greater than one would expect for the injury. B) The extremity becomes increasingly warmer and pinker. C) The pain subsides during passive stretch of the extremity. D) Distal pulses are bounding and reflexes are hyperactive.

A) The pain is greater than one would expect for the injury. Compartment syndrome develops when edema and swelling result in increased pressure in the compartment between the fascia and muscle, the osteofascial compartment. Because the fascia is limited in the amount it can stretch or expand, pressure increases within the compartment, which in turn interferes with circulation. Compartment syndrome commonly develops in the extremities and may occur in conjunction with open or closed injuries or when swelling occurs under restrictive immobilization devices such as a cast. The hallmark sign of developing compartment syndrome is disproportionate pain; in other words, the pain experienced by the patient is greater than one would expect. In addition, the pain worsens during passive stretching of the extremity. Other signs of compartment syndrome include pallor of the affected extremity, numbness and tingling (parasthesia), and weakening or an absence of distal pulses. Compartment syndrome is a time-sensitive emergency that requires prompt surgical intervention.

Which of the following is a complication associated with renal failure? A) Uremia B) Cystitis C) Renal calculi D) Low potassium

A) Uremia The kidneys play a major role in maintaining homeostasis, a stable internal environment. They preserve balance in the body by eliminating waste from the blood. When the kidneys fail, the patient loses the ability to excrete waste from the body, leading to a condition called uremia. This means that the waste product, urea, which is normally excreted into the urine, remains in the blood. Potassium is also eliminated in the urine; if the kidneys fail, the blood level of potassium case rise to life-threatening levels. Renal calculi (kidney stones) can block the passage or urine, causing injury to the kidneys; however, they are not caused by renal failure. Inflammation of the urinary bladder is called cystitis; while this can cause a urinary tract infection severe enough to injure the kidneys, it is not caused by renal failure.

By which of the following mechanisms does nitroglycerin relieve cardiac- related chest pain or discomfort? A) Vasodilation and increased myocardial oxygen supply B) Vasodilation and decreased myocardial oxygen supply C) Vasoconstriction and increased myocardial workload D) Vasoconstriction and increased cardiac oxygen demand

A) Vasodilation and increased myocardial oxygen supply Nitroglycerin is a vasodilator that causes smooth muscle relaxation. Smooth muscle is found within the walls of the blood vessels. Nitroglycerin causes va odilation, including dilation of the coronary arteries, which in turn increases the flow of oxygenated blood to the heart and reduces myocardial workload. However, care must be taken when administering nitroglycerin to a patient. Because of its vasodilatory effects, nitroglycerin can cause hypotension; therefore, it should not be given to patients with a systolic blood pressure of less than 100 mm Hg.

A 5-year-old, 40-pound child was bitten by fire ants and is semiconscious. His breathing is labored and shallow, he has audible stridor, and his blood pressure is low. Which of the fo llowing would be the MOST appropriate treatment for him? A) Ventilation with a bag-mask device and 0.15 mg epinephrine via auto-injector B) Oxygen via nonrebreathing mask and 0.3 mg epinephrine via auto-injector C) Ventilation with a bag-mask device and 0.3 mg epinephrine via auto-injector D) Oxygen via nonrebreathing mask and 0.15 mg epinephrine via auto-injector

A) Ventilation with a bag-mask device and 0.15 mg epinephrine via auto-injector The child is clearly in anaphylactic shock with inadequate ventilation. Therefore, you should assist his ventilations with a bag-mask device and administer epinephrine as soon as possible. The correct dose for a child who weighs between 15 and 30 kg (33 to 66 pounds) is 0.15 mg via EpiPen Jr. Patients who weigh more than 30 kg (66 pounds) should receive 0.3 mg of epinephrine (adult EpiPen).

After restoring a pulse in a cardiac arrest patient, you begin immediate transport. While en route to the hospital, the patient goes back into cardiac arrest. You should: A) advise your partner to stop the ambulance. B) begin CPR and proceed to the hospital. C) begin rescue breathing with a bag-mask device. D) analyze the patient's rhythm with the AED.

A) advise your partner to stop the ambulance. If you restore a pulse in a cardiac arrest patient, the AED pads should remain attached to the patient's chest during transport in case cardiac arrest reoccurs en route to the hospital. You should, however, turn the AED off or disconnect the pads from the AED. If cardiac arrest reoccurs, you should immediately tell your partner to stop the ambulance and assist you as you begin CPR. Remember that the AED will not analyze the cardiac rhythm if the patient is moving. Once your partner is avai lable to assist, you should analyze the cardiac rhythm, defibrillate if indicated, and immediately resume CPR. Contact medical control as soon as possible, but not before performing CPR and defibrillation.

During delivery of the head, it is noted that the umbilical cord is wrapped tightly around the baby's neck. An attempt to slide the cord over the baby's head is unsuccessful. The EMT should: A) clamp and cut the umbilical cord. B) position the mother on her left side. C) gently pull on the cord to loosen it. D) place an oxygen mask on the baby.

A) clamp and cut the umbilical cord. In most cases, a nuchal cord (cord wrapped around the baby's neck) can easily be slid over the baby. However, if this is not possible, you should clamp and cut the cord and continue with the delivery. A tightly-wrapped cord around the neck can cause (or worsen) fetal hypoxia. Do NOT pull on the umbilical cord as doing so could cause it to tear and bleed; this would be bad! Positioning the mother on her left side will do nothing if the cord is strangling the baby. Administer supplemental oxygen to the mother as needed, and transport without delay.

An 88-year-old woman experienced a syncopal episode. She is now conscious, but reports dizziness and generalized weakness. Her pulse is 170 beats/min and irregular. The EMT should recognize that t his patient's syncope was MOST likely caused by: A) an acute cardiac dysrhythmia. B) increased parasympathetic tone. C) widespread systemic vasodilation. D) decreased sympathetic tone.

A) an acute cardiac dysrhythmia. Because her heart is beating so fast, you should suspect that her syncope occurred because of a drop in her blood pressure secondary to reduced ventricular filling time; the faster the heart beats, the less time there is for the ventricles to fill in between contractions. A fast, irregular pulse indicates the presence of a cardiac dysrhythmia. The sympathetic nervous system is responsible for increasing the heart rate, whereas the parasympathetic nervous system is responsible for slowing it down. Therefore, if parasympathetic nervous system function was increased or sympathetic nervous system function was decreased, you would expect the patient to experience bradycardia, not tachycardia.

Rapid transport of a patient who ingested a large dose of Tylenol is important because: A) an antidote may prevent liver failure if administered early enough. B) activated charcoal cannot be given to patients who ingested Tylenol. C) it takes only a small dose of Tylenol to cause cardiopulmonary arrest. D) liver failure usually occurs within 6 hours following a Tylenol overdose.

A) an antidote may prevent liver failure if administered early enough. Acetaminophen (APAP), the active ingredient in Tylenol, is a safe drug if taken as directed. However, ingestion of more than 140 mg/kg in an adult can cause liver failure and death. Symptoms of APAP overdose do not present acutely; it can take up to a week before signs of liver failure are apparent. The antidote for acetaminophen poisoning is acetylcysteine (Acetadote, Mucomyst), which is given only at the hospital. However, it must be given promptly if liver failure is to be avoided. Activated charcoal can be given to patients with APAP overdose; however, it is effective only if the drug is still in the stomach. After 1 to 2 hours following ingestion, activated charcoal would likely be ineffective.

The myocardium receives its blood supply from the coronary arteries that branch directly from the: A) aorta. B) vena cavae. C) left atrium. D) right ventricle.

A) aorta. The aorta, which is the largest artery in the human body, originates immediately from the left ventricle. The coronary arteries branch directly off of the ascending aorta, thus allowing the myocardium to receive blood that has the highest concentration of oxygen. The vena cavae (superior and inferior) return oxygen-poor blood from the body back to the right atrium, where it is pumped to the right ventricle and then to the lungs. The left atrium receives freshly oxygenated blood from the lungs and then pumps it to the left ventricle, through the aorta, and to the body.

The coronary arteries fi ll with blood when the: A) aortic valve closes B) left ventricle contracts. C) aortic valve opens. D) right ventricle contracts.

A) aortic valve closes During systole (cardiac contraction), the aortic valve opens and covers the openings to the coronary arteries. During diastole (cardiac relaxation), the aortic valve closes; this causes blood to flow back down the aorta and fill the coronary arteries. Therefore, coronary perfusion occurs when the heart relaxes.

A young man has an open abdominal wound through which a small loop of bowel is protruding. There is minimal bleeding. The BEST way to treat his injury is to: A) apply a sterile trauma dressing moistened with sterile saline directly to the wound and secure the moist dressing in place with a dry sterile dressing. B) apply dry sterile gauze pads to the wound and then keep them continuously moist by pouring sterile saline or water on them throughout transport. C) gently clean the exposed loop of bowel with warm sterile saline, carefully replace it back into the wound, and cover it with a dry sterile dressing. D) cover the wound with a dry sterile trauma dressing and tightly secure it in place by circumferentially wrapping roller gauze around the abdomen.

A) apply a sterile trauma dressing moistened with sterile saline directly to the wound and secure the moist dressing in place with a dry sterile dressing. An abdominal evisceration occurs when a loop of bowel, an organ, or fat protrudes through an open abdominal injury. Never try to replace an organ that is protruding from an open abdominal wound, whether it is a small fold of peritoneum or nearly all of the intestines; this significantly increases the risk of infection. Instead, cover it with sterile gauze pads or a sterile trauma dressing moistened with sterile saline and secure the moist dressing in place with a dry sterile dressing. Some EMS protocols call for an occlusive dressing over the organs, secured by trauma dressings. Do not apply excessive pressure when dressing and bandaging the wound; this may force the protruding organ or loop of bowel back into the abdominal cavity.

In addition to chest pain or discomfort, a patient experiencing an acute coronary syndrome would MOST likely present with: A) ashen skin color, diaphoresis, and anxiety. B) severe projectile vomiting and flush ed skin. C) irregular breathing and low blood pressure. D) profound cyanosis, dry skin, and a headache.

A) ashen skin color, diaphoresis, and anxiety. Chest pain, pressure, or discomfort is the most common symptom of acute coronary syndrome (ACS; eg, unstable angina, acute myocardial infarction); it occurs in approximately 80% of cases. Patients with ACS are usually anxious and may have a feeling of impending doom. Nausea and vomiting are common complaints; however, projectile vomiting, which is typically associated with increased intracranial pressure, is uncommon. The skin is often ashen gray and clammy (diaphoretic) because of poor cardiac output and decreased perfusion. Less commonly, the patient's skin is cyanotic. Respirations are usually unlabored unless the patient has congestive heart fai lure, in which case respirations are rapid and labored; irregular breathing, however, is not common. Blood pressure may fall as a resu lt of decreased cardiac output; however, most patients will have a normal or elevated blood pressure. If the patient complains of a headache, it is usually a side effect of the nitroglycerin he or she took before your arrival; ACS itself usually does not cause a headache.

Which of the following patients with diabetes is the BEST candidate for oral glucose? A) confused patient who has cool, clammy skin B) A confused patient with suspected hyperglycemia C) A semiconscious patient with pale, clammy skin D) An unresponsive patient who took too much insulin

A) confused patient who has cool, clammy skin Oral glucose is indicated for patients with diabetes who are in insulin shock (hypoglycemic crisis) and for any patient with symptomatic hypoglycemia (as documented by glucometer). The patient must be conscious and alert enough to be able to swallow the glucose, which comes in a tube of gel. If the patient is unresponsive or otherwise unable to swallow the glucose, you should provide rapid transport, providing the appropriate airway management en route, and consider an ALS intercept.

You respond to a baseball field for a person who was struck by lightning. When you arrive, you see one patient who appears confused and is ambulatory; a second patient who is conscious, sitting on the ground, and holding his arm; and a third patient who is supine and motionless. After requesting additional responders, you should: A) assess the motionless victim and perform CPR and defibrillation if necessary. B) proceed to the ambulatory patient, sit him down, and assess his mental status. C) treat the conscious patients because the motionless patient is likely deceased. D) assess and begin treating the sitting patient first because he is obviously injured.

A) assess the motionless victim and perform CPR and defibrillation if necessary. The process of triaging multiple victims of a lightning strike is different than the conventional triage methods used during a mass-casualty incident. When a person is struck by lightning, respiratory or cardiac arrest, if it occurs, usually occurs immediately. Those who are conscious following a lightning strike are much less likely to develop delayed respiratory or cardiac arrest; they usually survive. Therefore, you should focus your efforts on those who are in respiratory or cardiac arrest. This process, called reverse triage, differs from conventional triage, where such patients would ordinarily be classified as deceased.

When an error occurs while using the AED, it is MOST often the result of: A) battery failure secondary to operator error. B) malfunction of the microchip inside the AED. C) misinterpretation of the patient's cardiac rhythm. D) excess patient movement during the analyze phase.

A) battery failure secondary to operator error. AEDs are highly sensitive and specific in recognizing shockable rhythms (eg, V-Fib, pulseless V-Tach); this means that they are highly reliable. It would be extremely rare for the AED to recommend a shock when one is not indicated or fail to recommend a shock when one is indicated. When an error does occur, it is usually the operator's fault. The most common error is not having a charged battery. To avoid this problem, many AEDs are equipped with an alarm that warns the operator if the battery is not fu lly charged. Some of the older AEDs, however, are not equipped with this feature. Therefore, it is important to check the AED daily, exercise the battery as often as the manufacturer recommends, and always have a spare, fully charged battery on hand.

Assessment of a patient with multisystem trauma reveals decerebrate posturing, rapid irregular breathing, and bradycardia. These clinical signs indicate injury to the: A) brainstem. B) myocardium. C) temporal lobe. D) thoracic spine.

A) brainstem. Posturing, either decorticate (flexor) or decerebrate (extensor), is an ominous sign in a patient with a head injury because it indicates significant intracranial pressure. Posturing in conjunction with an abnormal breathing pattern (ie, central neurogenic hyperventilation, Cheyne-Stokes breathing, ataxic breathing) indicates injury to the brainstem. Cushing triad (hypertension, bradycardia, abnormal breathing) is also representative of significant intracranial pressure. Temporal lobe injuries often manifest with loss of fine motor control. In order to posture, the spinal cord must be able to receive signals from the brain; therefore, a thoracic spine injury is unlikely. Myocardial injury would be more likely to present with signs of shock and possibly cardiac dysrhythmias.

A 58-year-old man is found unresponsive by his wife. According to his wife, he was complaining of a "dull ache" in his chest the day before, but refused to allow her to call 911. His blood pressure is 70/50 mm Hg, his pulse is 120 beats/min and weak, and his respirations are 28 breaths/min and labored. Further assessment reveals that his skin is cool, pale, and clammy. You should suspect: A) cardiogenic shock. B) hypovolemic shock. C) acute septic shock. D) pulmonary embolism.

A) cardiogenic shock. The patient in this scenario likely experienced an acute myocardial infarction the day before. However, because he refused medical care, the infarction has likely damaged a significant portion of his heart, resulting in cardiogenic shock. Cardiogenic shock occurs when the heart fai ls as a pump and can no longer meet the metabolic needs of the body; it has a very high mortality rate. The patient's labored breathing is likely caused by pulmonary edema, which occurs when blood backs up into the lungs because the heart cannot effectively pump it forward . There is no evidence of hypovolemia (ie, poor skin turgor) or sepsis (ie, fever). A pulmonary embolism typically presents with an acute onset of pleuritic (sharp) chest pain and difficulty breathing, and is also commonly associated with cyanosis.

You receive a call for a 3-month-old infant who was found unresponsive by his mother when she woke up. The infant is pulseless and apneic and his skin is cold and pale. You should: A) carefully inspect the environment in which the infant was found. B) attempt full resuscitative measures, and transport the infant to the hospital. C) perform CPR for 10 minutes and then contact medical control for further direction. D) withhold chest compressions but apply the AED to analyze the infant's cardiac rhythm.

A) carefully inspect the environment in which the infant was found. When managing a case of sudden unexpected infant death (SUID), you will be faced with three tasks: assessment and management of the infant, communicating with and providing emotional support to the family, and assessing the scene. It is clear that the infant in this scenario is deceased; therefore, resuscitation (in any form) would be futile and therefore is not indicated. Be very clear (yet tactful and empathetic) when advising the parents or caregivers of this. When assessing the scene, you should note the position in which the infant was found, any signs that suggest the infant was recently ill (eg, medications, humidifiers), and the general condition of the house (ie, clean or dirty). Assessment of the scene is an important part of the overall investigation, along with a post-mortem and review of the infant's medical history, in attempting to determine the cause of death.

A 42-year-old man has a large knife impaled in the center of his chest. He is unresponsive, pulseless, and apneic. You should: A) carefully remove the knife, control the bleeding, and begin CPR. B) carefully remove the knife, control the bleeding, and apply the AED. C) secure the knife in place with a bulky dressing and transport immediately. D) stabilize the knife with bulky dressings, begin CPR, and transport at once.

A) carefully remove the knife, control the bleeding, and begin CPR. Generally, impaled objects should be stabilized in place and not removed; however, if they interfere with the patient's airway or your ability to perform CPR, they must be carefully removed. The knife in this patient is impaled in the center of his chest (the precordium), which is where chest compressions are performed. Carefully remove the knife, control any external bleeding, begin CPR, and transport immediately. The AED is generally not indicated for victims of traumatic cardiac arrest. Massive blood loss is the most common cause of traumatic cardiac arrest, not a cardiac dysrhythmia.

After administering epinephrine to a woman in anaphylactic shock, her condition improves. The EMT should recognize that her clinical improvement is because epinephrine: A) constricts the blood vessels and dilates the bronchioles. B) decreases the heart rate, which improves cardiac output. C) blocks histamine receptors and stops the allergic reaction. D) causes fluid to shift back into the cells, which reduces edema.

A) constricts the blood vessels and dilates the bronchioles. The life-threatening complications of anaphylactic shock are caused by vasodilation, which results in hypo tension, and bronchoconstriction, which results in respiratory failure. Epinephrine reverses these effects because it stimulates alpha and beta receptors. Alpha receptor stimulation produces vasoconstriction, which increases the blood pressure. Beta receptor stimulation causes bronchodilation, which improves breathing. Epinephrine increases the heart rate, also through its beta stimulation properties. Epinephrine does not stop the allergic reaction; antihistamines (ie, diphenhydramine [Benadryl]), which bind to histamine receptors, actually terminate the reaction. Antihistamines also reduce the edema that occurs during a severe allergic reaction.

Your partner has applied the AED to a cardiac arrest patient and has received a shock advised message. While the AED is charging, you should: A) continue chest compressions until your partner tells you to stand clear. B) perform rescue breathing only until the AED is charged and ready to shock. C) cease all contact with the patient until the AED has delivered the shock. D) retrieve the airway equipment and prepare to ventilate the patient.

A) continue chest compressions until your partner tells you to stand clear. It is important to minimize interruptions in CPR, especially chest compressions, when at all possible. All contact with the patient must cease while the AED is analyzing. However, if the AED gives a shock advised message and begins charging, you should resume chest compressions until the AED is charged and ready to deliver the shock; at this point, you should cease contact with the patient. As soon as the AED delivers the shock, immediately resume CPR starting with chest compressions.

Because of the property of automaticity, cardiac muscle cells are able to: A) contract spontaneously without a stimulus from a nerve source. B) rest for up to 2 minutes before initiating another electrical impulse. C) survive for long periods of time if oxygen does not reach the heart. D) generate an electrical impulse from the same pacemaker every time.

A) contract spontaneously without a stimulus from a nerve source. Automaticity refers to the ability of the cardiac cells to spontaneously generate an electrical impulse without being stimulated by an outside nerve source. When cardiac muscle cells contract, so does the muscle. Clearly, the heart cannot rest for 2 minutes before initiating another impulse; extended "rest" of the heart is called cardiac arrest. Automaticity does not enable the heart to survive for any length of time without oxygen. The property of automaticity is present in every cardiac cell; it does not dictate where the common pacemaker site is located.

You are called to a local nightclub for an injured patient. Law enforcement personnel have secured the scene. Upon arrival, you see a young man who is lying on the ground screaming in pain; bright red blood is spurting from an apparent stab wound to his groin area. You should: A) control the bleeding. B) apply 100% oxygen. C) ensure an open airway. D) prevent hypothermia.

A) control the bleeding. This patient's airway is obviously patent; he is screaming in pain. Blood spurting from the groin area indicates arterial bleeding from the femoral artery. If this bleeding is not controlled immediately, the patient will die. Oxygen and other shock treatment (ie, keeping him warm) should be initiated after this lifethreatening bleeding is controlled. If you take the time to set up and administer oxygen prior to managing the bleeding, the patient will die. Base your treatment priorities on what will kill the patient first.

A patient experienced blunt chest trauma and has asymmetrical chest wall movement. This finding indicates: A) decreased air movement into one lung. B) several ribs broken in numerous places. C) shallow breathing secondary to severe pain. D) accumulation of blood in both of the lungs.

A) decreased air movement into one lung. Asymmetrical chest wall movement, when one side of the chest moves less than the other, indicates decreased air movement into one lung (eg, pneumothorax, hemothorax). Bleeding into both lungs and shallow breathing due to severe pain would likely cause decreased movement to both sides of the chest. If more than two ribs are fractured in several places, a free-floating (flail) segment of fractured ribs is created. This flail segment (not necessarily an entire half of the chest) collapses during inhalation and bulges during exhalation; this is called paradoxical chest movement.

In contrast to the contractions associated with true labor, Braxton-Hicks contractions: A) do not increase in intensity and are alleviated by a change in position. B) may be intensified by activity and are accompanied by a pink discharge. C) generally follow rupture of the amniotic sac and occur with regularity. D) consistently become stronger and are not alleviated by changing position.

A) do not increase in intensity and are alleviated by a change in position. During pregnancy, the mother may experience false labor, or Braxton-Hicks contractions, in which there are contractions but they do not represent true labor. Unlike true labor contractions, Braxton-Hicks contractions do not increase in intensity, are not regular, and are typically alleviated by activity or a change in position. The contractions associated with true labor, once they begin, consistently get stronger and closer together and are regular; a change in position does not relieve the contractions. True labor is also commonly followed by, or in some cases preceded by, a rupture of the amniotic sac (bag of waters) and a pink or red vaginal discharge that is generally accompanied by mucus (bloody show).

Appropriate treatment for a patient with widespread full-thickness burns includes: A) dry, sterile dressings; and keeping the patient warm. B) dry, sterile dressings; burn ointment, and cooling the patient. C) moist, sterile dressings; and keeping the patient warm. D) moist, sterile dressings; burn ointment; and cooling the patient.

A) dry, sterile dressings; and keeping the patient warm. Treatment of a patient with full -thickness burns includes high-flow oxygen (or assisted ventilations if needed); dry, sterile dressings; thermal management (keep the patient warm); and providing rapid transport. Moist, sterile dressings should not be applied to full -thickness burns, as they increase the risks of hypothermia and infection. Do not apply ointments, creams, or any other substance to the burn; this will just have to be removed at the hospital and may increase the risk for infection.

When caring for a patient with an acute behavioral crisis, your primary concern should be: A) ensuring your own and your partner's safety. B) providing safe transport to the hospital. C) gathering all of the patient's medications. D) obtaining a complete past medical history.

A) ensuring your own and your partner's safety. When caring for a patient with a behavioral or emotional crisis, your primary concern should be for your own personal safety as well as your partner's safety. Your ultimate goal is to transport the patient to the hospital safely. Gather as much medical history information as possible, but keep in mind that many patients experiencing an emotional or behavioral crisis will not readily provide this information. It is important to remember that patients with emotional or behavioral crises may appear calm initially; however, there is always the potential for them to turn violent.

General guidelines when assessing a 2-year-old child with abdominal pain and adequate perfusion include: A) examining the child in the parent's arms. B) palpating the painful area of the abdomen first. C) placing the child supine and palpating the abdomen. D) separating the child from the parent to ensure a reliable examination.

A) examining the child in the parent's arms. If the child's condition is stable, the parent should be allowed to hold the child during the examination. This will minimize anxiety in the child and will make the assessment easier for you. In general, you should avoid separating the child and parents unless the child's condition warrants it. When assessing the abdomen of any patient, you should palpate the most painful area last.

Following blunt trauma to the chest, a 33-year-old man has shallow, painful breathing. On assessment, you note that an area to the left side of his chest collapses during inhalation and bulges during exhalation. These are signs of a/an: A) flail chest. B) pneumothorax. C) isolated rib fracture. D) pulmonary contusion.

A) flail chest. If two or more ribs are fractured in two or more places or if the sternum is fractured along with several ribs, a segment of the chest wall may be detached from the rest of the thoracic cage. This injury is called a flail chest. In a flail chest, the detached portion of the chest wall moves opposite of normal; that is, it moves in during inhalation and out during exhalation (paradoxical motion). Isolated (single) rib fractures are not associated with paradoxical motion because they are usually fractured in only one place. In a pneumothorax, the patient's respirations are often labored; in severe cases, an entire side of the chest may not move at all (asymmetrical chest movement). A pulmonary contusion (bruising of the lung tissue) does not cause paradoxical chest motion unless associated with a flail chest.

Signs and symptoms of pelvic inflammatory disease include: A) generalized lower abdominal pain, nausea, and fever. B) pain around the umbilicus and heavy vaginal bleeding. C) abdominal cramping and an odorless vaginal discharge. D) upper abdominal pain, diarrhea, and an absence of fever.

A) generalized lower abdominal pain, nausea, and fever. Pelvic inflammatory disease (PIO) is an infection of the upper female reproductive organs, specifically, the uterus, ovaries, and fallopian tubes. It occurs almost exclusively in sexually active women. In PIO, disease-causing organisms enter the vagina during sexual activity and migrate through the cervical opening and into the uterine cavity. The infection may then progress to the fallopian tubes, producing scarring that can increase the risk for ectopic pregnancy. The most common presenting symptom of PIO is generalized lower abdominal pain. Other signs and symptoms include an abnormal and often foul-smelling vaginal discharge, increased pain during sexual intercourse, fever, general malaise, and nausea and vomiting. Diarrhea and vaginal bleeding are not common signs of PIO.

You are assessing a 26-year-old woman who is 38 weeks pregnant and is in labor. She tells you that she was pregnant once before, but had a miscarriage at 19 weeks. You should document her obstetric history as: A) gravida 2, para 0. B) gravida 1, para 1. C) gravida 0, para 2. D) gravida 2, para 1.

A) gravida 2, para 0. Gravida is the term used to describe the number of times a woman has been pregnant, regardless of whether she carried the infant to term. Para is the term used to describe the number of times a woman has carried a fetus beyond 28 weeks, regardless of whether the infant was born dead or alive. Because your patient is currently pregnant and was pregnant once before, she is gravida 2. However, because she had a miscarriage with her first pregnancy (she did not carry beyond 28 weeks) and has not yet delivered the baby she is currently carrying, she is para 0. When she delivers, she will become gravida 2 and para 1.

A 3-year-old child presents with respiratory distress. The child is being held by her mother and does not react to your presence. This finding indicates that the child: A) has severe hypoxia. B) probably is sleeping. C) is afraid of your presence. D) is reacting normally for her age.

A) has severe hypoxia. A 3-year-old child typically is very attentive to his or her surroundings, especially when a stranger enters the environment. The fact that the child does not acknowledge your presence is an abnormal (and ominous) sign and indicates significant hypoxia. Therefore, you must treat aggressively (ie, ventilatory support) to prevent respiratory arrest and subsequent cardiac arrest.

You are responding to a call for a 2-year-old child who fell from a second-story window. With the mechanism of injury and the age of the patient in mind, you should suspect that the primary injury occurred to the child's: A) head. B) chest. C) abdomen. D) lower extremities.

A) head. Because a child's head is proportionately larger than the rest of the body when compared to an adult, the head commonly is the primary site of injury in pediatric patients. This is especially true in fall - related injuries, in which gravity causes the head to precede the rest of the body. Head injury is a leading cause of traumatic death in infants and small children.

A man was stabbed in the right side of the chest, lateral to the nipple. He is tachypneic, tachycardic, and diaphoretic. His jugular veins are collapsed and breath sounds are difficult to hear on the right side. You should suspect a: A) hemothorax. B) ruptured spleen. C) liver laceration. D) pneumothorax.

A) hemothorax. Based on the injury location and the patient's clinical presentation, you should suspect a hemothorax. In addition to the lung, there is a TON of vasculature in the thoracic cavity that can easily be injured by a penetrating injury. Because the patient is losing blood, one would expect flattened or collapsed jugular veins and profound shock. In addition, because blood is filling the right hemithorax, breath sounds may be weak or absent on the affected side. Although a liver injury cannot be completely ruled out, the injury is above where the liver is located anatomically. If a pneumothorax is present, it is likely in conjunction with the hemothorax (hemopneumothorax). The spleen is located in the upper left quadrant; this patient's injury is on the right side.

You are assessing a 5-year-old boy with major trauma. His blood pressure is 70/40 mm Hg and his pulse rate is 140 beats/min. and weak. The child's blood pressure: A) indicates decompensated shock. B) reflects adequate compensation. C) is appropriate based on his age. D) suggests increased intracranial pressure.

A) indicates decompensated shock. The low normal systolic blood pressure (SBP) for a child between 1 and 10 years of age is calculated by multiplying his or her age (in years) by 2 and adding 70. Using this formula, the low normal SBP for a 5-year-old child is 80 mm Hg; anything less indicates hypotension. Therefore, a SBP of 70 mm Hg in a 5-year-old child, especially in the context of major trauma, indicates decompensated shock. Patients with increased intracranial pressure are typically hypertensive, not hypotensive. High normal SBP in this same age group is calculated by multiplying his or her age (in years) by 2 and adding 90. Therefore, the high normal SBP for a 5-year-old child is 100 mm Hg.

In contrast to an incision, a laceration: A) is a jagged cut. B) is a superficial injury. C) bleeds more severely. D) usually involves an artery.

A) is a jagged cut. A laceration is a jagged cut caused by a sharp object or a blunt force that tears the tissue, whereas an incision is a sharp, smooth cut. The depth of the injury can vary; it can extend through the skin and subcutaneous tissue or into the underlying muscles and adjacent nerves and blood vessels. Lacerations and incisions can involve arteries, veins, or both, potentially resulting in severe bleeding.

Compared to nonpregnant women, otherwise healthy pregnant woman are at greater risk for: A) joint dislocations. B) type 1 diabetes. C) pathologic fractures. D) congestive heart failure.

A) joint dislocations. There are a number of anatomic and physiologic changes that occur during pregnancy. Blood volume, cardiac output, and heart rate all increase. Blood pressure, however, decreases slightly. The increased size of the uterus directly affects the respiratory system. As the uterus grows, it pushes up on the diaphragm, displacing it from its normal position. As the pregnancy continues, respiratory capacity changes, with increased respiratory rates and decreased minute volumes. You may observe that a pregnant patient breathes faster, but has a decreased ability to take a deep breath. These are all normal pregnancyrelated changes. Increased hormones affect the musculoskeletal system by making the joints looser and less stable; this increases the risk of joint dislocations. A pathologic fracture is caused by an underlying abnormal condition, such as osteoporosis or bone cancer; pregnant patients are not at risk for pathologic fractures. Changes in the cardiovascular system and the increased demands of supporting the fetus significantly increase the workload of the heart. A healthy woman's body can handle the increased cardiac workload and demand. Not all women are healthy when they begin their pregnancy, however. Cardiac compromise, including heart failure can occur if the woman has preexisting heart disease. Some pregnant women develop gestational diabetes, which is caused when the pregnancy hormones create insulin resistance. This is different from type 1 diabetes, where the body does not produce any insulin.

Hypoxia- induced unresponsiveness during a submersion injury is usually the result of: A) laryngospasm. B) water in the lungs. C) a cardiac dysrhythmia. D) associated hypothermia.

A) laryngospasm. When a patient falls into the water or becomes panicked when in the water, he or she begins to swallow large amounts of water. Even a small amount of water near the larynx can cause a spasm, which closes off the airway. This results in hypoxia, loss of consciousness, and cardiac dysrhythmias. If the patient is not removed from the water at once and treated aggressively, hypoxia and acidosis will eventually result in cardiac arrest.

Appropriate care for an amputated body part includes : A) laying the wrapped body part on a bed of ice. B) placing it directly on ice to prevent tissue damage. C) keeping the part warm to prevent cellular damage. D) keeping the part at room temperature in a plastic bag.

A) laying the wrapped body part on a bed of ice. Appropriate care for an amputated body part includes wrapping the part in a sterile dressing and placing it in a plastic bag. Follow your local protocols regarding how to preserve amputated parts. In some areas, dry sterile dressings are recommended for wrapping amputated parts; in other areas, dressings moistened with sterile saline are recommended. Put the bag in a container filled with ice. Lay the wrapped part on a bed of ice; do not pack it in ice or place it in direct contact with ice. The goal is to keep the part cool without letting it freeze or develop frostbite. Freezing may cause cellular and tissue damage, which decreases the chance of successful reattachment.

An 80-year-old woman has pain in the right upper quadrant of her abdomen and a yellow tinge to her skin. You should suspect dysfunction of the: A) liver. B) spleen. C) pancreas. D) gallbladder.

A) liver. Pain in the right upper quadrant and skin with a yellow tinge to it Uaundice) indicates a problem with the liver. Jaundice is the result of an excess level of bilirubin in the blood. Bilirubin, a yellow-colored substance, is the waste product that remains in the bloodstream after iron is removed from hemoglobin. One of the functions of the liver is to filter out waste, such as bilirubin, from the blood. If the liver is not functioning properly, bilirubin accumulates in the bloodstream. Dysfunction of the pancreas would result in possible fluctuations in the levels of blood glucose. Gallbladder inflammation (cholecystitis) typically produces right upper quadrant pain and/or referred pain to the right shoulder that occurs shortly after eating. Dysfunction of the spleen would cause left upper quadrant pain and/or referred pain to the left shoulder.

Shortly after assisting a 60-year-old woman with her second nitroglycerin treatment, she tells you that she is lightheaded and feels as if she is going to faint. Her symptoms are MOST likely due to: A) low blood pressure. B) an irregular heartbeat. C) nervousness and anxiety. D) a drop in her blood sugar.

A) low blood pressure. Nitroglycerin (NTG) is a vasodilator; as such, it may cause a drop in blood pressure (hypotension) in some patients. Signs and symptoms of hypotension include dizziness, lightheadedness, and fainting (syncope), among others. For this reason, you should always assess a patient's blood pressure before and after administering nitroglycerin. If the patient's systolic BP is less than 100 mm Hg, NTG should not be given. The patient is probably nervous and anxious, and may even have an irregular heartbeat; however, hypotension is a more likely cause of her symptoms. NTG does not affect a patient's blood sugar level.

A woman who is 39 weeks' pregnant is unresponsive, apneic, and pulseless. When treating her, the EMT should: A) manually displace her uterus to the left. B) ventilate her at a rate of 20 breaths/min. C) ensure that she is positioned on her left side. D) delay defibrillation until ALS is at the scene.

A) manually displace her uterus to the left. During the later stages of pregnancy, a supine position should be avoided. The pregnant uterus can compress the aorta and inferior vena cava (lVC), thus impairing blood flow (aortocaval compression). In this case, however, the patient is in cardiac arrest; she must remain supine in order to perform effective CPR. Therefore, if the uterine fund us (top of the uterus) is above the level of the umbilicus (likely the case at 39 weeks), the EMT should manually displace her uterus to the left to relieve pressure off the IVC. The chest compression and ventilation rates are no different for pregnant patients than they are for non-pregnant patients. Do not delay defibrillation, if it is indicated.

Common complaints in a patient with an infectious disease include: A) nausea, rash, fever, and shortness of breath. B) headache, low back pain, and arm numbness. C) chest discomfort, weakness, and vomiting. D) joint pain, muscle aches, and blurred vision.

A) nausea, rash, fever, and shortness of breath. Nausea is common with a number of infectious diseases, such as hepatitis. A rash should alert you to the possibility of sepsis, such as what could be caused by meningitis. Petechiae are tiny, circular, non-raised patches that appear on the skin or in a mucous membrane; they occur as the result of bleeding under the skin. A petechial rash may be present in a patient who is septic from a number of causes. Fever indicates inflammation or infection; in the context of other symptoms (ie, rash, nausea), you should be suspicious of an infectious disease. Shortness of breath can occur in patients with diseases such as tuberculosis or severe acute respiratory syndrome coronavirus 2 (SARS-CoV-2), the virus that causes coronavirus disease 2019 (COVID-19). Arm numbness, chest discomfort, and blurred vision are not common symptoms of an infectious disease.

A 4-year-old girl fell from a third-story window and landed on her head. She is semiconscious with slow, irregular breathing and is bleeding from her mouth and nose. You should: A) open her airway with the jaw-thrust maneuver while manually stabilizing her head, suction her oropharynx, and assist her ventilations. B) open her airway by carefully tilting her head back, suction her oropharynx, and administer high-flow oxygen via nonrebreathing mask. C) manually stabilize her head, open her airway with the jaw-thrust maneuver, insert a nasopharyngeal airway, and suction her oropharynx. D) suction her oropharynx, open her airway with the jaw-thrust maneuver, insert an oropharyngeal airway, and assist her ventilations.

A) open her airway with the jaw-thrust maneuver while manually stabilizing her head, suction her oropharynx, and assist her ventilations. In any semiconscious or unconscious patient with a head injury, you should manually stabilize the head and open the airway with the jaw-thrust maneuver. If there are any secretions in the mouth, suction the oropharynx. If possible, insert a simple airway adjunct. The patient in this scenario is semiconscious and likely has an intact gag reflex; therefore, you should not attempt to insert an oropharyngeal airway. Conversely, you should not insert a nasopharyngeal airway in patients with a head injury, especially if there is fluid or blood draining from the nose (a sign of a skull fracture). After ensuring a patent airway, you should turn your attention to the patient's breathing. Slow, irregular breathing will not provide adequate minute volume and should be treated with ventilatory assistance.

When assessing distal circulation in a patient with a swollen deformed femur, you should: A) palpate for a dorsalis pedis pulse. B) assess the pulse behind the knee. C) touch his foot with a blunt object. D) ask the patient to wiggle his toes.

A) palpate for a dorsalis pedis pulse. Care for a musculoskeletal injury includes assessing distal circulatory, sensory, and motor functions before and after applying a splint. In the case of a femur injury, the dorsalis pedis (pedal) pulse, located on top of the foot, is the most distal pulse relative to the injury. If a pedal pulse can be palpated, circulation distal to the injury is present. The pop Ii teal pulse is located behind the knee; it is proximal to the pedal pulse. Touching the patient's foot and asking him if he can feel it and asking him to wiggle his toes are assessing sensory and motor functions , respectively, not circulatory function.

At the end of ventricular relaxation, the left ventricle contains 110 mL of blood. This is referred to as the: A) preload. B) afterload. C) stroke volume. D) cardiac output.

A) preload. Preload is the amount of pressure on the ventricular wall at the end of ventricular relaxation (diastole) and is influenced by the volume of blood in the ventricle just before it contracts. Afterload refers to the resistance that the ventricles must contract against. A patient with hypertension, for example, would have an increased afterload due to systemic vasoconstriction; the smaller the arteries, the greater the resistance the heart must contract against. Stroke volume is the volume of blood ejected from the ventricles in a single beat. Cardiac output is the volume of blood pumped by the heart each minute; it is calculated by multiplying the stroke volume and heart rate.

Aspirin is beneficial to patients experiencing an acute coronary syndrome because it: A) prevents a clot from getting larger. B) effectively relieves their chest pain. C) decreases cardiac workload by lowering the BP. D) destroys the clot that is blocking a coronary artery.

A) prevents a clot from getting larger. Early administration of baby aspirin (160 to 325 mg) to patients with acute coronary syndrome (ACS) has clearly been shown to reduce mortality and morbidity. Aspirin (acetylsalicylic acid [ASA]) prevents the clot in a coronary artery from getting larger by inhibiting platelet aggregation; in other words, it makes the platelets less sticky, which means that they will be less likely to clump together. Aspirin does not relieve the chest pain or discomfort associated with ACS, nor does it reduce blood pressure. Furthermore, aspirin does not remove the clot that is blocking a coronary artery; cardiac catheterization or fibrinolytic (clot-buster) drugs are required to do this.

A man is experiencing a severe allergic reaction after being stung by a scorpion. He does not have his own epinephrine; however, his wife is allergic to bees and has a prescribed epinephrine auto-injector. You should: A) provide rapid transport and consider an ALS intercept. B) assist the patient with the wife's prescribed epinephrine. C) administer an oral antihistamine and transport without delay. D) request an ALS unit to respond to the scene to administer epinephrine.

A) provide rapid transport and consider an ALS intercept. If a patient does not have a prescribed epinephrine auto-injector and is experiencing a severe allergic reaction, you should administer supplemental oxygen, assist the patient's ventilations if needed, and transport without delay. Closely monitor the patient's airway and breathing status en route and coordinate an ALS intercept if possible. If you carry an epinephrine auto-injector on your ambulance and your protocols allow you to administer it, do so without delay. Otherwise, the ALS unit will be able to administer epinephrine via the intramuscular or intravenous route. Never assist a patient with a medication that is not prescribed to him or her specifically. Antihistamines are given after epinephrine; in most systems, EMTs are not authorized to dispense oral medications other than glucose and aspirin.

All of the following are components of the Cincinnati Stroke Scale, EXCEPT: A) pupil size. B) facial droop. C) speech pattern. D) arm movement.

A) pupil size. The Cincinnati Stroke Scale, which tests speech, facial droop, and arm drift, is a reliable tool that should be used during your assessment of a patient suspected of having had a stroke. To test speech, ask the patient to repeat a simple phrase; he or she should be able to repeat the phrase without slurred speech. If the patient cannot repeat the phrase, or repeats it with slurred speech, a stroke should be assumed. To test facial movement, ask the patient to smile and show his or her teeth; both sides of the face should move symmetrically (equally). If only one side of the face is moving well (facial droop), a stroke should be assumed. To test arm movement, ask the patient to hold both arms in front of his or her body, palms up, with eyes closed and without moving. Over the next 10 seconds, watch the patient's hands. If one arm drifts down toward the ground, you know that side is weak. You should assess pupillary size, equality, and reactivity in any patient with a neurologic disorder; however, this is not a part of the Cincinnati Stroke Scale.

You are dispatched to a residence for a 20-year-old man with respiratory distress. When you arrive, you find that the patient has a tracheostomy tube and is ventilator dependent. His mother tells you that he was doing fine, but then suddenly began experiencing breathing difficulty. You should: A) remove him from the mechanical ventilator and ventilate him manually. B) check the settings on the ventilator to ensure that it is functioning properly. C) detach the ventilator, suction the tracheostomy tube, and reassess the patient. D) remove the ventilator tubing and place an oxygen mask over the tracheostomy tube.

A) remove him from the mechanical ventilator and ventilate him manually. If a ventilator-dependent patient experiences a sudden onset of respiratory distress, you should first remove him or her from the mechanical ventilator and begin manual ventilation with a bag-mask device; attach the bag directly to the tracheostomy tube. If the patient improves, you will know that the problem was a malfunction with the mechanical ventilator. If the patient does not improve, the tracheostomy tube is likely plugged with thick mucus secretions and requires suctioning. Unless you are familiar with the mechanical ventilator (most EMTs are not), do not attempt to troubleshoot the device by checking the settings; this simply wastes time and could harm the patient if you do not know what you are doing.

Which of the following patients would be MOST in need of a rapid head-to-toe assessment? A) responsive 22-year-old man with a small-caliber gunshot wound to the abdomen B) A responsive 25-year-old woman who fell 9 feet from a roof and landed on her side C) A 43-year-old woman with a unilaterally swollen, painful deformity of the midshaft femur D) A 60-year-old man who fell from a standing position and has small abrasions on his cheek

A) responsive 22-year-old man with a small-caliber gunshot wound to the abdomen A rapid head-to-toe assessment is indicated for any patient with abnormal findings in the primary assessment or when the mechanism of injury warrants it. Significant mechanisms of injury include falls in the adult of greater than 15 feet (or three times the patient's height); penetrating injuries to the head, neck, chest, or abdomen; and multiple long bone fractures, among others.

You are transporting a 35-year-old man who has a history of alcoholism. He stopped drinking 4 days ago and is now disoriented, diaphoretic, and tachycardic. This patient is at risk for: A) seizures. B) hypothermia. C) hyperglycemia. D) schizophrenia.

A) seizures. A patient in alcohol withdrawal may experience frightening hallucinations or delirium tremens (DTs), a syndrome characterized by restlessness, fever, disorientation, sweating, and agitation. In severe cases, generalized seizures can occur; these seizures can be prolonged and can result in permanent brain injury or death. About 1 to 7 days after a person stops drinking or when alcohol consumption levels are suddenly decreased, DTs may develop. Patients with DTs are often dehydrated due to sweating, fluid loss, insufficient fluid intake, or vomiting, and their hallucinations can be extremely frightening. The alcoholic patient would more likely be hypoglycemic. Schizophrenia is a psychiatric illness; it is not caused by alcohol use or withdrawal.

After delivering one shock with the AED and performing 2 minutes of CPR on a woman in cardiac arrest, you reanalyze her cardiac rhythm and receive a no shock advised message. This means that: A) she is not in a shockable rhythm. B) she has electrical activity but no pulse. C) her rhythm has deteriorated to asystole. D) the first shock restored a rhythm and pulse.

A) she is not in a shockable rhythm. If the AED gives a no shock advised message, it has determined that the patient i not in a shockable rhythm (eg, V-Fib, pulseless V-Tach). It does not indicate that the patient has a pulse, nor does it indicate that a normal cardiac rhythm has been restored. The AED does not distinguish pulseless electrical activity (PEA) from asystole; it only recognizes them as nonshockable. PEA is a condition in which organized cardiac electrical activity is present despite the absence of a pulse. Asystole is the absence of all cardiac electrical and mechanical activity. If the AED gives a no shock advised message, immediately resume CPR, starting with chest compressions, until ALS arrives or the patient starts to move.

A 50-year-old woman who is conscious and alert complains of a severe migraine headache. When caring for her, you should generally avoid: A) shining a light into her pupils. B) transporting her in a supine position. C) dimming the lights in the ambulance. D) applying ice packs to her forehead.

A) shining a light into her pupils. Patients with migraine or cluster headaches typically have photophobia (light sensitivity). Any type of bright light, especially if shone directly into the eyes, will cause the patient with a headache unnecessary severe pain. Dimming the lights in the ambulance and making the patient as comfortable as possible are the treatments of choice for a patient with a headache. Some patients benefit from ice packs applied to the forehead; just be sure to wrap the ice pack with roller gauze. Oxygen also should be administered as needed. Typically, the patient will prefer to lie supine or on the side.

Prior to your arrival at the scene, a young woman was removed from a body of water after being submerged for an unknown period of time. You should manage her airway appropriately while considering the possibility of: A) spinal injury. B) hyperthermia. C) internal bleeding. D) airway obstruction.

A) spinal injury. When caring for a patient with a submersion injury (ie, near-drowning), you should consider the possibility of a spinal injury. Many water-related incidents occur when a patient dives into shallow water and strikes his or her head. Water can be aspirated into the lungs, but will not cause an obstruction of the upper airway. Another common finding in patients with a submersion injury is hypothermia. Although it is possible for the patient to have internal bleeding at the same time, especially if he or she experienced a traumatic injury before the submersion, spinal injuries are more common.

A 40-year-old man with a history of depression and schizophrenia appears frightened and tells you that he sees snakes everywhere. You should: A) tell him that you do not see any snakes, but they are obviously scaring him. B) let him know that going to the hospital will keep him safe from the snakes. C) advise him that you suspect he has not been compliant with his medications. D) let him know that it is important for him to be transported as soon as possible.

A) tell him that you do not see any snakes, but they are obviously scaring him. When caring for any patient with a psychiatric crisis, your primary responsibility is to yourself, to stay safe. The patient in this scenario, although scared, is not violent. In cases such as this, you should be prepared to spend extra time with the patient; it may take longer to assess, listen to, and prepare the patient for transport. Just because he is experiencing an acute crisis does not mean that he has not been taking his medications; to make that assumption is merely speculation. You must be honest, reassuring, and nonjudgmental. Let the patient tell you how he or she is feeling in his or her own words, and acknowledge any auditory or visual hallucinations. Do not, however, play along with the hallucinations; this is cruel and simply reinforces that what the patient is hearing or seeing is real.

In general, you should avoid rewarming a frostbitten body part in the field if: A) the affected part could refreeze after rewarming. B) a paramedic is not present to administer analgesia. C) you are unable to obtain water that is at least 120°F. D) arrival at the emergency department will be delayed.

A) the affected part could refreeze after rewarming. Frostbitten body parts should not be rewarmed if there is a chance that they could refreeze after you have rewarmed them. If an extremity thaws and then refreezes, the amount of tissue and cellular damage may be worse than the damage caused by the initial freezing. A delay in getting the patient to the emergency department warrants rewarming. If rewarmed, the extremity should be immersed in water that is 105°F to 112°F. Analgesia would certainly be a comfort to the patient, although its absence does not negate rewarming a frostbitten body part in the field.

Oxygen and other nutrients are transferred to the developing fetus via the: A) umbilical vein. B) amniotic fluid. C) umbilical arteries. D) mother's liver.

A) umbilical vein. The organ of oxygen and carbon dioxide exchange between the mother and the developing fetus is the placenta. The fetus is attached to the placenta by the umbilical cord, which contains two arteries and one vein. The fetus receives its supply of oxygen and other nutrients from the placenta via the umbilical vein. Carbon dioxide and other waste products are returned from the fetus to the placenta via the umbilical arteries. The amniotic sac acts as a cushion for the developing fetus and helps protect it from infection.

Following return of spontaneous circulation, the patient remains apneic. The EMT should: A) ventilae at 10 breaths/min and maintain an oxygen saturation of 92% to 98% B) ventilate at 15 to 18 breaths/min and maintain an oxygen saturation of 100%. C) elevate the patient's lower extremities and ventilate at a rate of 20 breaths/min. D) remove the AED pads and vent ilate the patient at a rate of 24 breaths/min.

A) ventilae at 10 breaths/min and maintain an oxygen saturation of 92% to 98% Following return of spontaneous circulation (ROSC) in a patient who remains apneic, you should ventilate at a rate of 10 breaths/min and maintain an oxygen saturation of 92% to 98%. An oxygen saturation of 100% should not be achieved; there is evidence that such a high oxygen saturation post-ROSC can have a negative impact on neurologic recovery. Keep the patient supine, leave the AED pads in place, and transport without delay.

Assessment of a trauma patient reveals paradoxical movement to the left side of his chest. The patient is conscious, but restless, and is experiencing severe pain. His breathing is rapid and shallow and his pulse is rapid and weak. The EMT should: A) ventilate the patient with a bag-mask device. B) position the patient on the injured side and transport. C) administer high-flow oxygen via nonrebreathing mask. D) stabilize the unstable chest wall with bulky dressings.

A) ventilate the patient with a bag-mask device. The patient in this scenario has a flail chest and inadequate ventilation (ie, rapid, shallow [reduced tidal volume] breathing). A flail chest occurs when several ribs are fractured in more than one place; the result is a free-floating section of ribs (flail segment) that collapses during inhalation and bulges out during exhalation (paradoxical chest movement). As the flail segment collapses, the lung is compressed and ventilation is impaired. Treatment should include positive pressure ventilation and prompt transport. In the past, treatment included splinting of the flail segment with bulky dressings; however, restricting chest wall movement is no longer recommended and positive pressure ventilation is now preferred.

A patient whose artificial pacemaker has failed would MOST likely experience: A) weakness and bradycardia. B) hypertension and a headache. C) irreversible ventricular fibrillation. D) dizziness and excessive tachycardia.

A) weakness and bradycardia. An artificial pacemaker is implanted in a person whose own cardiac electrical conduction system cannot maintain a regular rhythm and rate. If a pacemaker stops working, as when the battery wears out or an internal lead becomes detached, the patient often experiences syncope, dizziness, or weakness because of a slow heart rate (bradycardia). The pulse rate is typically less than 60 beats/min because the heart is beating without the stimulus of the pacemaker and without regulation of its own electrical conduction system, which may be damaged. in these circumstances, the heart tends to assume a fixed slow rate that may not be fast enough to maintain adequate cardiac output. In some cases, the patient's heart rate may be so low that he or she becomes hypotensive.

Internal or external bleeding would be especially severe in a patient: A) with hemophilia. B) who takes aspirin. C) with heart disease. D) who is hypotensive.

A) with hemophilia. Hemophilia is a condition in which the patient lacks one or more of the blood's clotting factors. There are several forms of hemophilia, most of which are hereditary and some of which are severe. Sometimes bleeding occurs spontaneously in patients with hemophilia. Because the patient's blood does not clot, all injuries, no matter how minor they appear, are potentially serious. Aspirin does not destroy the blood's clotting factors; it decreases the ability of platelets to stick together. Although this may cause prolonged bleeding time, the patient with hemophilia, who lacks key clotting factors, will bleed for a much longer period of time. Many patients with heart disease take aspirin daily to prevent clot formation in a coronary artery. When blood pressure is low (hypotension), the driving force of the blood through the blood vessels is reduced; as a result, bleeding tends to be less severe relative to patients with high blood pressure. Unfortunately, hypotension indicates decompensated shock.

Which of the following questions would be MOST appropriate to ask when assessing a patient with chest pain? A) What does the pain feel like? B) Does the pain radiate to your arm? C) Would you describe the pain as sharp? D) Is the pain worse when you take a deep breath?

B) Does the pain radiate to your arm? When questioning any patient about any type of pain, you should avoid asking leading questions that can simply be answered yes or no. To obtain the most reliable assessment, open-ended questions should be asked to allow the patient to describe the quality of the pain in his or her own words.

After an advanced airway device has been inserted in a 6-month-old infant in cardiopulmonary arrest, you should deliver ventilations at a rate of: A) 8 breaths/min. B) 10 breaths/min. C) 12 breaths/min. D) 20 breaths/min.

B) 10 breaths/min. After an advanced airway device (eg, ET tube, supraglottic airway) has been inserted during cardiac arrest, ventilate the patient at a rate of 10 breaths/min (one breath every 6 seconds). This ventilation rate applies to all age groups, except the newborn. Health care providers often deliver excessive ventilation, particularly when an advanced airway device is in place. Excessive ventilation (eg, hyperventilation) is detrimental because it causes an increase in intrathoracic pressure, which impedes blood flow back to the heart and decreases coronary perfusion. Hyperventilation also increases the risks of regurgitation and aspiration in the patient who does not have an advanced airway device in place. A ventilation rate of20 to 30 breaths/min (one breath every 2 to 3 seconds) would be appropriate for an apneic infant or child who has a pulse.

The appropriate technique for performing two-rescuer CPR on a 4-year-old child includes: A) 30 compressions to 2 ventilations, compressing the chest one-third the depth of the chest, and delivering each breath over 1 second. B) 15 compressions to 2 ventilations, compressing the sternum with the heel of your hand, and ventilating until visible chest rise occurs. C) 30 compressions to 2 ventilations, compressing the sternum with the heel of both hands, and delivering each breath over 1 to 2 seconds. D) 15 compressions to 2 ventilations, compressing the sternum with your thumbs, and delivering 100 to 120 compressions per minute.

B) 15 compressions to 2 ventilations, compressing the sternum with the heel of your hand, and ventilating until visible chest rise occurs. When performing two-rescuer CPR on an infant (less than 1 year of age) or a child (1 year of age to the onset of puberty [12 to 14 years of age]), use a compression to ventilation ratio of 15:2. Compress the chest by one-third the depth of the chest (about 1.5 inches for infants; about 2 inches for children), at a rate of 100 to 120/min, and allow the chest to fully recoil in between compressions. Deliver each breath over 1 second, just enough to produce visible chest rise, and allow complete exhalation. For a child, use the heel of one or both hands to compress the chest, depending on the size of the child. For two-rescuer infant CPR, use the tips of your thumbs to compress the chest (two-thumb, encircling-hands technique); the two-finger technique may be used for onerescuer infant CPR. A 30:2 compression to ventilation ratio is used for all adult and one-rescuer CPR.

A 30-year-old man sustained partial-thickness burns to the anterior chest and both anterior arms. Based on the Rule of Nines, what percentage of his body surface area has been burned? A) 9% B) 18% C) 27% D) 36%

B) 18% According to the adult Rule of Nines, the anterior trunk (chest and abdomen) accounts for 18% of the total body surface area (TBSA) and each entire arm accounts for 9%. Therefore, the anterior chest, which is one half of the trunk, would account for 9% of the TBSA, and both anterior arms (4.5% each) would account for 9% TBSA, for a total of 18% TBSA burned.

A soft-tissue injury that results in a flap of torn skin is called a/an: A) incision. B) avulsion. C) abrasion. D) laceration.

B) avulsion. An avulsion is a soft- tissue injury in which a portion of the skin is torn away, leaving a flap of skin. A laceration is a jagged soft-tissue injury that can be caused by glass or other sharp objects. An abrasion is the scraping away of the epidermis, causing oozing of serous fluid from the capillary bed. Road rash is a classic example of an abrasion. An incision is similar to a laceration, but has smooth edges. Scalpels or knives are examples of instruments that would make an incision.

A 6-month-old infant with a head injury moans in response to pain. According to the modified Glasgow Coma Scale for infants, what score should be assigned for verbal response? A) 1 B) 2 C) 3 D) 4

B) 2 The Glasgow Coma Scale (GCS) is modified for infants (younger than 1 year of age) to accommodate their age-related behavior. Under the verbal component of the GCS, a score of 5 is assigned if the infant coos or babbles; a score of 4 is assigned if the infant has an irritable cry; a score of 3 is assigned if the infant cries in response to pain; a score of 2 is assigned if the infant moans in response to pain; and a score of 1 is assigned if there is no response at all.

A 40-year-old man has burns to the entire head, anterior chest, and both anterior upper extremities. Using the adult Rule of Nines, what percentage of his total body surface area has been burned? A) 18% B) 27% C) 36% D) 45%

B) 27% Using the adult Rule of Nines, the head accounts for 9% of the total body surface area (TBSA), the anterior chest for 9% (the entire anterior trunk [ chest and abdomen] accounts for 18%), and the anterior upper extremities for 4.5% each (each entire upper extremity is 9% of the TBSA). On the basis of this, the patient has sustained 27% TBSA burns.

In which of the fo llowing patients is nitroglycerin contraindicated? HIDE THIS MENU A) 41-year-old man with crushing substernal chest pressure, a blood pressure of160/90 mm Hg, and severe nausea B) 53-year-old man with chest discomfort, diaphoresis, a blood pressure of 146/66 mm Hg, and regular use of Levitra C) 58-year-old man with chest pain radiating to the left arm, a blood pressure of 130/64 mm Hg, and prescribed Tegretol D) 66-year-old woman with chest pressure of 6 hours' du ration, lightheadedness, and a blood pressure of 110/58 mm Hg

B) 53-year-old man with chest discomfort, diaphoresis, a blood pressure of 146/66 mm Hg, and regular use of Levitra Nitroglycerin is contraindicated in patients who do not have a prescription for nitroglycerin, in those with a systolic BP less than 100 mm Hg, and in patients who have taken medications for erectile dysfunction (ED) within the previous 24 to 48 hours. Such medications include sildenafil (Viagra), vardenafil (Levitra), and tadalafil (Cialis). Because bothED drugs and nitroglycerin cause vasodilation, concomitant use of these drugs may result in significant hypotension. Carbamazepine (Tegretol) is an anticonvulsant medication; there are no known interactions between Tegretol and nitroglycerin.

A trauma patient opens his eyes slightly and moans when the EMT applies a painful stimulus. When the EMT palpates the patient's arm, he pulls it away. His Glasgow Coma Scale (GCS) score is: A)7 B) 8 C) 9 D) 10

B) 8 If a patient opens his or her eyes in response to pain, a score of 2 is assigned for eye opening. If the patient moans or makes unintelligible sounds, a score of 2 is assigned for verbal response. If the patient withdraws from pain, a score of 4 is assigned for motor response. In total, this patient's GCS is 8, which indicates severe neurologic impairment.

Which of the following injury mechanisms involves axial loading? A) skater slips and falls, landing on her outstretched arm. B) A construction worker falls off a roof and lands feet first. C) A woman's knees impact the dash during a frontal collision. D) A man's neck is forced laterally during a side impact collision.

B) A construction worker falls off a roof and lands feet first. Axial loading injuries occur when a sudden, excessive compression force drives the long axis of the body toward the head, or the head toward the feet. Common injuries that involve axial loading are heavy objects falling on a patient's head, diving head first into shallow water, and falls in which the patient lands feet first. All of these mechanisms cause compression of the spine, potentially resulting in serious injury. None of the other injury mechanisms described are consistent with axial loading.

What occurs during recoil of the chest in between compressions? A) Positive pressure accumulates in the thoracic cavity B) A vacuum is created and blood returns to the heart C) Excessive amounts of air are expelled from the lungs D) Forward flow of blood moves to the body's vital organs

B) A vacuum is created and blood returns to the heart During CPR, it is critical to allow the chest to fully recoil before providing the next compression. Recoil of the chest produces a vacuum, which pulls air into the lungs and draws more blood back to the heart. If only partial recoil occurs, positive pressure can accumulate in the chest, which would impair blood return to the heart. Forward blood flow and expulsion of air from the lungs occurs during the downstroke of the compression, not the recoil phase.

Which of the following patients is the BEST candidate for the administration of nitroglycerin? A) woman who has taken three doses of prescribed nitroglycerin without relief of chest pain B) A woman with chest pain, prescribed nitroglycerin, and a blood pressure of 104/76 mm Hg C) A man with chest pain, expired nitroglycerin spray, and a blood pressure of 110/80 mm Hg D) An elderly man with crushing substernal chest pain and a blood pressure of 80/60 mm Hg

B) A woman with chest pain, prescribed nitroglycerin, and a blood pressure of 104/76 mm Hg Nitroglycerin should be administered to patients who have the prescribed, unexpired drug with them and a systolic blood pressure of greater than 100 mm Hg. No more than three nitroglycerin tablets or sprays should be administered to a patient in the prehospital setting. An expired medication should never be administered to any patient, even if the medication is otherwise indicated for his or her condition.

A woman struck the steering wheel with her chest when her car collided with a tree. She is conscious and alert; however, she is tachypneic and diaphoretic and her pulse is rapid and irregular. What should you do? A) Apply the AED and administer oxygen. B) Administer oxygen and protect her spine. C) Administer oxygen and position her on her side. D) Ventilate with a bag-mask device and apply a cervical collar.

B) Administer oxygen and protect her spine. Based on the mechanism of injury and the patient's clinical presentation (especially her rapid, irregular pulse), a myocardial contusion should be suspected. Large myocardial contusions can reduce the pumping function of the heart, resulting in shock. One cannot diagnose a myocardial contusion in the prehospital setting, and there is no specific treatment for this type of injury; therefore, prompt transport is essential. Treatment for the patient in this scenario includes supplemental oxygen (nasal cannula or nonrebreathing mask, depending on her oxygen saturation), spinal motion restriction (ie, cervical collar, backboard, etc.), and keeping her warm. A lateral recumbent position would likely not be comfortable for her. She does not require ventilation assistance at this point. The AED is not indicated because she is not in cardiac arrest.

A scuba diver complains of shortness of breath and severe muscle and joint pain immediately after ascending from a dive. He is cyanotic and is coughing up bloody froth. What should you suspect? A) The bends B) Air embolism C) Decompression sickness D) Nitrogen bubbles in the blood

B) Air embolism The most dangerous, and most common, emergency in scuba diving is an air embolism, a condition involving bubbles of air in the blood vessels. An air embolism may occur on a dive as shallow as 6 feet. The problem starts when the diver holds his or her breath during a rapid ascent. The air pressure in the lungs remains at a high level while the external pressure on the chest decreases. As a result, the air inside the lungs expands rapidly, causing alveolar rupture. The air released from the ruptured alveoli can cause air bubbles in the bloodstream (air emboli), pneumothorax, or pneumomediastinum. Signs and symptoms of an air embolism include skin mottling; pink or bloody froth at the nose and mouth; severe pain in the muscles, joints, or abdomen; dyspnea and/or chest pain; cough; cyanosis; and dizziness, nausea, and vomiting. Decompression sickness, also called the bends, is also an ascent problem. It occurs when bubbles of gas, especially nitrogen, obstruct the blood vessels. The most striking symptom of decompression sickness is abdominal and/or joint pain so severe that the patient literally doubles over or "bends!' It can be difficult to distinguish between air embolism and decompression sickness. As a general rule, air embolism occurs immediately on return to the surface, whereas the symptoms of decompression sickness may not occur for several hours. Treatment at the hospital is the same for air embolism and decompression sickness and involves placing the patient in a hyperbaric chamber.

Which of the following is considered a severe burn in a pediatric patient? A) Partial thickness burns to 15% of the body surface area B) Any full-thickness burn, regardless of body surface coverage C) Any partial-thickness burn that involves the lower extremities D) Superficial burns that cover 5% to 10% of the body surface area

B) Any full-thickness burn, regardless of body surface coverage Minor burns in pediatric patients include small coverage area superficial burns and partial-thickness burns that involve less than 10% of the body surface area. Partial-thickness burns involving 10% to 20% of the body surface area are classified as moderate burns. Severe burns in pediatric patients include any fullthickness burn, regardless of location or body surface area coverage; partial-thickness burns involving more than 20% of the body surface area; and any burn involving the hands, feet, face, airway, or genitalia.

A woman fell through a glass window and has a large laceration to her inner arm that is bleeding profusely. The EMT applies a trauma dressing and direct pressure to the wound, but the dressing immediately becomes soaked with blood. What should the EMT do next? A) Pack the wound cavity with a hemostatic-impregnated gauze B) Apply a proximal tourniquet and engage it until the bleeding stops C) Apply a second trauma dressing and elevate her arm above her heart D) Pack the wound with sterile gauze and then apply a pressure dressing

B) Apply a proximal tourniquet and engage it until the bleeding stops If direct pressure does not immediately stop massive hemorrhage, the EMT should apply a proximal tourniquet and engage it until the bleeding stops and the pulse distal to the injury is not palpable. Placing additional dressing over a massively hemorrhaging wound will NOT stop the bleeding, it will only hide it! The original paradigm of direct pressure, elevation, pressure point, pressure bandage is gone; these are often ineffective strategies in wounds that are massively hemorrhaging. The use of hemostatic-impregnated gauze is indicated for wounds that are too proximal for a tourniquet (ie, junctional injuries). For wounds that are amenable to a tourniquet, it is preferable to apply a tourniquet versus packing the wound.

A 3-year-old child presents with respiratory distress. She is conscious, crying, and clinging to her mother. She has mild intercostal retractions and an oxygen saturation of 91%. What should you do? A) Gently restrain the child and assist her ventilations. B) Ask the mother to hold an oxygen mask near the child's face. C) Apply a nasal cannula with the flow rate set to 6 L/min. D) Apply a nonrebreathing mask with the flow rate set to 12 L/min.

B) Ask the mother to hold an oxygen mask near the child's face. Do not assume that a child will simply allow you to administer oxygen to him or her as you would to an adult. The child in this scenario, who is in respiratory distress and is mildly hypoxemic (Sp02 of 91%), should receive supplemental oxygen; however, it should be given in a nonthreatening manner. Agitating a sick or injured child causes an increase in oxygen consumption and demand, which may cause the child's condition to deteriorate. In this scenario, ask the child's mother to hold an oxygen mask near the child's face (blow-by oxygen). Small children are not as likely to tolerate a nasal cannula or nonrebreathing mask as an adult would. Closely monitor the child's condition and be prepared to assist her ventilations with a bag-mask device if she deteriorates. Allow the child to assume a position of comfort and transport.

Which of the following sets of vital signs is MOST indicative of increased intracranial pressure in a patient with a head injury? A) BP, 84/42 mm Hg; pulse, 60 beats/min; respirations, 32 breaths/min B) BP, 176/98 mm Hg; pulse, 50 beats/min; respirations, 10 breaths/min C) BP, 92/60 mm Hg; pulse, 120 beats/min; respirations, 24 breaths/min D) BP, 160/72 mm Hg; pulse, 100 beats/min; respirations, 12 breaths/min

B) BP, 176/98 mm Hg; pulse, 50 beats/min; respirations, 10 breaths/min The body responds to a significant traumatic brain injury by shunting more oxygenated blood to the injured brain; it does this by increasing systemic blood pressure. In response to an increase in blood pressure, the pulse rate decreases. Pressure on the brainstem often causes an irregular breathing pattern that is either slow or fast. Therefore, patients with increased intracranial pressure present with hypertension, bradycardia, and irregular respirations that are fast or slow (Cushing triad). Vital signs representative of shock (eg, hypotension, tachycardia) are not common in patients with an isolated head injury and increased intracranial pressure. If the patient with a seemingly isolated head injury is hypo tensive and tachycardic, look for other injuries; internal or external bleeding is likely occurring elsewhere.

Which of the following clinical signs indicates left heart fai lure? A) An irregular pulse B) Dyspnea on exertion C) Severe peripheral edema D) Jugular venous distention

B) Dyspnea on exertion Signs of left heart failure are reflective of blood that backs up in the lungs and causes pulmonary edema. These include exertional dyspnea, paroxysmal nocturnal dyspnea, dyspnea while lying down (orthopnea), and coughing up blood (hemoptysis). Signs of right heart failure are reflective of blood that backs up in the systemic circulation. These include peripheral edema, jugular venous distention, and abdominal distention. An irregular pulse indicates the presence of a cardiac dysrhythmia, which could be present with left or right heart failure.

A 19-year-old man is experiencing hallucinations and paranoia after abusing an unknown substance. His heart rate is 170 beats/min, he complains of a headache, and he is experiencing muscle twitching. Which of the fo llowing substances would MOST likely explain his presentation? A) Heroin B) Bath salts C) Ketamine D)Valium

B) Bath salts Of the drugs listed, the patient's clinical presentation is the most consistent with use of synthetic cathinones (bath salts). Bath salts, not to be confused with products such as Epsom salt, are a class of drugs similar to MDMA (ecstasy). Cloud Nine and Ivory Wave are common streets names for bath salts. Bath salts produce euphoria, increased mental clarity, and sexual arousal. Most users snort the drug. Adverse effects include teeth grinding, loss of appetite, muscle twitching, lip -smacking, confusion, paranoia, headache, tachycardia, and hallucinations. Heroin (an opioid) and Valium (a benzodiazepine) are both central nervous system depressant drugs; therefore, they would not explain the patient's presentation. Ketamine (Ketalar) is a sedative-hypnotic drug; it would not explain the patient's signs and symptoms.

A 60-year-old man presents with an acute onset of confusion and slurred speech. Which of the following assessments would be the MOST likely to reveal the possible cause of his altered mental status? A) Oxygen saturation B) Blood glucose level C) Palpation of the skull D) Systolic blood pressure

B) Blood glucose level Altered mental status can be caused by a multitude of conditions, and they are too numerous to mention here. In this patient, the additional sign of slurred speech may help narrow your differential diagnosis; stroke and hypoglycemia both often present with slurred speech. Therefore, the patient's blood glucose level would most likely provide information as to the possible cause of his presentation; it should be assessed as soon as possible. A thorough assessment should be performed on this patient, including his oxygen saturation and vital signs. If the patient fell or struck his head, you should palpate his skull as part of your assessment.

Which of the following clinical findings is MOST indicative of a skull fracture? A) Non-reactive pupils B) Blood in the ear canal C) Decorticate posturing D) Increased blood pressure

B) Blood in the ear canal All of the clinical signs listed in this question indicate a head injury. However, not all head injuries are accompanied by skull fractures. Blood coming from the ear canal following a head injury, however, is the most suggestive of a skull fracture. Specifically, it suggests a basilar skull fracture. In many cases, this blood contains cerebrospinal fluid (CSF). Bloody CSF draining from the nose indicates a fractured cribriform plate; when this bone is fractured, CSF leaks into the sinuses and manifests with rhinorrhea (nasal discharge).

Which of the following clinical presentations is the MOST consistent with an opioid overdose? A) Irregular pulse; rapid breathing; and diaphoresis B) Bradycardia; shallow breathing; and hypotension C) Bradycardia; hypertension; and pupillary dilation D) Tachycardia; hot, flushed skin; and severe agitation

B) Bradycardia; shallow breathing; and hypotension Opioids are in a class of drugs called narcotics. While they can be used as an effective means of analgesia, they can cause significant central nervous system (CNS) depression if taken in excess. Opioid toxicity depresses the respiratory drive, resulting in slow, shallow breathing. Other CNS depressant effects of toxicity include a decreased level of consciousness (including coma), bradycardia, hypotension, and constricted pupils (miosis). By contrast, CNS stimulant drugs, such as methamphetamine and cocaine, would cause hypertension, tachycardia, hyperthermia, and agitation or combativeness.

Which of the following assessment parameters is a more reliable indicator of perfusion in infants than adults? A) Pulse quality B) Capillary refill C) Blood pressure D) Level of orientation

B) Capillary refill Capillary refill time (CRT) is a reliable indicator of perfusion in children younger than 6 years of age. When the capillary bed (eg, fingernail, forehead) is blanched, blood should return to the area in less than 2 seconds. Because peripheral perfusion decreases with age, CRT is a less reliable indicator of perfusion in older children and adults. Note that cold temperatures can affect CRT. Pulse quality is reliable in patients of any age; weak or absent peripheral pulses indicate poor perfusion in anyone. Blood pressure is the least reliable indicator of perfusion in patients of any age; it usually does not fall until the body's compensatory mechanisms have failed. Assessing an infant's level of orientation is not possible; infants do not know who they are, where they are, what happened, and what day it is. When assessing an infant's mental status, note his or her level of alertness and interactivity (eg, tracking with his or her eyes, crying versus quiet).

While performing CPR on a patient who is wearing an external defibrillator vest, the device alarms and a voice states that a shock is about to be delivered. What should you do? A) Remove the battery from the device and apply your AED B) Cease patient contact until the shock has been delivered C) Continue CPR because the vest delivers a low energy shock D) Allow it to shock, but immediately check for a carotid pulse

B) Cease patient contact until the shock has been delivered An alternative to the implantable cardiac defibrillator is the external defibrillator vest. This device is a vest with built-in monitoring electrodes and defibrillation pads; it is worn by the patient under his or her clothing. The monitor provides alerts and voice prompts when it recognizes a lethal dysrhythmia and before a shock is delivered. Unlike the implantable defibrillator, the defibrillator vest uses high-energy shocks similar to an AED; therefore, you should avoid contact with the patient until the shock has been delivered. Immediately after the shock has been delivered, you should immediately resume chest compressions; do not stop to check for a pulse. Removing the battery or othe,wise disabling the device when it is about to deliver a shock is clearly inappropriate. The vest should be left in place while CPR is being performed unless it interferes with chest compressions. If it is necessary to remove the vest, simply remove the battery from the monitor, remove the vest, and apply your AED.

EMTs are assessing a man who is under the influence of an unknown substance. The patient is agitated, his BP is 180/100 mm Hg, his pulse rate is 150 beats/min, and his skin is hot to the touch. Which of the following drugs would MOST likely cause these signs and symptoms? A) Heroin B) Cocaine C) Alcohol D)Valium

B) Cocaine

Which of the following is a later sign of hepatitis? A) Fatigue B) Jaundice C) Loss of appetite D) Fever and vomiting

B) Jaundice Early signs and symptoms of viral hepatitis include loss of appetite (anorexia), vomiting, fever, fatigue, and muscle and joint pain. Jaundice (yellow sclera and skin) and right upper quadrant abdominal pain are not common early manifestations of hepatitis; they usually develop within 1 to 2 weeks into the disease process.

Which of the following drugs can cause or worsen excited delirium? A) Heroin B) Cocaine C) Ketamine D) Marijuana

B) Cocaine Excited delirium, also referred to as agitated delirium, is a condition caused by a severe increase in the metabolic rate, combined with impairment of cognitive function. The symptoms of excited delirium include hyperactive irrational behavior with possible vivid hallucinations, which can create the potential for violence. Common physical manifestations include hypertension, tachycardia, diaphoresis, hyperthermia, and dilated pupils. Drugs that increase metabolism, such as cocaine or methamphetamine (sympathomimetics), can cause or worsen excited delirium. The condition can lead to sudden death, usually from metabolic acidosis. Heroin, an opioid, depresses metabolism; therefore, it would not cause or worsen excited delirium. Ketamine, a sedativehypnotic, is commonly used to treat patients with excited delirium. Marijuana can cause changes in reality perception; however, the more potent sympathomimetics are far more common in inducing or worsening excited delirium.

Which of the following statements regarding two-rescuer child CPR is correct? A) The chest should not be allowed to fully recoil in between compressions, as this may impair venous return. B) Compress the chest with one or two hands to a depth that is equal to one-third the diameter of the chest. C) The chest should be compressed with one hand and a compression to ventilation ratio of 30:2 should be delivered. D) A compression to ventilation ratio of 15:2 should be delivered without pauses in compressions to deliver ventilations.

B) Compress the chest with one or two hands to a depth that is equal to one-third the diameter of the chest. When performing two-rescuer CPR on a child (1 year of age to the onset of puberty [12 to 14 years of age]), the chest should be compressed with one or two hands (depending on the size of the child), and a compression to ventilation ratio of 15:2 should be used. It is important to compress the chest to an adequate depth: one-third the anterior-posterior diameter of the chest (about 2 inches in the child). The chest should be allowed to fully recoil in between compressions to maximize venous return to the heart. If an advanced airway device (ie, ET tube, supraglottic airway) is not in place, two rescuers should deliver "cycles" of CPR; the compressor should pause briefly so the ventilator can deliver two breaths. A compression to ventilation ratio of 30:2 is used for one-rescuer child CPR. After an advanced airway device has been inserted, "cycles" of CPR should not be performed; compressions should be continuous at a rate oflOO to 120/min and ventilations should be delivered at a rate oflO breaths/min (one breath every 6 seconds).

A 72-year-old woman reports dyspnea that woke her from her sleep. Her feet and ankles are swollen, and auscultation of her lungs reveals crackles to both lung bases. She has a history of hypertension, type 2 diabetes, and a heart attack 2 years ago. Her BP is 170/94 mm Hg, her pulse is 110 beats/min and irregular, her respirations are 24 breaths/min and labored, and her oxygen saturation is 85% on room air. What should you suspect? A) Acute asthma attack B) Congestive heart failure C) Acute hypertensive crisis D) Emphysema exacerbation

B) Congestive heart failure The patient's clinical presentation is consistent with congestive heart failure (CHF) with acute pulmonary edema. Dyspnea that awakens a person from sleep is called paroxysmal nocturnal dyspnea (PND) and is classic for CHF. Crackles auscultated over her lungs further support the diagnosis of pulmonary edema, and her low oxygen saturation indicates hypoxemia. Swelling to the feet and ankles suggests an element of right heart failure as well. Emphysema is unlikely because there is no mention of it in her medical history. Furthermore, patients with emphysema typically have dyspnea all the time, not just when they lie down to sleep. Acute asthma is also unlikely; one would expect wheezing (not crackles) in the lungs. Although the patient's BP is elevated, she has no other signs of acute hypertensive crisis, such as a headache, nausea and vomiting, and ringing in the ears.

Which of the following is the MOST detrimental effect of gastric distention in infants and children? A) Rupture of the trachea B) Decreased ventilatory volume C) Acute rupture of the diaphragm D) Less effective chest compressions

B) Decreased ventilatory volume Gastric distention can be lethal if not detected and managed appropriately in any patient, but especially infants and children. As air insufflates the stomach, the diaphragm is pushed into the thoracic cavity, which decreases the amount of air that can fill the lungs. This results in decreased ventilatory volumes during artificial ventilation. Gastric distention also increases the risks of regurgitation and aspiration. You must be able to deliver adequate ventilations to the patient. Remember, respiratory failure is the most common cause of cardiopulmonary arrest in children.

A 44-year-old man reports di zziness and light-headedness. His BP is 88/62 mm Hg; his heart rate is 190 beats/min and weak; and his skin is cool, clammy, and pale. Which of the following would BEST explain why his BP is low? A) Widespread systemic vasoconstriction B) Decreased ventricular filling time C) Increased ventricular stroke volume D) Decreased myocardial oxygen demand

B) Decreased ventricular filling time When the heart is beating exceedingly fast, there is less time in between contractions for the ventricles to fill. As a result, stroke volume decreases and the blood pressure falls; this is the most likely cause of this patient's hypotension. Widespread vasoconstriction would increase the blood pressure, not decrease it. Decreased myocardial oxygen demand simply means that the heart is not working too hard; this does not affect blood pressure.

After completing a rapid assessment of a patient with a penetrating chest injury, the EMT tells her partner that she suspects a pericardia! tamponade. Which of the following did the EMT MOST likely encounter during her assessment? A) Slow, bounding pulse B) Distended jugular veins C) Unequal breath sounds D) Widening pulse pressure

B) Distended jugular veins Pericardia! tamponade occurs when an injury to the heart causes blood to collect in the pericardia! sac; the accumulation of blood puts pressure on the heart and impairs its ability to fill. If the right side of the heart cannot fill, there is nothing for it to send to the left side of the heart; as a result, the patient's blood pressure falls. Beck triad is a classic presentation of pericardia! tamponade; it includes jugular vein distention (blood backs up into the venous system because of impaired cardiac filling), muffled or distant heart sounds, and a narrowing pulse pressure (the difference between the systolic and diastolic blood pressure). Other signs include tachycardia, weak pulses, and hypotension. The presence of unequal breath sounds should raise suspicion of a pneumothorax. A widening pulse pressure may be seen with increased intracranial pressure following a head injury.

Which of the following signs and symptoms would the EMT expect to encounter in a patient with organophosphate poisoning? A) Flushed skin, blurred vision, and tachycardia B) Excessive salivation, bradycardia, and vomiting C) Dilated pupils, jaundice, dry mouth, and nausea D) Hyperthermia, hypertension, and combativeness

B) Excessive salivation, bradycardia, and vomiting Organophosphates are found in industrial pesticides and insecticides; they are in a class of chemical called cholinergics. The parasympathetic nervous system is also referred to as the cholinergic nervous system, and it is responsible for slowing of the heart rate, as well as all your other resting functions (including the gastrointestinal tract). A patient with organophosphate poisoning would therefore be expected to present with bradycardia, hypersalivation, and other clinical signs of increased parasympathetic (cholinergic) activity. The clinical syndrome caused by organophosphate poisoning can be remembered with the mnemonic SLUDGEM, which stands for Salivation/Sweating; Lacrimation; Urination; Defecation, drooling, and diarrhea; Gastric upset and cramps; Emesis; and Muscle twitching and miosis (pinpoint pupils}. Another mnemonic is DUMBELS, which stands for Defecation, Urination, Miosis, Bradycardia, Emesis, Lacrimation, and Salivation.

Which of the following clinical findings is consistent with decompensated shock? A) Diaphoresis and pallor B) Falling blood pressure C) Restlessness and anxiety D) Tachycardia and tachypnea

B) Falling blood pressure During shock, the compensatory mechanisms of the body attempt to maintain the blood pressure. This is accomplished by increasing the heart rate, shunting blood from the skin to more vital organs, and increasing the respiratory rate to increase the oxygen content of the blood. Once these compensatory mechanisms fail, the blood pressure will fall (hypotension). Hypotension signifies a state of decompensated shock. You must not rely on the patient's blood pressure as an indicator of overall perfusion. Restlessness, anxiety, tachycardia, tachypnea, and cool, clammy skin (diaphoresis) are earlier signs of shock and do not necessarily indicate a decompensated state.

Which of the following patient presentations would MOST likely require the EMT to don a high-efficiency particulate air (HEPA) respirator? A) Fever, weight loss, and jaundice B) Fever, a cough, and night sweats C) Jaundice, vomiting, and weakness D) Nuchal rigidity, fever, and headache

B) Fever, a cough, and night sweats Fever, a cough, and night sweats are signs of tuberculosis (TB). If they are observed in a patient, you should don a high-efficiency particulate air (HEPA) respirator (N95 or higher) because TB is transmitted via the droplet (aerosolization) route. The HEPA respirator will trap the bacterial particles and prevent you from inhaling them. Jaundice is a sign of liver disease or dysfunction, not TB. Nuchal rigidity, especially with fever, is a sign of meningitis; it is not a sign of TB. A HEPA N95 should also be worn when caring for a patient with suspected COVID-19.

Which of the following statements regarding crowning is correct? A) Crowning represents the end of the second stage of labor. B) Gentle pressure should be applied to the baby's head during crowning. C) It is safe to transport the patient during crowning if the hospital is close. D) Crowning always occurs immediately after the amniotic sac has ruptured.

B) Gentle pressure should be applied to the baby's head during crowning. Crowning occurs when the baby's head is visible at the vaginal opening; it is an obvious sign of delivery in progress. When crowning is observed, you should apply gentle pressure to the infant's head to prevent an explosive delivery. Care must be taken to avoid putting pressure on the fontanelles (the soft spots on the infant's head). Crowning represents the end of the first stage of labor and the beginning of the second stage; it does not always occur immediately after the amniotic sac has ruptured. If the infant's head is born and the amniotic sac is still intact, you need to pinch the thin membrane with your fingers, which will usually cause the sac to easily rupture, and then suction the infant's mouth and nose.

A 57-year-old man presents with fl u-like symptoms. He is conscious and alert; his skin is pink, warm, and dry; and he denies chest pain or respiratory distress. During your assessment, you cannot feel radial or carotid pulses, and are unable to obtain a blood pressure. Which of the following would MOST likely explain this? A) He is in the advanced stages of shock B) He has a left ventricular assist device C) He has a dissecting aortic aneurysm D) He recently had a coronary artery bypass

B) He has a left ventricular assist device The patient is clearly stable, as evidenced by his level of consciousness; normal skin, color, condition, and temperature; and absence of chest pain or respiratory distress. These findings rule out shock, especially advanced shock. Patients with acute aortic dissection typically present with severe, tearing chest pain that often radiates to the back; the patient has none of these symptoms. A coronary artery bypass graft (CABG) would not explain the absence of pulses and blood pressure. The only possible option is that he has a left ventricular assist device (LVAD) with a continuous flow pump. Patients with this type of LVAD do not have palpable pulses, despite the fact that their body is adequately perfused. An LVAD with a pulsatile pump, however, would generate a palpable pulse.

Which of the following types of stroke would MOST likely present with a sudden, severe headache? A) Ischemic B) Hemorrhagic C) Thrombotic D) Embolic

B) Hemorrhagic A stroke occurs when blood flow to a portion of the brain is interrupted. There are two types of stroke, both of which are acute events: ischemic and hemorrhagic. Hemorrhagic stroke is caused by a ruptured artery in the brain, resulting in intracerebral bleeding, cerebral ischemia, and increased intracranial pressure. A classic feature of a hemorrhagic stroke is a sudden, severe headache (indicates cerebral artery rupture) that is followed by rapid deterioration of the patient's condition. By contrast, ischemic stroke (accounts for approximately 87% of all strokes) occurs when a clot occludes a cerebral artery. If the clot forms locally, it is referred to as a thrombotic stroke; if the clot travels to the brain from another part of the body, it is referred to as an embolic stroke. Acute ischemic stroke typically presents with a sudden onset of confusion, facial droop, slurred speech, and weakness to one side of the body. A headache is uncommon in patients with acute ischemic stroke.

Which of the fo llowing conditions would be the LEAST likely to be present in a patient who was submerged in water? A) Spinal injury B) Hyperglycemia C) Laryngospasm D) Gastric distention

B) Hyperglycemia Many factors can contribute to or result from a submersion injury (eg, drowning, near-drowning). It is not uncommon for a person to experience a spinal injury after diving head first into shallow water, especially if he or she is under the influence of alcohol. When a swimmer panics, he or she initially swallows large amounts of water, resulting in gastric distention. Gastric distention can cause aspiration if the patient regurgitates water during rescue breathing; protect the airway! During the panic phase, the victim expends a tremendous amount of energy (and glucose) by flailing around in the water, possibly resulting in hypoglycemia. Inhaling even a small amount of fresh or salt water can severely irritate the larynx, which sends the muscles of the larynx and vocal cords into spasm (laryngospasm), resulting in airway blockage and hypoxia.

Which of the following factors would reduce the blood's natural ability to clot, thus worsening internal or external bleeding? A) Tachycardia B) Hypothermia C) Hypotension D) Vasoconstriction

B) Hypothermia Hypothermia can cause an abnormality in blood clotting (coagulopathy), which can impair clotting factors and cause prolonged bleeding. This is why patients with hemorrhagic shock should be kept warm. Tachycardia, hypotension, and vasoconstriction do not impair the blood's ability to clot.

Which of the following is the MOST appropriate response when a patient with chest pain asks you if he or she is having a heart attack? A) Yes, so I recommend going to the hospital. B) I don't know, but we will take good care of you. C) Probably not, but we should transport you to be safe. D) I believe you are, but only a physician can tell for sure.

B) I don't know, but we will take good care of you. Patients experiencing chest pain often have a good idea about what is happening, so do not lie or offer false reassurance. Conversely, do not tell patients they are having a heart attack; EMTs are not trained to interpret the 12-lead ECG, and the 12-lead ECG is currently the only way a heart attack can be diagnosed in the prehospital setting. If asked, "Am I having a heart attack?", an appropriate response would be "I don't know for sure, but in case you are, we are going to take good care of you."

Which of the following statements regarding ventricular fibrillation (V-Fib) is correct? A) Loss of consciousness occurs within minutes after the onset ofV-Fib. B) In V-Fib, the heart is not pumping any blood and the patient is pulseless. C) Patients in V-Fib should be defibrillated after every 60 seconds of CPR. D) Any patient in V-Fib must receive CPR for 2 minutes prior to defibrillation.

B) In V-Fib, the heart is not pumping any blood and the patient is pulseless. Ventricular fibrillation (V-Fib) is a disorganized, ineffective quivering of the heart muscle. No blood is pumped through the body and the patient is pulseless. Loss of consciousness occurs within seconds following the onset of V- Fib. Patients in V-Fib are treated with high-quality CPR and defibrillation every 2 minutes if needed. When treating a patient in cardiac arrest, whether the patient's arrest was witnessed or unwitnessed, begin immediate CPR and apply the AED as soon as it is available.

A trauma patient has a BP of 172/94 mm Hg, a pulse rate of 45 beats/min, and a respiratory rate of 6 breaths/min. Which of the following conditions would MOST likely produce this vital sign pattern? A) Severe internal hemorrhage B) Increased intracranial pressure C) Increased intrathoracic pressure D) Bleeding into the pericardia! sac

B) Increased intracranial pressure The classic vital sign pattern of Cushing's Triad (hypertension, bradycardia, abnormal breathing) is present. This vital sign pattern is seen in patients with increased intracranial pressure (ICP) secondary to a head injury; it can also be observed in patients who have experienced a severe hemorrhagic stroke. With a head injury, the blood pressure increases in an effort to push more blood to the brain; bradycardia occurs as a reflex response to the increased in blood pressure. Excessive ICP can produce a variety of abnormal breathing patterns, from slow and irregular to rapid with no identifiable pattern. Severe internal hemorrhage would be expected to present with signs of shock (ie, hypotension, tachycardia, tachypnea), not hypertension, bradycardia, and bradypnea. Likewise, an increase in intrathoracic pressure can also produce symptoms similar to shock; as intrathoracic pressure increases, venous return to the right side of the heart decreases. This can cause decreased cardiac output and hypotension. Bleeding into the pericardia! sac (ie, pericardia! tamponade) causes hypotension with a narrowing pulse pressure and tachycardia.

Which of the following is the MOST detrimental effect that tachycardia can have on a patient experiencing a cardiac problem? A) Increased blood pressure B) Increased oxygen demand C) Increased stress and anxiety D) Decreased cardiac functioning

B) Increased oxygen demand As the heart beats faster, it consumes and demands more oxygen. Tachycardia can be extremely detrimental to the patient with a compromised heart that is already deprived of oxygen. Keeping the patient calm cannot be overemphasized. The more stressed and anxious the patient gets, the faster his or her heart will beat; this is NOT a favorable effect in a patient experiencing acute cardiac ischemia!

A man was struck in the head with a baseball bat. He is unresponsive, his breathing is slow and irregular, and his pulse is slow and bounding. What should you do? A) Insert a nasopharyngeal airway and give oxygen via nonrebreathing mask. B) Insert an oropharyngeal airway and ventilate with a bag-mask device. C) Administer oxygen via nonrebreathing mask and elevate his legs 6 to 12 inches. D) Administer oxygen via nasal cannula and elevate his torso to a 45-degree angle.

B) Insert an oropharyngeal airway and ventilate with a bag-mask device. Based on the mechanism of injury and the patient's signs and symptoms, you should suspect that he has a significant closed head injury with increasing intracranial pressure. Since he is unresponsive, you should insert an airway adjunct to obtain a patent airway; the oropharyngeal airway would be the best choice since nasopharyngeal airways should generally be avoided in patients with a head injury. Slow, irregular breathing is not adequate and should be treated with bag-mask ventilation; ventilate the patient at a rate of 10 breaths/min. Hypoxia can have disastrous effects on a patient with a severe head injury, so be sure to attach supplemental oxygen to the bag-mask device. Elevating the patient's lower extremities should be avoided; doing so may further increase intracranial pressure. Instead, consider elevating the patient's head and torso to 30 degrees.

Following delivery of a baby, the newborn has a strong cry and a heart rate of 130 beats/min. What should you do? A) Suction the baby's mouth and nose B) Keep the baby warm and transport C) Ventilate with a bag-mask device D) Administer free-flow oxygen by mask

B) Keep the baby warm and transport If the newborn has no signs of distress, has pink skin, and a heart rate greater than 100 beats/min, simply keep the baby warm and transport to the hospital. Re-suctioning the mouth and nose is not necessary if the newborn is breathing adequately. Bag-mask ventilation is indicated if the newborn is apneic or has gasping respirations, or if the heart rate is less than 100 beats/min. If the baby has central cyanosis- cyanosis to the face or trunk- administer free -flow oxygen by face mask or oxygen tubing.

A 34-year-old woman, who is 36 weeks' pregnant, is having a seizure. You should transport her in which position? A) Prone B) Left side C) Supine D) Semi-Fowler

B) Left side Initial care for any patient who is seizing--pregnant or otherwise--involves ensuring a patent airway, ensuring adequate ventilation, and administering highflow oxygen. If the patient is breathing inadequately, ventilation assistance is indicated. Suction any secretions from the patient's mouth. The pregnant patient should be placed on her left side (lateral recumbent position); this will prevent supine hypotensive syndrome, a condition in which the pregnant uterus compresses the inferior vena cava and reduces cardiac output. A lateral recumbent position will also facilitate the draining of oral secretions, thus minimizing the risk of aspiration.

A 24-year-old woman presents with a rash to her left leg and swollen, painful knee joints. She tells you that she and her friends returned from a hiking trip in the mountains a week ago. She is conscious and alert with a blood pressure of 112/62 mm Hg, a pulse of 84 beats/min, and respirations of 14 breaths/min. What should you suspect? A) Tetanus B) Lyme disease C) An allergic reaction D) Rocky Mountain spotted fever

B) Lyme disease The patient's symptoms and her history of a recent hiking trip are consistent with Lyme disease, which is the result of a tick bite. Ticks can carry two infectious diseases: Lyme disease and Rocky Mountain spotted fever. Both are spread through the tick's saliva, which is injected into the skin when the tick attaches itself. The first symptom of Lyme disease, a rash that may spread to several parts of the body, begins about 3 days after the bite of an infected tick. The rash may eventually resemble a target or bull's-eye pattern in one-third of patients. After a few more days or weeks, painful swelling of the joints, particularly the knees, occurs. If recognized and treated promptly with antibiotics, many patients recover completely. Rocky Mountain spotted fever occurs within 7 to 10 days after being bitten by an infected tick. Its symptoms include nausea, vomiting, headache, weakness, paralysis, and possibly cardiopulmonary failure. Tetanus is not causing the patient's symptoms; patient's with tetanus experience severe muscle spasms (tetany). An allergic reaction would present within a few minutes to hours following exposure to an offending agent; this patient's symptoms are not consistent with an allergic reaction.

A 3-year-old child experienced a seizure that lasted about 10 minutes. He has a fever of 103.5°F, his skin is hot to the touch, and he has a rash on his trunk. What should you suspect? A) Epilepsy B) Meningitis C) Intracranial hemorrhage D) An allergic reaction

B) Meningitis Febrile seizures and fever with seizures are not one in the same. Febrile seizures are caused by fever and fever alone. Conversely, fever with seizures could be something else. The rash on the child's trunk indicates possible sepsis, and should make the EMT suspect meningitis; a rash is not a common finding with simple febrile seizures. Rash and fever are not common findings with intracranial hemorrhage, and fever is not commonly associated with an allergic reaction.

A 42-year-old man presents with fever, a severe headache, and a stiff neck. He is conscious, but confused. His wife tells you that he does not have any medical problems and does not take any medications. What should you suspect? A) Influenza B) Meningitis C) Tuberculosis D) Acute stroke

B) Meningitis Meningitis is an inflammation of the protective coverings of the brain and spinal cord (meninges). Common signs and symptoms of meningitis include fever, headache, neck stiffness (nuchal rigidity), and vomiting. An altered mental status is common in severe cases. Meningococcal meningitis, caused by a bacterium, is the most contagious and potentially fatal type of meningitis. The patient's signs and symptoms are not consistent with acute stroke, tuberculosis (TB}, or influenza (the flu). Although fever is common with both TB and the flu, neither causes neck stiffness. Acute stroke may be associated with a headache, especially a hemorrhagic stroke; however, stroke patients typically do not have a fever.

Law enforcement requests your assistance to evaluate a young woman whom they believe has overdosed. The patient is conscious, is displaying bizarre behavior, and is very restless. Her BP is 170/90 mm Hg and her heart rate is 130 beats/min. You see needle tracks on both of her arms. Which of the following would MOST likely explain her presentation? A) An opioid B) Methamphetamine C) A sedative-hypnotic D) A benzodiazepine

B) Methamphetamine The needle tracks on the patient's arms indicate opioid abuse, most likely heroin; however, her signs and symptoms are not consistent with those caused by an opioid or any other central nervous system (CNS) depressant (eg, benzodiazepine [a sedative-hypnotic]). Signs of CNS depression include a decreased level of consciousness, hypoventilation, bradycardia, and hypotension. Her signs and symptoms are consistent with a sympathomimetic (upper) overdose. Sympathomimetic drugs mimic the effects of the sympathetic (fight or flight) nervous system; therefore, restlessness or hyperactivity, hypertension, tachycardia, and pupillary dilation are common findings in patients who overdose. Amphetamine and methamphetamine (eg, ice, speed, crack) are examples of sympathomimetics.

Your assessment of a mother in labor reveals that a fetal limb is protruding from the vagina. What should you do? A) Position the mother in a semi-Fowler position and administer oxygen. B) Position the mother with her hips elevated and administering high-flow oxygen. C) Apply gentle traction to the protruding limb to remove pressure of the fetus from the umbilical cord. D) Give the mother 100% oxygen, and attempt to manipulate the protruding limb so that delivery can occur.

B) Position the mother with her hips elevated and administering high-flow oxygen. Limb presentations represent a dire emergency for the newborn and do not spontaneously deliver in the field. You should position the mother in a manner so that her hips are elevated in an attempt to slide the infant slightly back into the birth canal and remove pressure from the umbilical cord. Administer high-flow oxygen to the mother, cover the protruding limb with a sterile sheet (or any clean sheet, if a sterile sheet is not available), and transport immediately. Do NOT pull on the protruding limb, as this may cause injury to the newborn.

Your assessment of a 2-year-old boy reveals that he has is not interacting normally and has poor muscle tone. His respirations are 30 breaths/min and unlabored and his skin is pink. These findings suggest a problem with which of the following body systems? A) Cardiac B) Neurologic C) Respiratory D) Circulatory

B) Neurologic The Pediatric Assessment Triangle (PAT) is a structured assessment tool that allows you to rapidly form a general impression of the child's condition without touching him or her. This first glance assessment, which can be performed in less than 30 seconds, will help you identify the general category (neurologic, respiratory, or circulatory) of the patient's physical problem and establish urgency for treatment and/or transport. The PAT is comprised of three components: appearance, work of breathing, and skin circulation. Abnormal mentation and poor muscle tone suggest abnormal neurologic function, such as what caused by hypoglycemia, stroke, or other neurologic problems. A respiratory rate of 30 breaths/min is normal for a 2-year-old child, and considering that his breathing is not labored, a primary respiratory problem is unlikely. Pallor or mottled skin suggests a circulatory problem; however, the child's skin is pink.

Which of the following signs or symptoms would you MOST likely encounter in a patient with an inflamed gallbladder? A) Vomiting of dark red blood B) Pain in the right shoulder C) Bruising around the umbilicus D) Pain to the right lower quadrant

B) Pain in the right shoulder Inflammation of the gallbladder is called cholecystitis. Classic symptoms include right upper quadrant pain, usually shortly after a meal, and referred pain to the right shoulder. Nausea and vomiting may also occur. Hematemesis (vomiting blood) is a sign of gastrointestinal bleeding, not cholecystitis. Bruising around the umbilicus (Cullen sign) indicates blood in the peritoneal cavity. Pain in the lower right abdominal quadrant is common in patients with appendicitis, not cholecystitis.

Which of the following is the MOST reliable indicator of a fractured spinal vertebra? A) Lack of pain at the site of the injury B) Palpable pain at the site of the injury C) Decreased movement on one side of the body D) Decreased grip strength in the upper extremities

B) Palpable pain at the site of the injury Of the options listed, the presence of palpable pain (specifically, point tenderness directly over the injury site) is the most reliable indicator of an underlying vertebral fracture. In fact, point tenderness, second only to gross deformity, is the most reliable indicator of an underlying fracture to any bone.

A 50-year-old man reports severe pain to both upper abdominal quadrants. He states that the pain moves to his back and that he is nauseated and has been running a fever. His wife advises that he does not take care of himself and that he drinks a lot of alcohol. What should you suspect? A) Esophagitis B) Pancreatitis C) Cholecystitis D) Appendicitis

B) Pancreatitis Pancreatitis, inflammation of the pancreas, can be caused by an obstructing gallstone, alcohol abuse, and other diseases. Severe pain may present in the upper left and right abdominal quadrants and often radiates to the back. Other symptoms include nausea and vomiting, abdominal distention, and tenderness. Complications like sepsis or hemorrhage can occur, in which case your assessment may also reveal fever and/or tachycardia. Esophagi tis occurs when the esophageal lining becomes inflamed by infection or from the acids in the stomach. Symptoms of esophagi tis include a burning sensation in the chest (heartburn), pain with swallowing, and the feeling of an object stuck in the throat. The patient's symptoms are not consistent with esophagi tis. Cholecystitis, inflammation of the gallbladder, typically presents with right upper quadrant pain with referred pain to the right shoulder; the symptoms often appear shortly after a meal. The patient's symptoms are not consistent with cholecystitis. Patients with appendicitis can also present with nausea, vomiting, and fever; however, the pain is generally located around the umbilical area and radiates to the lower right abdominal quadrant. The patient's symptoms are not consistent with appendicitis.

A 9-month-old infant presents with a cough, runny nose, and respiratory distress. A whoop sound is heard during inhalation. What should you suspect? A) Croup B) Pertussis C) Bronchiolitis D) Pneumonia

B) Pertussis Pertussis, also known as whooping cough, is a communicable disease caused by a bacterium that is spread via the droplet route. The typical signs and symptoms are similar to a common cold, and include coughing, sneezing, and a runny nose. As the disease progresses, the coughing becomes more severe and is characterized by a distinctive whoop sound heard during inhalation. Infants with pertussis may develop pneumonia or respiratory failure. Croup is unlikely; it is caused by upper airway inflammation and is characterized by stridor and a seal-bark cough. Bronchiolitis is also unlikely; it is a viral infection that causes lower airway constriction and inflammation. Wheezing is commonly observed in children with bronchiolitis. Although pneumonia cannot be completely ruled out, the whoop sound heard during inhalation makes pertussis the most likely cause of this child's symptoms.

You are transporting a woman who is 35 weeks' pregnant and reports a headache. She suddenly becomes pale, light-headed, and tachycardic. What should you do? A) Administer high-flow oxygen. B) Position her on her left side. C) Reassess her blood pressure. D) Elevate her lower extremities.

B) Position her on her left side. Supine hypotensive syndrome can occur beyond the second trimester. When the mother lies supine, the pregnant uterus can compress the inferior vena cava (IVC), which reduces venous return to the right side of the heart. As a result, cardiac output decreases and the patient develops signs of shock (ie, pallor, tachycardia, hypotension). You should immediately position the mother on her left side to relieve pressure off of the IVC. After this intervention, reassess her to determine if there is any improvement. If she has respiratory distress and/or her oxygen saturation is less than 94% on room air, administer supplemental oxygen. Elevating her lower extremities will not resolve the problem and may only make it more difficult for her to breathe.

A 30-year-old woman presents with vaginal bleeding and light-headedness. She states that she has soaked four sanitary pads. Her BP is 90/50 mm Hg, her pulse rate is 120 beats/min, and her respiratory rate is 24 breaths/min. What should you do? A) Instruct her to insert a tampon to help control the bleeding. B) Position her supine and prepare for immediate transport. C) Position her on her side and reassess her blood pressure. D) Place a trauma dressing over her vagina and apply pressure.

B) Position her supine and prepare for immediate transport. The patient is in shock and requires immediate transport. Position her supine, keep her warm, and administer supplemental oxygen. Her bleeding is coming from a source that you cannot control. Direct pressure to the vaginal area will not accomplish bleeding control, nor will a tampon

Which of the following would MOST likely occur in conjunction with a breech presentation? A) Vertex presentation B) Prolapsed umbilical cord C) Maternal hypertension D) Premature rupture of the amniotic sac

B) Prolapsed umbilical cord In most deliveries, the baby's head is the presenting part (vertex presentation). In a breech presentation, the baby's head is not the presenting part; the buttocks usually are. Breech presentations are associated with a higher incidence of prolapsed umbilical cord, a condition in which a part of the umbilical cord precedes the presenting part of the baby. Furthermore, breech presentations increase the risk of delivery- related trauma to the baby. Maternal hypertension and premature rupture of the amniotic sac (bag of waters) are not common concurrent findings in women with a breech presentation.

While performing a visual inspection of a 30-year-old woman in labor, you can see the umbilical cord at the vaginal opening. What should you do? A) Massage the uterus to facilitate delivery of the fetus. B) Relieve pressure from the cord with your gloved fingers. C) Place the mother on her left side and provide rapid transport. D) Elevate the mother's lower extremities and provide rapid transport.

B) Relieve pressure from the cord with your gloved fingers. The goal in treating a prolapsed umbilical cord is to maintain a pulsatile cord. Place your gloved fingers into the vagina and lift the presenting part of the baby off of the umbilical cord. Positioning the mother with her hips elevated may allow the baby to slide off of the umbilical cord. Continued pressure on the umbilical cord will cut off the baby's oxygen supply. In addition, you should keep the cord moist by covering it in saline-soaked dressings. Give the mother high-flow oxygen and provide rapid transport to the hospital. Uterine massage is only indicated AITER delivery to control postpartum bleeding.

A 2-year-old boy experienced a seizure that lasted 3 minutes. His mother states that he began running a fever earlier in the day. He is alert, crying, and is clinging to his mother. His skin is hot and flushed and his heart rate is 140 beats/min. What should you do? A) Place him in a cold water bath to reduce his fever. B) Remove any unnecessary clothing and transport. C) Administer chewable aspirin if his mother has any. D) Allow his mother to transport him to his pediatrician.

B) Remove any unnecessary clothing and transport. The child's presentation is consistent with a febrile seizure. Most febrile seizures are short-lived, so in most cases, the seizure will have stopped prior to EMS arrival. The primary focus of treatment is to make the child comfortable, remove any heat-retaining clothing, and transport to the hospital. Administer oxygen if there is evidence of hypoxemia. Placing the child in cool (or cold) water should be avoided; this would induce shivering (which produces heat) and could cause an acute temperature spike, which could cause another seizure. Aspirin should not be administered to children because it increases the risk of Reye syndrome, a rare but serious condition that causes swelling in the liver and brain. The child should be transported via EMS in case he has another seizure; it is not advisable to allow a parent to transport.

Which of the following signs would you expect to see in the early stages of shock? A) Hypotension B) Restlessness C) Thready pulses D) Unconsciousness

B) Restlessness In the early stages of shock, decreased perfusion to the brain causes the patient to become restless and anxious. As shock progresses, the pulse becomes thready (weak), signifying a falling blood pressure (hypotension), and the patient eventually loses consciousness. It is critical to recognize the early signs of shock and initiate immediate care and rapid transport. You should not rely on the blood pressure as an indicator of perfusion in any patient; by the time hypotension manifests, the patient's compensatory mechanisms have failed and he or she is in decompensated shock.

A 60-year-old man is in cardiac arrest. You begin CPR while your partner applies the AED. What should you do if you receive a no shock message? A) Assess for a carotid pulse for up to 10 seconds. B) Resume CPR, starting with chest compressions. C) Ensure that the AED electrodes are properly applied. D) Reanalyze his cardiac rhythm after 30 seconds of CPR.

B) Resume CPR, starting with chest compressions. If the AED gives a no shock advised message, you should immediately resume CPR, starting with chest compressions. After 2 minutes of CPR, reanalyze the patient's cardiac rhythm and follow the AED voice prompts. You should not assess for a pulse if the AED gives a no shock message; this will simply cause an unnecessary delay in pe1iorming chest compressions. Rarely, if ever, does CPR alone restore a normal cardiac rhythm and pulse. If the AED electrodes are improperly applied, it will not analyze the patient's cardiac rhythm; instead, you will receive a "check patient" or "check electrodes" message. Continue CPR, rhythm analysis every 2 minutes, and defibrillation (if indicated) until ALS personnel arrive or the patient starts to move.

You are assessing a 70-year-old man who reports pain in both of his legs. He is conscious and alert, has a blood pressure of 160/90 mm Hg, a pulse rate of 110 beats/min, and respirations of 14 breaths/min and unlabored. Further assessment reveals edema to both of his feet and legs and jugular venous distention. What should you suspect? A) Left heart fa ilure B) Right heart failure C) Pulmonary edema D) Chronic hypertension

B) Right heart failure If the right side of the heart is damaged, fluid collects in the body (edema), often showing in the feet and legs. The collection of fluid in the part of the body that is closest to the ground is called dependent edema. The swelling causes relatively few symptoms other than discomfort. Another feature of right heart failure is jugular venous distention, which is an indication of blood backing up into the systemic circulation. Left heart failure typically presents with shortness of breath due to fluid in the lungs (pulmonary edema), which indicates blood backing up from the left side of the heart into the lungs. In severe pulmonary edema, the patient may cough up pink, frothy sputum. Right heart failure and/or left heart failw·e are also referred to as congestive heart failure (CHF). Chronic hypertension cannot be established on the basis of a single blood pressure reading.

After attaching the AED and pushing the analyze button on an adult patient in cardiac arrest, the AED states that a shock is advised. Which ca rdiac rhythm is the patient MOST likely in? A) Asystole B) Ventricular fibrillation C) Ventricular tachycardia D) Pulseless electrical activity

B) Ventricular fibrillation Ventricular fibrillation (V-Fib) is the most common initial cardiac dysrhythmia in adult cardiac arrest patients. V- Fib is a chaoti quivering of the heart muscle that does not produce a pulse and is due to a massive, uncontrolled electrical discharge of the cardiac cells. The most effective treatment for V-Fib is defibrillation. Some patients are in ventricular tachycardia (V-Tach) without a pulse, which is also treated with defibrillation. Although asystole and pulseless electrical activity (PEA) do not produce a pulse, they are not treated with defibrillation.

A woman with type 1 diabetes presents with deep, rapid breathing; tachycardia, dehydration; and an altered mental status. Which of the following would MOST likely explain her clinical presentation? A) Her blood sugar level is less than 60 mg/dL. B) She has not taken her insulin in several days. C) She took her insulin but has not eaten today. D) She has an infection that lead to dehydration.

B) She has not taken her insulin in several days. The patient's clinical presentation is consistent with hyperglycemic ketoacidosis. Of the options listed, the only logical explanation is that she has not taken her insulin in several days. Without insulin, glucose cannot enter the cell to make energy, so it pools in the blood to excessive levels (hyperglycemia). Excess blood glucose levels cause the body to eliminate large amounts of water via the kidneys, resulting in dehydration. In the absence of glucose, the cells will metabolize fat, which produces ketoacids. The respiratory system will attempt to eliminate these ketoacids from the body by increasing the rate and depth of breathing (Kussmaul respirations). Had she taken her insulin but did not eat, her blood glucose level would be lower than normal. Although an underlying infection cannot be ruled out, the primary cause of her problem is failure to take her prescribed insulin.

A woman has a closed fracture to her midshaft tibia. You splinted the injury and are monitoring her during transport. During reassessment, she complains of rapidly increasing pain and you note that her leg is pale and cool. What should you suspect? A) The splint was applied too loosely B) She is developing compartment syndrome C) The fracture is accompanied by a dislocation D) She has another injury proximal to the tibia

B) She is developing compartment syndrome Compartment syndrome occurs when expanding, bleeding muscle increases pressure within the osteofascial compartment, the space in between the muscle and fascia. Because the fascia expands very little, increased compartment pressure can impair distal circulation (essentially, compartment syndrome is like an internal tourniquet). Compartment syndrome commonly develops in patients with injuries distal to the elbow and knee. This is because there are two bones in these areas and numerous muscle compartments. A hallmark sign of compartment syndrome is disproportionate pain; the patient's pain is worse that you would expect for the injury. As the condition develops, the extremity may become pale and cool to the touch, and the patient may complain of numbness and tingling (parasthesia) The skin over the injury may also become very tight. Eventually, the patient may lose all distal circulation, and potentially, their extremity. Prehospital treatment involves recognizing the developing signs and promptly transporting the patient to an appropriate facility. Treatment at the hospital involves a procedure called a fasciotomy, which is usually performed by a surgeon. A splint that is applied too tightly could also cause signs and symptoms similar to compartment syndrome, namely pallor; cool skin distal to the splint; and weak or absent distal pulses. Simply loosening the splint until distal circulation improves is often all that is needed. Fracture/dislocations occur at the joints, not the midshaft of a bone. There is no evidence to suggest that she has another injury proximal to the tibia; furthermore, the presence of one would not explain her symptoms.

Following a motor vehicle crash, a 3-year-old girl has an obvious deformity to the mid-shaft humerus. What should you suspect? A) This is not an uncommon injury in a small child B) She was exposed to significant traumatic force C) She has a greenstick fracture because her bones are soft D) She has an underlying condition that causes brittle bones

B) She was exposed to significant traumatic force The deformity to the child's humerus indicates a complete fracture. Fractures of the humerus or femur do not normally occur without major trauma, such as falls from a high place or a motor vehicle crash. Your index of suspicion should be high if an infant or young child sustains a complete fracture of any bone. Infants and small children are more likely to experience greenstick (incomplete) fractures , as opposed to complete fractures, due to their soft and pliable bones. A complete fracture in a pediatric patient indicates that the child was exposed to significant traumatic force. In rare instances, children may have an underlying medical condition that causes brittle bones; however, this would not be a safe assumption to make.

While assessing a woman in labor, the EMT notices that a loop of the umbilical cord is protruding from her vagina. In which position should the EMT place the patient? A) Left side with her head elevated B) Supine with her pelvis elevated C) Left side with her legs elevated D) Semi-sitting with her knees bent

B) Supine with her pelvis elevated Treatment for a prolapsed umbilical cord focuses on maintaining a pulsatile cord. Elevate the mother's pelvis; this will allow the baby to slide slightly back into the vagina, off of the cord. The knee-to -chest position may accomplish this same goal. Cover any exposed cord with a moist, sterile dressing, administer oxygen to the mother, and transport without delay.

Which of the following parameters is the LEAST reliable when assessing the perfusion status of a 2-year-old child? A) Capillary refill time B) Systolic blood pressure C) Skin color and temperature D) Presence of peripheral pulses

B) Systolic blood pressure You should not rely on the systolic blood pressure (SBP) when concluding the perfusion status of anyone. More reliable parameters include peripheral pulses, capillary refill time (most reliable in children younger than 6 years of age), and the condition and temperature of the skin. Remember that the body's compensatory mechanisms work to maintain the SBP, so when it falls, this corresponds to decompensated shock. A maintained SBP does NOT rule out shock!

Which of the following statements regarding lightning-related injuries is correct? A) The majority of patients who are struck by lightning die, even if CPR is provided immediately. B) The cardiovascular and nervous systems are most commonly injured during a lightning strike C) Because of the high electrical energy associated with lightning, full-thickness burns are a common finding. D) Multiple fractures, including those of the cervical spine, are the most common cause of lightning-related deaths.

B) The cardiovascular and nervous systems are most commonly injured during a lightning strike The cardiovascular and nervous systems are most commonly injured during a lightning strike; therefore, respiratory or cardiac arrest is the most common cause of lightning-related deaths, especially if CPR is delayed. A lightning strike typically induces asystole (cardiac standstill); however, this spontaneously resolves in some people or can be reversed with early, high-quality CPR. The energy associated with lightning is composed of direct current (DC) of up to 200,000 amperes and a potential of 100 million volts or more; however, the duration of a lightning strike is short. Therefore, skin burns are usually superficial; full-thickness (third-degree) burns are rare. Lightning causes massive contraction of all the body's muscles, potentially resulting in long bone and spinal fractures. Although this can clearly increase morbidity, it is not the most common cause of lightning-related deaths.

A SO-year-old man with a history of hypertension began experiencing chest pressure while mowing his lawn. After taking a baby aspirin and resting for approximately 10 minutes, the pressure in his chest completely resolved. Which of the following would BEST explain this? A) The patient's blood pressure increased after he ceased exertion, causing his chest pressure to resolve. B) The cessation of exertion decreased the workload of the heart and blood flow to the myocardium improved. C) The aspirin dissolved a small clot in a coronary artery and reestablished blood flow to that part of the heart. D) Blood flow to a part of the myocardium was restored because of const riction of a diseased coronary artery.

B) The cessation of exertion decreased the workload of the heart and blood flow to the myocardium improved. The patient's symptoms are consistent with angina. Angina is a condition in which cardiac oxygen demand exceeds the available supply; it indicates the presence of coronary artery disease. In this case, the patient's chest pressure resolved after he stopped exerting himself; this decreased the workload on the heart, provided for some dilation (not constriction) of the coronary arteries, and rebalanced cardiac oxygen supply and demand. As a result, blood flow through the diseased coronary artery improved and his chest pressure resolved. An increase in blood pressure, which increases cardiac workload, would not cause the patient's symptoms to resolve; it would likely make them worse. Aspirin will not dissolve a coronary thrombus; it will simply prevent platelets from clumping to the existing thrombus, so this would not explain why his symptoms resolved.

Which of the following statements regarding pediatric anatomy is correct? A) The child's trachea is more rigid and less prone to collapse. B) The occiput is proportionately larger when compared to an adult. C) Relative to the overall size of the airway, a child's epiglottis is smaller. D) Airway obstruction is common in children because of their large uvula.

B) The occiput is proportionately larger when compared to an adult. Compared to adults, infants and small children have a proportionately larger head, specifically the occiput (back of the head). Therefore, when positioning an infant's or child's airway, padding in between the shoulder blades is often needed to maintain neutral alignment of the head. Infants and children are at risk for an airway obstruction because their entire airway is smaller, not because their uvula is large. An infant's or child's trachea is less rigid than an adult's; therefore, it collapses more easily during respiratory distress. Relative to the overall size of an infant or a child's airway, the epiglottis is larger; it is also floppier.

A 60-year-old man presents with a right-sided headache and weakness to his left side. He is conscious and alert. His blood pressure is 190/100 mm Hg, pulse is 88 beats/min, and respirations are 14 breaths/min. His past medical history includes diabetes, arthritis, and hypertension. Which of the following should concern you the MOST? A) His blood glucose level B) The presence of hemiparesis C) His current blood pressure D) The location of his headache

B) The presence of hemiparesis Headaches are a common medical complaint and have numerous causes, most of which are not life threatening. Common causes of a headache include migraine and cluster headaches, sinusitis, and muscle tension, among others. In some patients, however, a headache can signal a serious problem. In this patient, you should be most concerned with his unilateral weakness (hemiparesis); this is not a common finding in patients with otherwise benign headaches and suggests a serious underlying problem (eg, intracranial bleeding, brain tumor). The fact that his headache is to the right side of his head and his weakness is to the left side of his body suggests that the problem lies in the right side of the brain. It is doubtful that his blood glucose level (BGL) is abnormal; he is conscious and alert. However, it would not be unreasonable to assess his BGL. His blood pressure, which is significantly elevated, could be causing his headache or may be the result of other factors, such as noncompliance with his antihypertensive medication, anxiety, or pain.

Which of the following BEST describes the mechanism of injury? A) The energy of an object in motion B) The way in which traumatic injuries occur C) Your concern for potentially serious injuries D) The product of mass, force of gravity, and height

B) The way in which traumatic injuries occur The mechanism of injury (MO!) is the way in which traumatic injuries occur; it describes the forces (or energy transmission) acting on the body that cause injury. Index of suspicion is your concern for potentially serious underlying and unseen (occult) injuries, which is based on your assessment of the MO!. A significant MO! (eg, fall from a significant height, ejection from a motor vehicle) should increase your index of suspicion for serious injuries. The energy of an object in motion is called kinetic injury. Potential injury is the product of mass (weight), force of gravity, and height; it is mostly associated with the energy of falling objects.

A 3-year-old girl is unresponsive and has slow, shallow breathing. Her heart rate is 65 beats/min and her skin is cyanotic. Her caregiver advises that the child ingested an unknown quantity of her narcotic painkillers. What should you do? A) Administer naloxone via the intranasal route B) Ventilate with a bag-mask device at 20 breaths/min C) Begin chest compressions at a rate of100 to 120/min D) Administer high-flow oxygen via nonrebreathing mask

B) Ventilate with a bag-mask device at 20 breaths/min The child's breathing is clearly inadequate and she is significantly hypoxemic. Initial treatment for an opioid overdose, regardless of the patient's age, is to optimize ventilation and oxygenation. Opioid-related deaths are the result of respiratory failure; the most immediate and effective way to prevent this is to ventilate the patient. For the child, provide 20 to 30 breaths/min (one breath every 2 to 3 seconds). The child is also significantly bradycardic, which is likely the result of a combination of the opioid and hypoxemia. After restoring ventilation and oxygenation, administer naloxone (Narcan) via the intranasal route, if allowed by local protocol, in a dose that is appropriate for the pediatric patient. If her heart rate falls below 60 beats/min despite adequate ventilation and oxygenation, begin chest compressions.

A 5-year-old child is unresponsive and cyanotic. His pulses are weak and slow, and his breathing is slow. What should you do? A) Begin chest compressions. B) Ventilate with a bag-mask device. C) Give oxygen by nonrebreathing mask. D) Apply the automated external defibrillator.

B) Ventilate with a bag-mask device. Most cardiac arrests in infants and children are caused by hypoxia secondary to respiratory failure. Unlike adults, they rarely develop dysrhythmias such as ventricular fibrillation. When infants and children become hypoxic, they develop bradycardia. Any infant or child who has bradycardia and poor breathing should be ventilated with a bag-mask device immediately. The presence of cyanosis indicates severe hypoxia and must be treated aggressively. If the child's heart rate falls below 60 beats/min despite adequate bag-mask ventilation, begin chest compressions. The automated external defibrillator (AED) is only indicated for patients in cardiac arrest; the child in this scenario is not in cardiac arrest.

A 5-year-old child in compensated shock secondary to severe vomiting and diarrhea would be expected to have: A) slow, shallow respirations. B) a slow capillary refill time. C) strong, bounding radial pulses. D) a weakly palpable carotid pulse.

B) a slow capillary refill time. Compensated shock in the infant or child is characterized by poor peripheral perfusion (eg, delayed[> 2 seconds] capillary refill time, weak peripheral pulses, pallor), tachycardia, and tachypnea. In compensated shock, the child's blood pressure is maintained and his or her mental status is adequate. In decompensated shock, the child's compensatory mechanisms have failed; blood pressure falls, central pulses become weak, and mental status begins to deteriorate. It is critical to recognize an infant or a child in compensated shock, begin immediate treatment, and transport without delay.

A 65-year-old man has generalized weakness and chest pressure. He has a bottle of prescribed nitroglycerin, but states that he has not taken any ofhi medication. The EMT should: A) apply the AED and prepare the patient for immediate transport. B) administer up to 325 mg of aspirin if the patient is not allergic to it. C) assist the patient with his nitroglycerin with medical control approval. D) perform a secondary assessment and obtain baseline vital signs.

B) administer up to 325 mg of aspirin if the patient is not allergic to it. Aspirin has clearly been shown to reduce mortality and morbidity associated with acute coronary syndrome (ACS) and should be given as early as possible; the dose is 160 to 325 mg. Even though this patient has chest pain and prescribed nitroglycerin, you must first complete a secondary assessment and obtain baseline vital signs. Medical control will need this information--specifically, the patient's blood pressure--to determine whether you should assist the patient with his nitroglycerin. The AED is not indicated for this patient because he is not in cardiac arrest.

During transport of a woman in labor, the patient tells you that she feels the urge to push. You assess her and see the top of the baby's head bulging from the vagina. You should: A) allow the head to deliver and check for the location of the cord. B) advise your partner to stop the ambulance and assist with the delivery. C) ask the mother to take short, quick breaths until you arrive at the hospital. D) apply gentle pressure to the baby's head and notify the hospital immediately.

B) advise your partner to stop the ambulance and assist with the delivery. If, during transport, the mother begins to deliver the infant, your first action should be to advise your partner to stop the ambulance and assist you with the delivery. Delivery of a baby should NEVER be attempted in the back of a moving ambulance; it is not safe for you, the mother, or the baby. During delivery, you should apply gentle pressure to the top of the baby's head (be careful of the fontanelles) to prevent an explosive delivery. After the head is delivered, you should quickly run your fingers around its neck to determine if the cord is wrapped around its neck (nuchal cord). If a nuchal cord is not present, suction the baby's mouth and nose and continue with the delivery.

You and your partner are performing CPR on a 2-year-old woman in cardiac arrest. During your resuscitation attempt, you should: A) hyperventilate her because she is severely hypoxic. B) allow the chest to fully recoil in between compressions. C) perform compressions and ventilations at a ratio of 30:2. D) attach the AED pads after 5 minutes of high-quality CPR.

B) allow the chest to fully recoil in between compressions. When performing two-rescuer CPR on an infant or a child, use a compression to ventilation ratio of 15:2 (30:2 for one-rescuer infant or child CPR), compress the chest by one-third the depth of the chest (about 2 inches), and allow the chest to fully recoil between compressions. Full recoil of the chest is essential to high-quality CPR; it maximizes the amount of blood that returns to the heart, which maximizes the amount of blood ejected from the left ventricle during chest compressions. Do not hyperventilate any patient; deliver each breath over 1 second while observing the chest for visible rise. Hyperventilation causes gastric distention and increases the risk of aspiration if regurgitation occurs. Furthermore, hyperventilation causes a reduction in blood return to the heart because it hyperinflates the lungs and puts pressure on the heart. Attach pediatric AED pads as soon as possible, analyze the child's cardiac rhythm, and deliver a single shock if indicated.

A patient is unresponsive with snoring respirations. His arm is amputated just above the elbow and is bleeding heavily. The EMT should: A) open the patient's airway. B) apply a proximal tourniquet. C) administer high-flow oxygen. D) ventilate with a bag-mask device.

B) apply a proximal tourniquet. Major hemorrhage kills patients faster than a compromised airway, so in this case, bleeding control has the highest priority. The EMT should apply a proximal tourniquet and stop the bleeding immediately. Attention can then turn to the patient's airway and breathing status. As with any patient, treatment priorities must focus on injuries or conditions that will be the MOST immediately fatal.

You assess an unresponsive 65-year-old man and find that he is apneic and pulseless. The patient's wife tells you that he has an automatic implanted cardioverter/defibrillator. After initiating CPR, you should: A) deactivate the implanted defibrillator by running a magnet over it. B) apply the AED as soon as possible and analyze his cardiac rhythm. C) ask the wife why and when he had the automatic defibrillator implanted. D) avoid using the AED because the implanted defibrillator is more effective.

B) apply the AED as soon as possible and analyze his cardiac rhythm. Some patients who are at high risk for sudden cardiac arrest due to ventricular fibrillation (V-Fib) have a small automatic implanted cardioverter/defibrillator (Al CD). The AICD attaches directly to the heart and continuously monitors the cardiac rhythm, delivering a shock if V-Fib or another lethal dysrhythmia is detected. Regardless of whether the patient has an Al CD, he or she should be treated like all other cardiac arrest patients. Pe1form CPR and use the AED as usual; however, you should ensure that the AED pads are at least 1 inch away from the implanted device. Generally, the electricity from the Al CD is so low that it will have no effect on rescuers and therefore should not be of concern to you. Do not deactivate an implanted AICD, especially if it is working and delivering shocks as it is supposed to. When treating a cardiac arrest patient who has an AICD, your priority is to provide CPR and defibrillate with the AED if indicated, not to determine when and why the AICD was implanted.

A 28-year-old woman has severe lower quadrant abdominal pain. When assessing her abdomen, you should: A) ask her where the pain is located and palpate that area first. B) ask her where the pain is located and palpate that area last. C) auscultate for bowel sounds for approximately 2 to 5 minutes. D) encourage the patient to lie supine with her legs fully extended.

B) ask her where the pain is located and palpate that area last. Assessment of a patient's abdomen includes asking where the pain is located and then palpating that area last. Palpating the painful area first may interfere with the rest of your assessment because of the significant pain the patient will be in. Bowel sounds are of little value in the field and generally are not included in the abdominal assessment. Patients with severe abdominal pain typically prefer to lie on their side with their knees drawn up into their chest (fetal position). Moving them from this position will aggravate their pain.

After defibrillating a man in cardiac arrest, you resume CPR. As you are about to reanalyze his cardiac rhythm 2 minutes later, your partner tells you she can definitely feel a strong carotid pulse. You should: A) continue with the rhythm analysis. B) assess the patient's breathing effort. C) ask her to obtain a blood pressure reading. D) remove the AED pads from the patient's chest.

B) assess the patient's breathing effort. If return of spontaneous circulation (ROSC) occurs, your first action should be to reassess the patient's airway status and breathing effort. If the patient is still apneic or is breathing inadequately, continue ventilations and frequently reassess his or her pulse. If the patient is breathing adequately, administer oxygen as needed to maintain an oxygen saturation of greater than 94%. If the patient is still unresponsive, as is often the case, insert an appropriate airway adjunct (if not already done) to assist in maintaining airway patency. After reassessing airway and breathing, obtain the patient's blood pressure and treat hypotension if needed. Do not remove the AED pads from the patient's chest, even if ROSC has occurred. The risk of cardiac arrest is still high and the patient may need further defibrillation. You should, however, disconnect the pads from the AED or simply turn the AED off.

A 30-year-old man with a history of diabetes is found unresponsive on his couch. His skin is cool and clammy, his pulse is rapid and weak, and his breathing is rapid and shallow. His blood glucose level reads 20 mg/dL. The EMTs should: A) place oral glucose in between his cheeks and gums. B) assist his ventilations and transport without delay. C) request a paramedic unit to administer his insulin. D) place him on his side, keep him cool, and transport.

B) assist his ventilations and transport without delay. A blood glucose level of 20 mg/dL is dangerously low. Combined with the patient's other signs and symptoms, this presentation is classic for insulin shock. However, because the patient is unresponsive, you cannot administer oral glucose because he could aspirate it. His rapid, shallow (ineffective) breathing, however, is something that you can address. Assist his ventilations with a bag-mask device, keep him warm, and transport without delay. A paramedic intercept should strongly be considered (if available); however, they would NOT administer insulin because that would only lower his blood glucose further. They would administer intravenous glucose.

An unresponsive patient with multi-system trauma has slow, shallow breathing; weak radial pulses; and severe bleeding from a lower extremity wound. You should direct your partner to: A) radio for a paramedic ambulance to respond to the scene. B) assist the patient's ventilations while you control the bleeding. C) apply oxygen via nonrebreathing mask while you control the bleeding. D) prepare the long spine board and straps for rapid spinal immobilization.

B) assist the patient's ventilations while you control the bleeding. The goal of the primary assessment is to rapidly identify and correct all life-threatening injuries or conditions. In the case of this patient, as your partner maintains in-line cervical spine control, he or she should assist the patient's ventilations. An unresponsive patient with slow, shallow breathing is not breathing adequately and should be treated with ventilatory assistance, not a nonrebreathing mask. As your partner is managing the patient's airway and providing ventilatory assistance, you should apply direct pressure (or a tourniquet, if needed) to the extremity wound to control the bleeding. It is important for you and your partner to work together so that all life threats can be corrected as soon as possible. Most EMS systems work with two-person crews and do not have the luxury of a third EMT If the police or fire department is on the scene, you can ask them to gather equipment for you. The request for an ALS ambulance is based on factors such as the patient's condition and transport time to the closest appropriate hospital.

A 55-year-old woman with a history of diabetes is found unresponsive with rapid, shallow respirations. The patient's husband tells you that he does not know when his wife last took her insulin. Management of this patient should include: A) assisted ventilations and oral glucose. B) assisted ventilations nd rapid transport. C) oral glucose and oxygen via nonrebreathing mask. D) subcutaneous injection of insulin and 100% oxygen.

B) assisted ventilations nd rapid transport. Without knowing if and when the patient last took her insulin, it is difficult to determine if she is experiencing diabetic coma or insulin shock. Nonetheless, her rapid, shallow respirations--which are likely not producing adequate tidal volume--should be treated with ventilation assistance. Because she is unresponsive and obviously unable to swallow, oral glucose is contraindicated. Assess the patient's blood glucose level and pass this information along to the hospital; if possible, arrange for a paramedic intercept so she can receive intravenous dextrose. If the patient is experiencing diabetic coma, insulin is what she truly needs; however, insulin is rarely, if ever, administered in the prehospital setting, even by paramedics. After ensuring adequate oxygenation and ventilation, transport the patient without delay.

In patients with heart disease, acute coronary syndrome is MOST often the result of: A) atrial damage. B) atherosclerosis. C) coronary artery spasm. D) coronary artery rupture.

B) atherosclerosis. In most patients with acute coronary syndrome {ACS; eg, unstable angina, acute myocardial infarction), atherosclerosis is the underlying problem that causes heart disease. Athero clerosis is a disorder in which calcium and a fatty material called cholesterol build up and form a plaque inside the walls of blood vessels, obstructing blood flow. ACS due to atherosclerosis usually occurs when a fragment of plaque ruptures and occludes a coronary artery; further occlusion occurs when platelets aggregate in the area and clump together. Less commonly, acute coronary artery spasm may result in ACS. The cause of acute coronary vasospasm is largely unknown. Rupture of a coronary artery is a rare cause of ACS. Atrial or ventricular damage is usually caused by, rather than the cause of, ACS.

Activated charcoal is contraindicated for a patient who is: A) emotionally upset and has ingested two bottles of aspirin. B) awake and alert and has swallowed a commercial drain cleaner. C) agitated and claims to have ingested a bottle of Tylenol. D) conscious and alert and has ingested a large amount of Motrin.

B) awake and alert and has swallowed a commercial drain cleaner. Activated charcoal adsorbs (binds to) many ingested substances, preventing them from being absorbed into the body by the stomach or intestines. In some cases, you may give activated charcoal to patients who have ingested certain substances, if approved by medical control or local protocol. Activated charcoal is contraindicated for patients who have ingested an acid or alkali (ie, drain cleaner) or a petroleum product (ie, gasoline), who have a decreased level of consciousness and cannot protect their own airway, or who are unable to swallow.

After 30 seconds of positive pressure ventilation, a newborn's heart rate is 50 beats/min and its face and trunk are cyanotic. The EMT should: A) give blow-by oxygen. B) begin chest compressions. C) increase the ventilation rate. D) resuction the mouth and nose.

B) begin chest compressions. Immediate treatment for newborn bradycardia (heart rate less than 100 beats/min) involves positive pressure ventilation. If the heart falls below 60 beats/min despite positive pressure ventilation, you should begin chest compressions. Central cyanosis (cyanosis to the face and trunk) and severe bradycardia indicate significant hypoxia; if immediate corrective action is not taken, cardiopulmonary arrest could follow. Blow-by oxygen is appropriate for newborns with central cyanosis who are otherwise breathing adequately and have a heart rate that is greater than 100 beats/min. The mouth and nose should be suctioned if secretions are present. Increasing the ventilation rate could cause more harm secondary to increased intrathoracic pressure.

Following the initial steps of resuscitation, a newborn remains apneic and cyanotic. You should: A) immediately resuction its mouth and nose. B) begin ventilations with a bag-mask device. C) gently flick the soles of its feet for up to 60 seconds. D) start CPR if the heart rate is less than 80 beats/min.

B) begin ventilations with a bag-mask device. The initial steps of newborn resuscitation, which are performed on all newborns following delivery, include drying, warming, positioning, suctioning, and tactile stimulation. If the newborn remains apneic (or is gasping) after the initial steps of resuscitation, or has a heart rate less than 100 beats/min, you should begin positive pressure ventilations (PPV) with a bag-mask device at a rate of 40 to 60 breaths/min. Continued tactile stimulation (eg, flicking the soles of the feet, rubbing the lateral thorax) of an apneic newborn wastes time; you must ventilate at once. If the newborn's heart rate is less than 60 beats/min despite effective PPV, you should begin chest compressions.

Hypoglycemia and acute ischemic stroke can present similarly because: A) the most common cause of stroke is hypoglycemia. B) both oxygen and glucose are needed for brain function. C) the majority of stroke patients have a history of diabetes. D) both are caused by low levels of glucose in the blood.

B) both oxygen and glucose are needed for brain function. Although stroke and hypoglycemia are two distinctly different conditions, their signs and symptoms are often similar. This is because the brain requires both oxygen and glucose to function normally. An acute ischemic stroke is caused by a lack of oxygen to a part of the brain due to a blocked cerebral artery, whereas hypoglycemia (low blood glucose level) deprives the entire brain of glucose. In either case, the patient presents with signs of impaired brain function (ie, slurred speech, weakness, altered mental status). Both conditions may lead to permanent brain damage or death if not treated promptly.

A patient who overdosed on methamphetamine would be expected to have all of the following clinical signs, EXCEPT: A) agitation. B) bradycardia. C) hypertension. D) dilated pupils.

B) bradycardia. Methamphetamine, an upper, stimulates the central nervous system, causing it to release excessive amounts of adrenaline (epinephrine). Epinephrine increases heart rate and blood pressure; therefore, the patient would experience tachycardia and hypertension. Other signs of methamphetamine overdose, which also indicate an adrenaline surge, include pupillary dilation, agitation, and hyperthermia. Bradycardia would be expected in patients who have overdosed on drugs that suppress-not stimulate-the central nervous system (eg, narcotics, benzodiazepines, barbiturates).

You should suspect that your patient has pulmonary edema if he or she: A) has swollen feet and ankles B) cannot breathe while lying down. C) is hypertensive and tachycardic. D) has a dry, nonproductive cough.

B) cannot breathe while lying down. Pulmonary edema is often caused by failure of the left side of the heart. When the patient is lying down, he or she experiences worsened difficulty breathing (orthopnea) because more blood backs up in the lungs. Patients with severe pulmonary edema often produce pink, frothy sputum when they cough; this is another sign of blood backing up in the lungs. A dry, nonproductive cough is not common. Hypertension and tachycardia are common in patients with pulmonary edema; however, many other conditions can cause these findings. Swelling of the feet and ankles is commonly seen in patients with right heart failure, and occurs when blood backs up beyond the right atrium; it is not a common sign of left hear-t failure and pulmonary edema.

A 4-year-old boy with a tracheostomy tube has intercostal retractions, a heart rate of 80 beats/min, and an oxygen saturation of 85%. During his attempts to breathe, a gurgling sound is heard in the tracheostomy tube. You should: A) ventilate through the tracheostomy tube. B) carefully suction the tracheostomy tube. C) remove the tracheostomy tube and clean it. D) place an oxygen mask over the tracheostomy tube.

B) carefully suction the tracheostomy tube. Obstruction of a tracheostomy tube with thick secretions is a common, and potentially life -threatening, complication. You must first suction the tube to ensure that it is clear of secretions; this will often improve the patient's condition. Do not vigorously suction the tube, however, as this may induce or worsen bradycardia. Placing an oxygen mask over an obstructed tracheostomy tube will be of little to no benefit. If the child's condition does not improve following suctioning (eg, he remains bradycardic, his oxygen saturation remains low), attach a bag-mask device to the tracheostomy tube and begin ventilating him. Do not remove the tracheostomy tube; this is beyond the EMT's scope of practice, plus there is no guarantee that you will be able to replace it.

The MOST important initial steps of assessing and managing a newborn include: A) suctioning the airway and obtaining a heart rate. B) clearing the airway and keeping the infant warm. C) keeping the infant warm and counting respirations. D) drying and warming the infant and obtaining an APGAR score.

B) clearing the airway and keeping the infant warm. In the initial steps of assessing and managing the newborn, the most important aspects include clearing the airway of amniotic fluid and making sure that the baby stays warm. The APGAR score should not be relied on as the initial indicator for resuscitation because it is not performed until the child is 1 minute old. Clearly, this is too long to wait before assessment and intervention. After the airway has been cleared and the newborn has been warmed, the respirations, heart rate, color, and oxygen saturation (Sp02) should be assessed and managed accordingly.

Immediately following a generalized motor seizure, most patients are: A) apneic. B) confused. C) hyperactive. D) awake and alert.

B) confused. After a generalized motor seizure, the patient typically will be confused, sleepy, or, in some cases, combative. This is referred to as the postictal phase. The patient's level of consciousness typically improves within 30 minutes. In many cases, the patient's respirations will be fast (tachypnea) following a seizure; this is the body's attempt to eliminate excess carbon dioxide that accumulated in the blood during the seizure. Tachycardia is also common immediately following a generalized motor seizure.

A 30-year-old man with a history of schizophrenia cut his wrists and is bleeding profusely. He is confused and combative and has slurred speech. With the assistance of law enforcement personnel, you and your partner physically restrain him so that you can provide care and transport. In this situation, a court of law would MOST likely: A) determine that the patient had decision-making capacity. B) consider your actions in providing care to be appropriate. C) conclude that you should have had a court order to restrain the patient. D) agree that you and your partner are guilty of assault and battery.

B) consider your actions in providing care to be appropriate. An adult with decision-making capacity (ie, a mentally competent adult) has the legal right to refuse medical treatment, even if that treatment involves lifesaving care. In psychiatric cases, however, a court of law would likely consider your actions in providing lifesaving care to be appropriate, particularly if you have a reasonable belief that the patient will harm himself or herself or others without your intervention. In addition, a patient who is in any way impaired, whether by mental illness, medical condition, or intoxication, may not be considered competent to refuse treatment and transport. If you are unsure of a patient's decision-making capacity, err on the side of treatment and transport. Few would argue that it would be easier to defend why you treated a patient than to justify or defend why you abandoned a patient.

A high school student was splashed in the eyes with a strong acid chemical during a lab experiment. He is in severe pain and is unable to open his eyes. You should: A) flush both eyes with sterile water for no more than 5 minutes. B) continuously flush his eyes with saline for at least 20 minutes. C) force his eyes open and assess for the presence of severe burns. D) cover both of his eyes with sterile gauze and transport at once.

B) continuously flush his eyes with saline for at least 20 minutes. Chemical burns to the eyes, usually caused by acid or alkaline solutions, require immediate emergency care. This consists of flushing the eyes with water or a sterile saline irrigation solution. Forcing the eyes open for the expressed purpose of assessing for burns is impractical and wastes time. You may have to force the eyes open, however, to effectively irrigate them. If sterile water is not available, use any clean water. Irrigate the eyes for at least 5 minutes. If the burn was caused by an alkali or strong acid, you should irrigate the eyes continuously for 20 minutes. If irrigation can be carried out effectively in the ambulance, it should be done during transport to save time. Strong acids and alkaline solutions can penetrate deeply, requiring prolonged irrigation. After you have completed irrigation, cover the eyes with clean, dry dressings.

A 30-year-old woman has an open deformity to her left leg and is in severe pain. She is conscious and alert, has a patent airway, and is breathing adequately. Your primary concern should be: A) administering high-flow oxygen. B) controlling any external bleeding. C) assessing pulses distal to the injury. D) covering the wound to prevent infection.

B) controlling any external bleeding. Initial care for any open injury involves controlling external bleeding. Further care involves manually stabilizing the injury site; applying a sterile dressing to keep gross contaminants from entering the wound; assessing distal perfusion (eg, a pulse), motor, and sensory functions; and stabilizing the injury with an appropriate splint. The patient in this scenario is conscious and alert, has a patent airway, and is breathing adequately. Depending on other assessment findings, oxygen may be indicated. Your primary concern, however, should be to ensure that all external bleeding has been controlled.

A 7-year-old child has an altered mental status, high fever, and a generalized rash. You perform your assessment and administer supplemental oxygen. En route to the hospital, you should be MOST alert for: A) hypotension. B) convulsions. C) combativeness. D) respiratory distress.

B) convulsions. High fever and an altered mental status indicate sepsis (severe infection). A generalized rash should alert you to the possibility of meningitis, a condition caused by infection and inflammation of the meninges that protect the brain and spinal cord. Children with meningitis are at risk for seizures (convulsions), usually due to increased intracranial pressure (lCP) and/or high fever; therefore, you must continually monitor the child's condition en route to the hospital and be prepared to treat seizures if they occur. Remember that seizure deaths are caused by cerebral hypoxia. You should also be alert for vomiting, which can jeopardize the airway. Hypotension can occur in patients with sepsis and should also be of concern; however, seizures directly compromise ventilation and oxygenation.

A young man experienced a syncopal episode after working in the heat for several hours. He is conscious and alert; has cool, clammy skin; and complains of nausea and lightheadedness. You should: A) provide rapid cooling. B) transport him on his side. C) give him cold water to drink. D) advise him to go home and rest.

B) transport him on his side. The patient is experiencing heat exhaustion and should be transported to the hospital for evaluation, especially since he experienced a syncopal episode (fainting). Because he is nauseated, he should not be given anything to drink and should be placed on his side during transport to prevent aspiration if he vomits. Loosen any restrictive clothing that may trap heat. Rapid cooling is indicated for patients with heatstroke, the signs of which include an altered mental status and hot, flushed skin (dry or moist).

Following a head injury, a young woman is semiconscious and is bleeding from the nose and left ear. You should: A) place a pressure dressing over her ear to prevent blood loss B) cover her ear and nose with a loose gauze pad to collect the blood. C) control the bleeding from her nose by pinching her nostrils closed. D) insert a nasal airway to keep her tongue from blocking the airway.

B) cover her ear and nose with a loose gauze pad to collect the blood. Blood draining from the ears or nose following a head injury may contain cerebrospinal fluid (CSF) and indicates a skull fracture. In these cases, do NOT attempt to stop the flow of blood. Applying excessive pressure may force the blood leaking from the ears or nose to collect within the cranium, which could increase intracranial pressure and cause permanent damage. Loosely cover the ears or nose with a sterile gauze pad to collect the blood and help keep contaminants out (patients with a skull fracture and CSF leakage are at risk for meningitis). The nasopharyngeal (nasal) airway is contraindicated in patients with a possible skull fracture, especially if blood is draining from the nose. Although rare, the airway adjunct may inadvertently enter the cranial vault through the fracture.

A 44-year-old man has a traumatic leg amputation just below the knee. He is lying in a large pool of blood and the wound is bleeding profusely. The EMT should: A) locate the femoral artery and apply pressure to it until the bleeding stops. B) cover the wound with a trauma dressing and apply a proximal tourniquet. C) apply an icepack to the wound to constrict the vessels and stop the bleeding. D) apply a pressure dressing and elevate the injured extremity at least 12 inches.

B) cover the wound with a trauma dressing and apply a proximal tourniquet. In most cases, external bleeding can be controlled with direct pressure and a securely placed pressure dressing. However, if this is unsuccessful, you should apply a proximal tourniquet immediately or the patient will bleed to death. Of the options listed, covering the wound with a trauma dressing (while applying direct pressure) and then applying a proximal tourniquet will be the most effective means of controlling this severe hemorrhage. Evidence has shown that locating and applying adequate pressure to an arterial pressure point is often difficult and time-consuming; the patient in this scenario does not have that kind of time!

The bite of a brown recluse spider is characterized by: A) immediate pain, swelling at the site, and painful muscle spasms. B) delayed onset of pain, swelling, and blister formation at the site. C) two small puncture marks, swelling, and delayed onset of pain. D) rapid swelling within 30 minutes and a decline in mental status.

B) delayed onset of pain, swelling, and blister formation at the site. The venom of a brown recluse spider is cytotoxic; that is, it causes tissue and cellular damage. The bite itself is usually painless; however, the patient typically begins to complain of pain within a few hours. The area becomes swollen and tender, developing a pale, mottled, cyanotic center and possibly a small blister. Over the next several days, a scab of dead skin, fat, and debris will form and dig down into the skin, producing a large ulcer that may not heal unless promptly treated. The bite of a brown recluse spider is very small; therefore, puncture marks are not visible. Unlike the brown recluse spider, the black widow spider bite is characterized by immediate pain. Its venom is neurotoxic; that is, it suppresses the central nervous system. The patient typically complains of intense muscle spasms, especially of the abdomen, and systemic signs of nervous system involvement (eg, dizziness, chest pain, difficulty breathing).

The correct method for inserting an oropharyngeal airway in a small child is to: A) insert the airway with the curvature toward the roof of the mouth and then rotate it 180 degrees. B) depress the tongue with a tongue blade and insert the airway with the downward curve facing the tongue. C) open the airway with the tongue-jaw lift maneuver and insert the airway until you meet slight resistance. D) insert the airway as you would in an adult, but use an airway that is one size smaller than you would normally use.

B) depress the tongue with a tongue blade and insert the airway with the downward curve facing the tongue. Keeping in mind that a child's tongue is proportionately large, the correct method for inserting an oropharyngeal (oral) airway is to use a tongue blade to depress the tongue and slide the airway straight in, with the downward curve of the airway facing the tongue, until it rests just beyond the curvature of the tongue. If you use an oral airway that is too small, it will not reach the curvature of the tongue and propel it forward. If you use an oral airway that is too large, it may obstruct the airway. If you meet resistance when inserting an oral airway, you are likely using an airway that is too large. Oral airways are rigid, and the hard palate of a child is rather fragile. An improperly inserted oral airway could lacerate or fracture a child's hard palate.

A patient reports pain in the upper midabdominal area. This region of the abdomen is called the: A) peritoneum. B) epigastrium. C) mediastinum. D) retroperitoneum.

B) epigastrium. The mid-upper region of the abdomen is referred to as the epigastrium because of its location over the stomach (epi = upon, gastric= stomach). This is a common site of pain or discomfort in patients experiencing a cardiac problem, which frequently causes them to attribute their pain or discomfort to indigestion

The presence of subcutaneous emphysema following blunt trauma to the anterior neck should make you MOST suspicious for a: A) pneumothorax. B) fractured larynx. C) ruptured esophagus. D) carotid artery injury.

B) fractured larynx. Crushing or blunt trauma to the anterior neck can injure the trachea or larynx. Once the cartilages of the upper airway and larynx are fractured, they do not spring back to their normal position. Such a fracture can lead to loss of voice, airway obstruction, and leakage of air into the soft tissues of the neck. Air leakage into the soft tissues is called subcutaneous emphysema. Subcutaneous emphysema may also be observed in patients with a tension pneumothorax, although it is typically located in the chest. Esophageal rupture would likely present with difficulty swallowing (dysphagia) and vomiting blood (hematemesis). You should suspect injury to a carotid artery or jugular vein if you observe a rapidly expanding hematoma to the neck following blunt trauma.

A 36-year-old man who is a known diabetic, presents with severe weakness, diaphoresis, and tachycardia. He is conscious, but confused. His blood pressure is 110/58 mm Hg, pulse is 120 beats/min and weak, and respirations are 24 breaths/min. The glucometer reads error after several attempts to assess his blood glucose level. Medical control will MOST likely order you to: A) assist the patient in taking his insulin. B) give at least one tube of oral glucose C) transport only and closely monitor him. D) give him a salt-containing solution to drink.

B) give at least one tube of oral glucose The patient"s signs and symptoms are consistent with hypoglycemia. When you are in doubt as to a patient's blood glucose level, you should err on the side of caution and give sugar; this is what medical control will likely order you to do. The patient, although confused, is conscious and will likely be able to swallow. Insulin is not administered to patients in the field, even if hyperglycemia is documented; EMTs and paramedics are usually not familiar with all of the different types of insulin and their respective doses, and profound hypoglycemia, potentially resulting in death, can occur if too much insulin is given. After giving oral glucose, reassess the patient's blood glucose level, mental status, and vital signs.

A patient who is experiencing an acute myocardial infarction: A) most often describes his or her chest pain as being sharp or tearing. B) has chest pain or discomfort that does not change with each breath. C) often experiences relief of his or her chest pain after taking nitroglycerin. D) often complains of a different type of pain than a patient with angina.

B) has chest pain or discomfort that does not change with each breath. The type of chest pain or discomfort associated with acute myocardial infarction (AMI) is the same that is experienced by patients with angina pectoris (eg, dull, crushing, pressure, heaviness); thus, you cannot distinguish AMI from angina pectoris based solely on the type or quality of pain. Furthermore, the pain associated with AMI, like that of angina, often radiates to the arm, jaw, back, or epigastrium. Relative to other causes of chest pain or discomfort (eg, pleurisy, pneumothorax), the pain associated with AMI and angina does not worsen or improve when the patient takes a breath. Rest and nitroglycerin often relieve the pain associated with stable angina, but are less likely to relieve the pain associated with unstable angina or AMI.

All of the fo llowing are signs of gastrointestinal bleeding, EXCEPT: A) melena. B) hemoptysis. C) tachycardia. D) hematemesis.

B) hemoptysis. Signs and symptoms of gastrointestinal (GI} bleeding include abdominal pain; vomiting blood (hematemesis); the passage of dark, tarry stools (melena); and bright red rectal bleeding (hematochezia). If blood loss is significant, the patient may have signs of shock (eg, tachycardia, diaphoresis, tachypnea, hypotension). Hemoptysis (coughing up blood) is a sign of a pulmonary injury or left side heart failure with pulmonary edema, not GI bleeding.

Shock following major trauma is MOST often the result of: A) a head injury B) hemorrhage. C) spinal injury. D) long bone fractures.

B) hemorrhage. Shock following major trauma is usually caused by hemorrhage (bleeding), which can be external and obvious (gross), internal and hidden (occult), or both. Trauma to the chest and/or abdomen and multiple long bone fractures are common causes of hemorrhage that result in shock. An isolated head injury usually does not cause shock; rather, it causes increased intracranial pressure. If the patient with a seemingly isolated head injury has signs of shock, look for other injuries. Major trauma may also be associated with spinal injury. If the spinal cord is injured, the patient may develop shock because the nerves that control the diameter of the blood vessels are damaged, resulting in widespread vasodilation (neurogenic shock).

A 69-year-old man is unresponsive. He has a weak carotid pulse at 40 beats/min and his skin is cool and pale. From this information, the EMT should conclude that: A) he is in ventricular fibrillation. B) his cardiac output is reduced. C) he has a blocked coronary artery. D) his systolic BP is at least 90 mm Hg.

B) his cardiac output is reduced. Although the patient's pulse is slow and weak, it is present; this rules out ventricular fibrillation (V-Fib) because V-Fib does not produce a pulse. Without a 12- lead ECG and other advanced diagnostic tools, there is no way of knowing if the patient has a blocked coronary artery. An unresponsive patient with severe bradycardia; a weak pulse; and pale, cool skin likely has a BP that is less than 90 mm Hg. Therefore, the EMT should conclude from this patient's presentation that his cardiac output is reduced.

The function of the uterus is to: A) dilate and expel the baby from the cervix. B) house the fetus as it grows for 40 weeks. C) provide oxygen and other nutrients to the fetus. D) provide a cushion and protect the fetus from infection.

B) house the fetus as it grows for 40 weeks. The uterus is a muscular organ where the fetus grows for 39 to 40 weeks. It is responsible for contracting during labor, which in conjunction with dilation of the cervix (the opening of the uterus), expels the baby from the uterus into the birth canal. The placenta is the organ of exchange that delivers oxygen and other nutrients from the mother to the fetus and returns metabolic waste products from the fetus to the mother. The amniotic sac, also called the bag of waters, provides a cushion for the developing fetus and helps protect it from infection.

After attaching the AED to a 7-year-old child in cardiac arrest, you push the analyze button and receive a shock advised message. After delivering the shock, you should: A) assess for a carotid pulse. B) immediately perform CPR. C) reanalyze the cardiac rhythm. D) open the airway and ventilate.

B) immediately perform CPR. After the AED delivers a shock, you should immediately begin or resume CPR, starting with chest compressions. Perform CPR for 2 minutes and then reanalyze the child's cardiac rhythm. If the AED states that a shock is advised, defibrillate without delay and then continue CPR. If the AED states no shock advised, resume CPR and reassess in 2 minutes. Do not check for a pulse after defibrillation or if the AED gives a no shock message; this merely causes an unnecessary delay in performing chest compressions. Continue CPR and cardiac rhythm analysis and defibrillation (if indicated) every 2 minutes until ALS personnel arrive or the patient starts to move spontaneously.

A man was kicked in the head and chest and stabbed in the abdomen. He is conscious, but restless, and is coughing up blood. His breathing is rapid and shallow, his skin is cool and pale, and his pulse is rapid and weak. The EMT should recognize that this patient's signs and symptoms are MOST likely the result of: A) a non-patent airway. B) internal hemorrhage. C) a spinal cord injury. D) severe brain trauma.

B) internal hemorrhage. The classic symptoms of hemorrhagic shock are present in this patient. He is restless, tachypneic, tachycardic, pale, and diaphoretic. The EMT should suspect that he is bleeding into his abdomen, chest, or both. Although the patient's rapid, shallow breathing may be inadequate, there is no evidence that his airway is non-patent. Although a head injury cannot be ruled out, the patient's symptoms are not indicative of what you would expect to encounter with severe brain trauma (ie, decreased LOC, hypertension, bradycardia, irregular breathing, posturing). Spinal cord injury is also unlikely as the signs of neurogenic shock (ie, warm, dry skin; normal [or slow] heart rate) are not present.

A newborn is considered to be premature if it: A) weighs less than 6.5 pounds. B) is born before 37 weeks' gestation. C) is born to a heroin-addicted mother. D) has meconium in or around its mouth.

B) is born before 37 weeks' gestation. A term gestation is between 37 and 42 weeks. A premature newborn is one that is born before 37 weeks' gestation or weighs less than 5.5 pounds (2.5 kg). Compared to women who do not abuse drugs, smoke, or drink alcohol during pregnancy, women who do are more likely to deliver prematurely or to deliver a low-birth-weight baby. The risk of the fetus voiding its first bowel movement (meconium) in utero increases any time the fetus is distressed, regardless of its gestational age or weight; the presence of meconium does not define a premature newborn.

Witnesses state that a 49-year-old man had a seizure that lasted approximately 5 minutes. If the patient truly experienced a seizure, you would MOST likely find that he: A) has a slow heart rate. B) is confused and disoriented. C) is apneic and cyanotic. D) has a low blood pressure.

B) is confused and disoriented. Most generalized tonic-clonic seizures are followed by a postictal phase, in which the patient is unresponsive at first and then gradually regains consciousness. In most cases, the postictal patient appears dazed, confused, or disoriented; in some cases, he or she is combative. The postictal phase typically lasts from 5 to 30 minutes. During the seizure, the patient is often not breathing or is breathing inadequately; however, when the seizure stops, breathing usually resumes. You will often find that postictal patients are tachycardic and hypertensive; this is the result of an adrenaline surge that occurs during the the seizure.

The energy setting for a biphasic AED: A) is typically 360 joules. B) is manufacturer specific. C) must be manually set by the EMT. D) increases by 50 joules with each shock.

B) is manufacturer specific. A biphasic AED sends energy in two directions simultaneously. This is advantageous because it produces a more efficient defibrillation than monophasic defibrillation, which sends energy in only one direction, and uses a lower energy setting. The energy setting on a biphasic AED is device specific, but typically ranges from 120 to 200 joules. AEDs are preprogrammed by the manufacturer to deliver a specific amount of energy, which eliminates the need to manually set the energy level. Some biphasic AEDs deliver the same amount of energy for each shock; others may escalate the energy level for each shock. Refer to the manufacturer's documentation that accompanies the AED you use regarding the amount of biphasic energy it delivers.

The pain associated with acute aortic dissection: A) typically comes on gradually and progressively worsens. B) is typically described as a stabbing or tearing sensation. C) is usually preceded by nausea, sweating, and weakness. D) originates in the epigastrium and radiates down both legs.

B) is typically described as a stabbing or tearing sensation. Aortic dissection occurs when the inner layers of the aorta become separated, allowing blood to flow between the layers at high pressure. This separation of layers significantly weakens the aortic wall, making it prone to rupture. Signs and symptoms of acute aortic dissection include a sudden onset of a ripping, tearing, or stabbing pain in the anterior part of the chest or in the back between the scapulae. It may be difficult to differentiate the pain of acute aortic dissection from that of an acute myocardial infarction (AMI), but a number of distinctive features may help. The pain from an AMI is often preceded by other signs and symptoms (ie, nausea, indigestion, weakness, sweating [diaphoresis]). It tends to come on gradually and becomes more severe as time progresses, and is usually described as a crushing pain or as a feeling of heaviness or pressure. By contrast, the pain associated with aortic dissection is acute and is often of maximum intensity from the onset; it is typically described as a ripping, tearing, or stabbing sensation.

A 42-year-old man was ejected from his car after it struck a bridge pillar at a high rate of speed. You find him in a prone position approximately 50 feet from his car. He is not moving and does not appear to be breathing. You should: A) assess his breathing effort. B) manually stabilize his head. C) administer high-flow oxygen. D) use the jaw-thrust maneuver.

B) manually stabilize his head. When a trauma patient is found in a prone (face-down) position, especially if he or she is unresponsive, your first action should be to manually stabilize his or her head; this action is based on the assumption that the patient has a spinal injury. Next, log roll the patient to a supine position (while continuing to manually stabilize the head), open the airway with the jaw-thrust maneuver, clear the airway with suction if needed, and assess for breathing. It would be extremely difficult to adequately open the patient's airway while he or she is in a prone position. Depending on the patient's breathing effort, administer high-flow oxygen or ventilate using a bag-mask device.

You are called to a local park for an ill person. It is a hot day and the humidity is high. When you arrive, a bystander directs you to the patient, a young man who is semiconscious. His skin is flushed, hot, and moist. Your FIRST action in the management of this patient should be to: A) ensure an open airway. B) move him to a cool area. C) administer high-flow oxygen. D) begin rapid cooling measures.

B) move him to a cool area. Your first action in a heat-related emergency is to move the patient to a cooler environment. Once you have moved the patient to a cooler place, you should begin your assessment and treat the patient accordingly. Remember, you must FIRST prevent further harm to the patient. The patient in this scenario has signs of heatstroke. After ensuring airway patency, adequate breathing, and adequate circulation, you should proceed with rapid cooling measures.

Prior to administering nitroglycerin to a patient with chest pain, you should: A) elevate the patient's lower extremities. B) obtain vital signs to detect hypotension. C) inquire about an allergy to salicylates. D) auscultate the patient's breath sounds.

B) obtain vital signs to detect hypotension. Prior to assisting a patient with his or her prescribed nitroglycerin, there are two things that you must do: take the patient's vital signs and obtain authorization from medical control. Nitroglycerin is contraindicated for patients with a systolic blood pressure that is less than 100 mm Hg. If the patient develops hypotension after being given nitroglycerin, position him or her supine and transport without delay. Salicylates are a class of drugs that include aspirin, not nitroglycerin (nitroglycerin is a nitrate). Although you should inquire about medication allergies in general, it is not necessary to inquire specifically about an allergy to salicylates unless you are going to administer aspirin. Assessment of a patient with a possible cardiac or respiratory problem should include auscultation of breath sounds; however, this does not necessarily have to be done before assisting the patient with his or her nitroglycerin.

A 44-year-old man experienced burns to his anterior trunk and both arms. He is conscious and alert, but is in extreme pain. Assessment of the burns reveals reddening and blisters. This patient has ________________ burns that cover _____ of his total body surface area. A) first-degree, 27% B) partial-thickness, 36% C) second-degree, 45% D) full-thickness, 18%

B) partial-thickness, 36% Partial-thickness (second-degree) burns damage the epidermis and part of the dermis, and are characterized by blistering and severe pain. Areas of superficial (first-degree) burns, which cause reddening of the skin, commonly surround a partial-thickness burn. The anterior trunk (chest and abdomen) accounts for 18% of the total body surface area (TBSA) and each entire arm accounts for 9%. Therefore, this patient has partial-thickness burns that cover 36% of his TBSA. Full-thickness (third-degree) burns are characterized by charred or white, leathery skin. Because the entire dermis, including the nerves, is destroyed, full-thickness burns are usually painless. The surrounding areas of partial-thickness burns, however, are very painful.

A 5-year-old boy was struck by a car and is found lying 15 feet away. Based on the child's age and mechanism of injury, you should suspect that the PRIMARY injury occurred to the: A) head. B) pelvis. C) lower legs. D) upper thorax.

B) pelvis. Children are smaller than adults; therefore, when they are injured by the same mechanism as an adult, the location of their injuries may differ from those of an adult. For example, when an adult is struck by a vehicle, the primary injury typically occurs at or below the knees, depending on the height of the bumper at the time of impact. Because the child is shorter, initial impact typically occurs at or near the pelvis. Secondary injury occurs when child's chest collides with the vehicle's grille. Tertiary injury occurs when the child strikes his or her head on the pavement after being propelled away from the vehicle. In some cases, the child is pulled underneath the vehicle and is dragged.

When treating a patient with chest pain, pressure, or discomfort, you should FIRST: A) administer supplemental oxygen B) place the patient in a position of comfort. C) request an ALS ambulance response to the scene. D) assess the blood pressure and give nitroglycerin.

B) place the patient in a position of comfort. An important aspect of treating a patient with chest pain, pressure, or discomfort is to ensure that the patient is in a comfortable position. Most of the time, the patient will already be in this position upon your arrival. A comfortable position will help minimize anxiety, which in turn decreases cardiac oxygen consumption and demand. The decision to administer supplemental oxygen depends on the patient's oxygen saturation and/or whether or not respiratory distress is present. Following your assessment, if you feel that ALS support is needed, you should request it. If the patient has prescribed, unexpired nitroglycerin; the systolic blood pressure is greater than 100 mm Hg; and the patient has not taken the maximum of three doses, you should contact medical control to obtain permission to assist the patient in taking the nitroglycerin.

A 50-year-old man's implanted defibrillator has fired twice within the last hour. He is conscious and alert and reports a "sore chest." Further assessment reveals that his chest pain is reproducible to palpation and is localized to the area of his implanted defibrillator. Treatment for him should include: A) application of the AED and transport to the hospital. B) prompt transport with continuous monitoring en route. C) deactivating his defibrillator by running a magnet over it. D) up to three doses of nitroglycerin and prompt transport.

B) prompt transport with continuous monitoring en route. Palients who are at high risk for lethal cardiac dysrhythmias (ie, VF, VT) may have an automated implantable cardioverter/defibrillator (AICD). This small device is usually implanted in the upper left chest.just below the left clavicle. The AJCD detects cardiac dysrhythmias and rapidly delivers a shock. When treating a patient whose AJCD has fired, you should determine the number of limes the device fired, assess vital signs, and transport to the hospital with continuous monitoring en route. Administer supplemental oxygen if the patient is experiencing respiratory distress and/or the oxygen saturation is less than 94%. Application of the AED is not indicated; however, if the patient develops cardiac arrest, you should use the AED as you normally would (remember to apply the pads at least 1 inch away from the implanted device). The pain that the patient is experiencing, which is reproducible and localized near his AICD, is likely musculoskeletal pain as the result of his AICD shocking him; therefore, nitroglycerin is not indicated. Because the AICD works so quickly (much faster than you can apply an AED), you should not make any attempt to deactivate it.

A 50-year-old woman with a history of epilepsy is actively seizing. Care for this patient should focus primarily on: A) frequently suctioning her airway and carefully restraining her. B) protecting her from injury and ensuring adequate ventilation. C) administering high-flow oxygen and requesting an ALS ambulance. D) placing a bite block in between her molars and giving her oxygen.

B) protecting her from injury and ensuring adequate ventilation. Seizure deaths are most frequently the result of hypoxia. When a person is actively seizing, he or she is not breathing adequately. Your primary focus when treating a seizure patient is to protect him or her from injury and to ensure adequate ventilation and oxygenation. Many seizing patients require assisted ventilation. Suction the oropharynx only if the patient has secretions in his or her mouth. Do NOT insert anything into the mouth of a seizing patient; doing so may cause an airway obstruction or damage the soft tissues of the mouth, resulting in bleeding. Do not attempt to restrain an actively seizing patient; doing so may result in musculoskeletal injuries. Request an ALS ambulance per your local protocols.

A 78-year-old woman reports pain to her right groin area after she fell. Her right leg is straight but is externally rotated and shorter than the left. The EMT should suspect a: A) pelvic girdle fracture. B) proximal femur fracture. C) posterior hip dislocation. D) symphysis pubis fracture.

B) proximal femur fracture. Fractures of the proximal (upper) end of the femur are common fractures, especially in older patients and patients with osteoporosis. Although these fractures are often called hip fractures, they rarely involve the hip joint. Instead, the fracture occurs at the neck of the femur. Patient with displaced fractures of the proximal femur present with a very characteristic deformity. The leg is straight but is externally rotated and shorter than the uninjured leg. Pain is typically located in the hip region or in the groin or inner aspect of the thigh. Posterior dislocation of the hip most commonly occurs as a result of a motor vehicle crash in which the knee meets with a direct force, such as the dashboard, and the entire femur is driven posteriorly, dislocating the hip joint. Patients with a posterior hip dislocation typically lie with the hip joint flexed (the knee joint drawn up toward the chest) and the thigh rotated internally. Pelvic fractures, including those of the symphysis pubis, typically do not cause shortening or lengthening of an extremity, nor are they typically associated with internal or external rotation of the legs. Fracture of the symphysis pubis is characterized by palpable pain over the pubic bone.

A patient presents with a swollen, painful deformity to the lateral bone of the left forearm. You should recognize that he has injured his: A) ulna. B) radius. C) clavicle. D) humerus.

B) radius. Recalling the body in the anatomic position, the radius is the lateral (thumb side) bone of the forearm and the ulna is the medial (pinky side) bone. The humerus is the long bone of the arm and the clavicle is the collarbone, which extends from the sternum laterally to the shoulder.

A 22-year-old man had a strong acid chemical splashed into both of his eyes. He is conscious and alert, is experiencing intense pain, and states that he is wearing contact lenses. Treatment should include: A) leaving the contact lenses in and beginning irrigation of both eyes. B) removing the contact lenses and beginning irrigation of both eyes. C) leaving the contact lenses in and covering both eyes with sterile gauze. D) removing the contact lenses and covering both eyes with sterile gauze.

B) removing the contact lenses and beginning irrigation of both eyes. As a general rule, contact lenses should be left in place. Chemical eye burns are an exception to this rule. If left in place, the chemical could get behind the contact lens and continue to cause injury. Therefore, you should remove the contact lenses and immediately irrigate the eyes with sterile saline or water. If needed, continue to irrigate the eyes throughout transport.

The AED analyzes your pulseless and apneic patient's cardiac rhythm and advises that a shock is NOT indicated. You should: A) assess for a pulse for no more than 10 seconds. B) resume CPR, starting with chest compressions. C) open the patient's airway and check for breathing. D) reanalyze the cardiac rhythm for positive confirmation.

B) resume CPR, starting with chest compressions. If the AED advises "no shock;' you should immediately resume CPR, starting with chest compressions. Unless the patient starts to move or has other signs of life (ie, coughing), stopping CPR to assess for a pulse should be avoided because it just causes an unnecessary delay in performing chest compressions. The AED is a highly reliable device; if it advises that a shock is or is not indicated, take its word for it! There is no need to reanalyze the patient's cardiac rhythm; this merely causes an unnecessary delay in defibrillation (if indicated) and chest compressions.

Active rewarming of a patient with moderate or severe hypothermia should be avoided in the field because: A) the risk of inadvertently inducing hyperthermia is too high. B) rewarming too quickly can cause a fatal cardiac dysrhythmia. C) it is painful for the patient and you cannot give analgesic drugs. D) active rewarming has been shown to cause severe hypertension.

B) rewarming too quickly can cause a fatal cardiac dysrhythmia. When caring for a patient with hypothermia, your goal is to prevent further heat loss; this involves removing wet clothing, applying warm blankets, and allowing the patient's body temperature to rise gradually and naturally (passive rewarming). If the patient is moderately or severely hypothermic, you should not try to rewarm him or her actively (placing heat on or into the body). Rewarming too quickly may cause a fatal cardiac dysrhythmia, such as ventricular fibrillation (VF). Active rewarming may also cause rewarming shock, a condition in which the blood vessels dilate when heat is applied to the body, resulting in significant hypotension. For these reasons, active rewarming should be performed only in the controlled setting of a hospital.

When applying the ECG electrodes, the negative (white) lead is placed on the: A) left arm. B) right arm. C) left leg. D) right leg.

B) right arm. Proper lead placement is paramount in obtaining an accurate electrocardiographic tracing of the heart. Misplaced leads can produce erroneous ECG findings, or hide an abnormality that correct lead placement would otherwi e have revealed. The negative (white) lead is placed on the right arm, the ground (black) lead is placed on the left arm, the positive (red) lead is placed on the left leg, and the green lead (completes the circuit) is placed on the right leg.

In most people, the inferior aspect of the left ventricle receives its blood supply from the: A) circumflex artery. B) right coronary artery. C) left main coronary artery. D) left anterior descending artery.

B) right coronary artery. The right coronary artery (RCA} supplies blood to the entire right side of the heart; in most people, it also supplies blood to the inferior aspect of the left ventricle in most people. The left main coronary artery branches into the left anterior descending (LAD} and circumflex arteries. The LAD artery supplies the ventricular septum and the anterior aspect of the left ventricle. The circumflex artery supplies the lateral and posterior aspects of the left ventricle.

Common signs and symptoms of a hypertensive emergency include: A) unequal pupils, irregular pulse, and pallor B) ringing in the ears, headache, and epistaxis. C) chest discomfort, weak pulses, and cool skin. D) vomiting without nausea and hemiparesis.

B) ringing in the ears, headache, and epistaxis. Although different sources cite various values, most agree that a hypertensive emergency exists when the systolic blood pressure exceeds 180 mm Hg and the diastolic blood pressure exceeds 110 mm Hg. A hypertensive emergency also exists when the systolic BP suddenly rises and produces signs and symptoms. One of the most common symptoms of a hypertensive emergency is a severe heada he. Other signs and symptoms include ringing in the ears (tinnitus), epistaxis (nosebleed), bounding pulses, flushed skin (dry or moist), nausea and vomiting, and dizziness. In severe cases, mental status changes may occur and the patient may experience a sudden onset of pulmonary edema. Untreated hypertension can cause a stroke, heart failure, or aortic dissection, to name a few. By itself, a hypertensive emergency does not usual ly cause unequal pupils or weakness to one side of the body (hemiparesis); if these signs are present in a hypertensive patient, the EMT should suspect a stroke.

The wall that separates the left and right sides of the heart is called the: A) carina. B) septum. C) pericardium. D) mediastinum.

B) septum. The septum is the wall that separates the left and right sides of the heart. There is a septum for both the atria and the ventricles. The carina is the bifurcation point of the trachea, and the mediastinum is the space between the lungs in which the heart, great vessels, and a portion of the esophagus lie. The pericardium is the sac that surrounds the heart and contains pericardia! fluid.

A middle-aged woman took three of her prescribed nitroglycerin tablets after she began experiencing chest pain. She complains of a bad headache and is still experiencing chest pain. You should assume that: A) her blood pressure is elevated. B) she has ongoing cardiac ischemia. C) her nitroglycerin is no longer potent. D) her chest pain is not cardiac related.

B) she has ongoing cardiac ischemia. A headache and/or a bitter taste under the tongue are common side effects of nitroglycerin ( TG) that many patients experience. If the patient does not experience these side effects, the NTG may have lost its potency. However, if a patient with chest pain takes NTG and experiences these side effects, but still has chest pain, you should assume that his or her pain is the resu lt of cardiac ischemia, a relative deprivation of oxygen to the heart. NTG is a vasodilator drug; therefore, it would lower her blood pressure, not raise it. Any patient with nontraumatic chest pain or pressure should be assumed to be experiencing cardiac ischemia, especially if the pain or pressure is not rel ieved with NTG.

Switching compressors during two-rescuer CPR: A) should take no more than 15 seconds to accomplish. B) should occur every 2 minutes throughout the arrest. C) is necessary only if the compressor becomes fatigued. D) is performed after every 10 to 20 cycles of adult CPR.

B) should occur every 2 minutes throughout the arrest. Rescuer fatigue leads to inadequate chest compression rate and/or depth. Fatigue is common after 1 minute of CPR, although the rescuer may not recognize it for 5 minutes or longer. Therefore, compressor should be changed every 2 minutes (after 5 cycles of CPR at a 30:2 ratio) throughout the resuscitation attempt. If the compressor is not switched until he or she recognizes the fatigue, the patient has likely been without effective chest compressions for at least 4 or 5 minutes. In general, interruptions in CPR should be infrequent and should not exceed 10 seconds. However, every effort should be made to switch compressors in less than 5 seconds.

A 46-year-old man presents with generalized weakness and shortness of breath after he was bitten on the leg by a rattlesnake. His blood pressure is 106/58 mm Hg and his pulse rate is 112 beats/min. Treatment for this patient should include: A) elevation of the affected part and ice packs. B) splinting and lowering of the affected part. C) ice packs to the wound and splinting. D) proximal arterial constricting band and splinting.

B) splinting and lowering of the affected part. Care for a patient with a bite from a pit viper (rattlesnake, copperhead, water moccasin) includes keeping the patient calm, administering supplemental oxygen, splinting the affected part, and keeping it below the level of the heart. Do NOT apply ice to a snakebite; this will cause local vasoconstriction and may force the venom deeper into the patient's circulation. If a constricting band is applied, it should be proximal to the bite and should be tight enough to slow venous return only, not cut off arterial supply.

You are dispatched to the scene of a motorcycle crash. Upon arrival, you find the patient lying face down approximately 25 feet from his bike. He is not wearing a helmet and is moaning. You should: A) apply a cervical collar. B) stabilize his head manually. C) log roll him into a supine position. D) evaluate the status of his airway.

B) stabilize his head manually. The mechanism of injury for this patient was significant. In his present position (prone), you cannot effectively assess his airway. Therefore, your first action should be to manually stabilize his head. Then, you must log roll him into a supine position, keeping his head in an in-line position. If possible, log roll him directly onto a long backboard. After the patient is supine, assess the status of his airway, assess his breathing adequacy, administer high-flow oxygen or begin assisted ventilations if needed, and continue with your primary assessment. Apply a cervical collar as soon as possible, but assess his posterior neck first.

A 70-year-old woman fell and struck her head two days ago, but did not seek medical attention. Today, she is confused, is vomiting, and has slurred speech. The EMT should suspect a/an: A) epidural hematoma. B) subdural hematoma. C) intracerebral hematoma. D) acute ischemic stroke.

B) subdural hematoma. This case is classic for a subdural hematoma. A subdural hematoma occurs when bleeding occurs between the dura mater (the outer meningeal layer) and the surface of the brain; it is typically caused by venous bleeding. As such, subdural hematomas often do not present with symptoms until several hours, or even days, have past since a head injury. By contrast, epidural hematoma, bleeding between the skull and dura mater, is usually caused by arterial bleeding; patients with this type of injury usually present with symptoms immediately following the injury. lntracerebral hemorrhage, bleeding within the brain itself, would also be expected to produce immediate symptoms. While acute ischemic stroke could also explain this patient's symptoms, the fact that she recently experienced a head injury makes the diagnosis of a subdural hematoma more likely.

A man was stabbed in the cheek with a dinner fork, and the fork is still impaled in his cheek. He is conscious and alert, breathing adequately, and has blood in his oropharynx. You should: A) apply high-flow oxygen via a nonrebreathing mask, carefully remove the fork, and control any external bleeding. B) suction his oropharynx, control any external bleeding, stabilize the fork in place, and protect it with bulky dressings. C) carefully remove the fork, suction his oropharynx as needed, and pack the inside of his cheek with sterile gauze pads. D) suction his oropharynx, carefully cut the fork to make it shorter, control any external bleeding, and secure the fork in place.

B) suction his oropharynx, control any external bleeding, stabilize the fork in place, and protect it with bulky dressings. An impaled object in the cheek should be removed if it interferes with your ability to manage the patient's airway. In this case, however, the patient is breathing adequately and does not require aggressive airway care (eg, ventilatory assistance). The most practical approach is to suction the blood from his oropharynx, which will prevent him from swallowing it, vomiting it, or aspirating it. Stabilize the fork in place and protect it with bulky dressings; removing an impaled object from the cheek in the opposite direction it entered would clearly cause further soft-tissue injury and bleeding. Transport the patient in a sitting position and suction his oropharynx en route as needed. There is no reason to cut the fork to make it shorter; this will only unnecessarily manipulate it, potentially causing further soft tissue damage and increased bleeding.

Treatment for a responsive 4-year-old child with a mild airway obstruction, who has respiratory distress, a strong cough, and normal skin color, includes: A) oxygen, back slaps, and transport. B) supplemental oxygen and transport. C) assisted ventilations, back slaps, and transport. D) subdiaphragmatic thrusts until the object is expelled.

B) supplemental oxygen and transport. If a child (1 year of age to the onset of puberty [12 to 14 years of age]) with a mild airway obstruction is alert and has adequate air movement (ie, a strong cough, normal skin color), you should offer oxygen, avoid agitating the child, and provide transport to the hospital. Attempts to relieve a mild airway obstruction may result in a severe airway obstruction. If signs of a severe airway obstruction develop, you must take immediate action to remove the object (eg, back slaps and chest thrusts in a responsive infant; subdiaphragmatic [abdominal] thrusts in a responsive adult or child). Finger sweeps are indicated ONLY if the patient is unresponsive and you can see the object in his or her mouth.

You will know that the third stage of labor has begun when: A) the placenta has delivered. B) the entire baby has delivered. C) the mother's contractions become regular. D) the baby's head is visible at the vaginal opening.

B) the entire baby has delivered. Labor is divided into three stages. The first stage begins with the onset of contractions and ends when the cervix is fully dilated. In the field, the EMT cannot determine the degree of cervical dilation, so the appearance of the baby's head at the vaginal opening (crowning) is used to mark the end of the first stage of labor. As the first stage of labor progresses, the mother's contractions become more frequent and regular. The second stage of labor begins with full cervical dilation (or in the field, crowning) and ends when the baby is completely delivered. The third stage of labor begins with the birth of the baby and ends when the placenta (afterbirth) has delivered.

When assessing a patient who has stroke-like symptoms, you should recall that: A) the majority strokes are caused by a ruptured cerebral artery. B) the patient may be unable to communicate, but can often understand. C) right-sided weakness indicates a stroke in the right cerebral hemisphere. D) fibrinolytic therapy must be given within 6 hours following the stroke.

B) the patient may be unable to communicate, but can often understand. Some patients who have had a stroke may be unable to communicate (expressive aphasia), but they can often understand what is being said around them; be aware of this possibility. According to the American Heart Association, 87% of all strokes are caused by an occluded cerebral artery (ischemic stroke); strokes caused by a ruptured cerebral artery {hemorrhagic stroke) are less common. Because the left side of the brain controls the right side of the body, and vice versa, right-sided weakness {hemiparesis) indicates a stroke in the left cerebral hemisphere. Some patients who have had a stroke may benefit from fibrinolytic (clot-buster) therapy; however, to be most effective, this treatment must be given within the first 3 hours (4.5 hours in certain patients) following the onset of stroke symptoms.

When assessing a conscious patient who overdosed on a drug, you should FIRST determine: A) the patient's weight in kilograms. B) the type of medication ingested. C) when the medication was ingested. D) if there is a history of prior overdose.

B) the type of medication ingested. When assessing a patient who has overdosed on a drug, you should first determine what was ingested, which will provide you with immediate information about whether the substance is toxic. You should then find out when the medication was ingested. This information will provide medical control (or poison control) with the information needed to direct the most appropriate treatment. The patient's weight also should be estimated in kilograms in the event that an antidote is required. Information regarding prior drug overdoses would normally be obtained during the SAMPLE history; it is not the most critical information during acute resuscitation.

lschemic heart disease is a condition in which: A) a portion of the heart muscle dies because of a prolonged lack of oxygen B) there is a decrease in blood flow to one or more portions of the heart muscle. C) the coronary arteries dilate, thus preventing effective blood flow to the heart. D) an acute event leads to a significant decrease in the pumping force of the heart.

B) there is a decrease in blood flow to one or more portions of the heart muscle. Chest pain or discomfort that is related to the heart usually stems from a condition called ischemia (insufficient oxygen). Because of a partial or complete blockage of blood flow through one or more coronary arteries, the tissue of the heart muscle (myocardium) fails to get enough oxygen and nutrients relative to its needs. Therefore, ischemic heart disease is a condition involving a decrease in blood flow, and therefore oxygen, to one or more portions of the myocardium. If blood flow to the ischemic portion of the myocardium is not restored, it eventually dies (myocardial infarction). Dilation of the coronary arteries increases, not decreases, blood flow to the heart. If an event such as a myocardial infarction damages the heart and significantly decreases its ability to contract forcefully, heart failure may occur.

A 39-year-old man asks you to take him to the hospital because has had a fever, headache, and diarrhea for the past 2 days. His blood pressure is 120/60 mm Hg, his pulse is 110 beats/min, and his respirations are 16 breaths/min. You should: A) ask him if he has a history of HIV infection or hepatitis. B) transport him to the hospital in a position of comfort. C) request an ALS ambulance to the scene to start an IV line. D) advise him that he can drive himself to his family physician.

B) transport him to the hospital in a position of comfort. Although the patient is likely experiencing the flu, there are other diseases, some of which are communicable, that can cause similar symptoms. The patient is requesting EMS transport; failure to comply constitutes abandonment. Although he is tachycardic, the remainder of his vital signs are stable; therefore, requesting an ALS ambulance to the scene to start an IV is not necessary. Simply transport him in a position of comfort and monitor him en route. If the patient is infected with HIV or hepatitis, he may choose to voluntarily disclose that information. However, to inquire about infection with such diseases is unethical.

A 45-year-old woman calls EMS because of severe chest pain. When you arrive, she advises you that she has taken two of her husband's nitroglycerin tablets without relief. Her BP is 110/60 mm Hg, her pulse is 100 beats/min, her respirations are 16 breaths/min, and her oxygen saturation is 95%. You should: A) attach the AED, administer 100% oxygen, and contact medical control for advice. B) transport the patient without delay and monitor her blood pressure en route. C) Call medical control and request permission to assist the patient with one more NTG tablet. D) Apply oxygen, assess the patient's blood pressure, and give a third and final NTG tablet.

B) transport the patient without delay and monitor her blood pressure en route. Do not administer, or assist a patient with, a medication that is not prescribed to him or her. In this case, the best course of action is to transport the patient without delay and monitor her blood pressure en route. Additionally, if she has no drug allergies, you should administer up to 325 mg of chewable aspirin. Oxygen is not indicated for this patient; there is no evidence of respiratory compromise and her oxygen saturation is greater than 94%. The AED is applied only to patients who are in cardiac arrest; the patient in this scenario is not in cardiac arrest.

During your assessment of a 70-year-old man with crushing chest pain, you note that his blood pressure is 80/50 mm Hg. Your MOST important action should be to: A) give high-flow oxygen. B) transport without delay. C) keep the patient warm. D) assess his oxygen saturation.

B) transport without delay. Patients with chest pain, pressure, or discomfort with a systolic BP less than 100 mm Hg should be transported to the hospital without delay. Hypotension in a patient with chest pain indicates cardiogenic shock due to severe cardiac damage and requires treatment that can only be given at the hospital. Any delay in transport delays definitive care and increases the patient's chance of death. High-flow oxygen, thermal management, and assessment of oxygen saturation are clearly important; however, your primary focus should be to get the patient to the hospital as soon as possible.

An unrestrained driver was crushed in between the steering wheel and his seat when his truck collided with a tree. Assessment reveals cyanosis to his neck and face, jugular venous distention, and bleeding into the sclera of his eyes. The EMT should suspect: A) aortic dissection. B) traumatic asphyxia. C) massive hemothorax. D) pericardia! tamponade.

B) traumatic asphyxia. Rather classic signs of traumatic asphyxia are present in this patient. Traumatic asphyxia occurs when a sudden compressive force is applied to the chest, which results in a sudden massive amount of blood being shunted to the neck, face, and head. Signs include jugular venous distention (JVD); cyanosis to the neck, face, and head; bulging eyes; and scleral hemorrhage (blood in the whites of the eyes). Traumatic asphyxia has a high mortality rate, mainly because of the massive injuries that occur within the thoracic cavity. Aortic dissection and massive hemothorax do not present with ND because both of these injuries are associated with massive blood loss; one would expect collapsed jugular veins. Pericardia! tamponade can cause JVD; however, cyanosis to the upper body and scleral hemorrhage are not observed.

You are called to an assisted living center where an attendant found a 72-year-old man unresponsive. The patient had recent hip surgery and has been taking Vicodin for pain. His respirations are slow and shallow, and his pulse is slow and weak. You should: A) apply high-flow oxygen via a nonrebreathing mask. B) ventilate the patient with a bag-mask device. C) administer intranasal naloxone at 0.4 mg. D) apply the AED and analyze his cardiac rhythm.

B) ventilate the patient with a bag-mask device. Vicodin is a combination of hydrocodone and acetaminophen (APAP}, the active ingredient in Tylenol. Hydrocodone is a narcotic analgesic. When taken in excess, it can suppress the central nervous system and cause respiratory depression, bradycardia, and hypotension. Initial management of any patient who has overdosed on a medication of this type is to ensure a patent airway and support breathing. Because the patient is breathing inadequately (slow and shallow), immediate ventilation assistance is needed. If you carry naloxone (Narcan) and are authorized to administer it, do so. Naloxone binds to opiate receptor sites and reverses the effects of narcotic drugs. If you are not authorized to administer naloxone, consider requesting a paramedic intercept, especially if your transport time will be lengthy. The AED is not indicated for this patient; it is applied only to patients in cardiac arrest.

Assessment of an injured man reveals that he opens his eyes when the EMT speaks to him and pulls his arm away when the EMT palpates it. He knows his name, but cannot remember what happened and does not know what day it is. He should be assigned a Glasgow Coma Scale (GCS) score of: A) 9 B) lO C) 11 D) 12

C) 11 The patient's GCS is calculated at 11. He opens his eyes in response to verbal stimuli, which equates to a score of 3 for eye opening. Although he knows his name, he is confused in that he cannot remember what happened or what day it is; this equates to a score of 4 for verbal response. Because the patient withdraws from pain, he receives a score of 4 for motor response.

A high normal respiratory rate for an infant between 1 month and 1 year of age is: A) 30 breaths/min. B) 40 breaths/min. C) 60 breaths/min. D) 70 breaths/min.

C) 60 breaths/min. Normal respiratory rates for infants between 1 month and 1 year of age range between 30 to 60 breaths/min. Normal respiratory rates for a toddler between 1 and 3 years of age range between 24 to 40 breaths/min. Normal respiratory rates for preschool-age children between 4 and 5 years of age range between 22 and 34 breaths/min. Normal respiratory rates for school-age children between 6 and 12 years of age range between 18 and 30 breaths/min. These respiratory rate ranges are cited in the 2015 Pediatric Advanced Life Support Guidelines from the American Heart Association.

Which of the following patients would MOST likely present with vague or unusual symptoms of an acute myocardial infarction? A) 55-year-old obese woman B) 66-year-old man with angina C) 72-year-old woman with diabetes D) 75-year-old man with hypertension

C) 72-year-old woman with diabetes Not all patients experiencing acute myocardial infarction (AMI) present with the classic signs and symptoms one would expect. Middle-aged men often minim ize their symptoms and attribute their chest pain or discomfort to indigestion. Some patients, however, do not experience any pain. In particular, elderly women with diabetes may present with vague, unusual, or atypical symptoms of AMI; their only presenting complaint may be fatigue or syncope. Do not rule out a cardiac problem just because a patient is not experiencing chest pain, pressure, or discomfort; this is especially true in elderly females with diabetes.

An adult patient opens his eyes in response to a painful stimulus, moans when you ask him questions, and pulls his arm away when you palpate it. What is his Glasgow Coma Scale (GCS) score? A) 6 B) 7 C) 8 D) 9

C) 8

A newborn has a heart rate of 130 beats/min, cyanosis of the hands and feet, and rapid respirations. The infant cries when you flick the soles of its feet and resists attempts to straighten its legs. You should assign an APGAR score of: A) 7 B) 8 C)9 D) 10

C) 9 The APGAR score, which is obtained at 1 and 5 minutes after birth (and every 5 minutes thereafter), assigns numbers (0, 1, or 2) to the following five areas: appearance, pulse, grimace, activity, and respirations. A score of 1 is assigned for appearance if the newborn's body is pink, but its hands and feet remain blue. If its heart rate is greater than 100 beats/min, it receives a score of 2 for the pulse. If it cries and tries to move its foot away when soles of its feet are flicked, it is assigned a score of 2 for grimace/irritability. If it resists attempts to straighten its hips and knees, a score of 2 is assigned for activity/muscle tone. If its respirations are rapid, a score of 2 is assigned. Based on these parameters, the newborn in this scenario would receive an APGAR score of 9.

During a soccer game, a 20-year-old man collided shoulder-to-shoulder with another player. He has pain and a noticeable anterior bulge to the left shoulder. Which of the following is the MOST effective method of immobilizing this injury? A) An air-inflatable splint with the left arm immobilized in the flexed position B) A long board splint with the left arm immobilized in the extended position C) A sling to support the left arm and swathes to secure the arm to the body D) A sling to support the left arm and swathes to maintain downward traction

C) A sling to support the left arm and swathes to secure the arm to the body Injuries to the shoulder are most effectively immobilized with the use of a sling and swathe. The sling will provide support and relieve pain to the shoulder, and the swathe will secure the arm to the body. The purpose of the swathe is not to facilitate traction. Patients with dislocated or fractured shoulders will not allow you to extend their arm, so any attempt to immobilize the injury in such a fashion will not be possible and could worsen the injury.

You are caring for a 6-year-old child with a possible fractured arm and have reason to believe that the child was abused. What should you do? A) Inform the parents of your suspicions. B) Call the police and have the parents arrested. C) Advise the parents that the child needs to be transported. D) Transport the child to the hospital regardless of the parent's wishes.

C) Advise the parents that the child needs to be transported. You must handle cases of suspected child abuse with great care. You must never accuse the parents or caregiver of abuse. If you are wrong, you could be held liable for slander. Actions that would suggest such an accusation include summoning the police to have the parents arrested. Instead, you should advise the parents or caregiver that the child needs to be transported by ambulance, even if the injury is not life-threatening. The goal is to get the child to safety; however, this must be done legally (with parental consent). In most cases, you need the consent of only one parent to transport the child. Once at the hospital, you must apprise the receiving physician of your suspicions.

Which of the following is a physiologic effect of epinephrine when used to treat anaphylactic shock? A) As a vasodilator, it increases the blood pressure. B) As a vasoconstrictor, it lowers the blood pressure. C) As a bronchodilator, it improves the patient's breathing. D) As an antihistamine, it blocks chemicals that cause the reaction.

C) As a bronchodilator, it improves the patient's breathing. Epinephrine possesses dual effects. As a bronchodilator, it relaxes the smooth muscle of the bronchioles and improves the patient's breathing. As a vasoconstrictor, it constricts the blood vessels and increases the patient's blood pressure. Diphenhydramine (Benadryl) is an antihistamine; it blocks H1 histamine receptor sites, which blocks the release of the chemicals (histamines) that are causing the allergic reaction.

When you begin to assess a woman in labor, she states that her contractions are occurring every 4 to 5 minutes and lasting approximately 30 seconds each. Which of the following questions would be MOST appropriate to ask next? A) Has your bag of waters broken yet? B) Have you had regular prenatal care? C) At how many weeks' gestation are you? D) How many other children do you have?

C) At how many weeks' gestation are you? When assessing a patient in labor, the first question you should ask is how far along in the pregnancy she is. If she is less than 39 weeks gestation (a pregnancy is considered full term once it reaches 39 weeks, but has not gone beyond 40 weeks and 6 days), you should prepare for possible resuscitation of the newborn if delivery occurs in the field. Other questions, such as asking if her amniotic sac (bag of waters) has ruptured and whether she has received prenatal care, also can help you anticipate and prepare for potential complications. You should also inquire as to how many times the patient has been pregnant, regardless of whether she carried the baby to term (gravida), and the number of times she has carried a baby beyond 28 weeks, regardless of whether it was born dead or alive (para).

Which of the following sets of vital signs is the MOST consistent with hemorrhagic shock? A) BP, 80/40 mm Hg; pulse, 70 beats/min; respirations, 24 breaths/min B) BP, 190/100 mm Hg; pulse, 50 beats/min; respirations, 8 breaths/min C) BP, 88/50 mm Hg; pulse, 120 beats/min; respirations, 28 breaths/min D) BP, 160/70 mm Hg; pulse, 140 beats/min; respirations, 12 breaths/min

C) BP, 88/50 mm Hg; pulse, 120 beats/min; respirations, 28 breaths/min Of the vital sign values listed, hypotension, tachycardia, and tachypnea are the most consistent with hemorrhagic shock. In fact, the presence of hypotension indicates decompensated shock. Hypotension and a normal (or slow) heart rate is consistent with neurogenic shock. Hypertension, bradycardia, and abnormal breathing (Cushing's triad) are consistent with increased intracranial pressure from a head injury. Hypertension, tachycardia, and normal breathing may be observed in a patient with a hypertensive emergency; they are not consistent with hemorrhagic shock.

A 23-year-old woman with sickle cell disease reports a sudden onset of severe pain in her lower extremities. Which of the following is the MOST likely cause of her symptoms? A) Red blood cells are being destroyed at an abnormal rate. B) Spontaneous bleeding is occurring within the muscles. C) Blood flow is diminished because of vascular occlusion. D) Blood clots have formed because of excessive platelets.

C) Blood flow is diminished because of vascular occlusion. Sickle cell disease is an inherited blood disorder in which the red blood cells are misshapen and take on a sickle appearance. The sharp and misshapen red blood cells lead to dysfunction in oxygen binding and unintentional clot formation. These unintentional clots may lead to a condition called vasoocclusive crisis. The patient's symptoms indicate that this is what has happened in her lower extremities. The blood clots in her lower extremities did not form because of an excessive platelet count (thrombocythemia) because sickle cell disease is not a condition of excessive platelet production. Spontaneous bleeding is not the cause of her symptoms because sickle cell disease is not a condition of low platelets (thrombocytopenia). Sickle cell disease is not a condition of abnormal red blood cell destruction.

Which of the following mechanisms cause respiratory and circulatory collapse during anaphylactic shock? A) Bronchodilation and vasodilation B) Bronchodilation and vasoconstriction C) Bronchoconstriction and vasodilation D) Bronchoconstriction and vasoconstriction

C) Bronchoconstriction and vasodilation During anaphylaxis, histamines released from the immune system cause two negative effects that result in shock (hypoperfusion): vasodilation, which causes the blood pressure to fall , and bronchoconstriction, which impairs breathing.

Which of the following injuries is MOST indicative of child abuse? A) Small laceration to the chin B) Burned hand with splash marks C) Bruising to the upper back D) Multiple bruises to the shins

C) Bruising to the upper back To detect child abuse, you must be familiar with injury locations and patterns consistent with an accident versus those that were intentionally inflicted. It is common for children to trip, fall, and strike their chin or forehead on a solid object; therefore, chin lacerations and hematomas to the forehead are common injuries. Small children frequently hit their legs on coffee tables, resulting in bruises to the shins. If a child accidentally sticks his or her hand in hot water, the hand is quickly pulled back by reflex, resulting in a splash pattern of burns. Injuries found in anatomically unlikely areas, such as the torso (back or front), upper arms and legs, or genitalia, should raise your index of suspicion. Burns that are not accompanied by splash marks should also make you suspicious. For example, if a child's hand or foot is intentionally held in hot water, you will see a clear line of demarcation (stocking-glove effect) without evidence of splash burns.

In which of the following situations would the EMT MOST likely deliver a baby at the scene? A) Contractions are 8 to 10 minutes apart and irregular. B) The hospital is 15 miles away and crowning is not present. C) Contractions are 3 minutes apart and mother feels the urge to push. D) The amniotic sac has ruptured and contractions occur regularly.

C) Contractions are 3 minutes apart and mother feels the urge to push. The EMT should prepare for delivery at the scene when delivery is imminent or can be expected within a few minutes (eg, the mother feels the urge to push, contractions are 2 to 3 minutes apart and regular, crowning is present); when a natural disaster (eg, flood, fire, tornado) has occurred and your route to the hospital is blocked; and when transportation is not available (eg, mechanical malfunction of the ambulance). These are just some of the factors to consider when making that critical decision. Rupture of the amniotic sac and the onset of contractions signals the beginning of the first stage of labor; delivery is usually not imminent at this point and it is generally safe to transport.

A man was stabbed to the left side of the chest. His skin is cool and clammy, his blood pressure is 90/60 mm Hg, his respirations are 22 breaths/min, and his pulse is 120 beats/min and weak. His breath sounds are equal bilaterally and his jugular veins are distended. What should you do? A) Cover the wound and position him on his left side. B) Ventilate the patient, apply an AED, and transport. C) Cover the wound, administer oxygen, and transport. D) Suspect a pneumothorax and contact medical control.

C) Cover the wound, administer oxygen, and transport. The mechanism of injury and clinical presentation indicate a pericardia! tamponade. Pericardia! tamponade occurs when blood fills the pericardia! sac and restricts the heart from relaxing. If the heart cannot relax, it cannot fill with blood. If the right side of the heart cannot fill with blood, there is nothing for it to send to the left side of the heart; as a result, the patient's cardiac output drops and shock develops. Signs of pericardia! tamponade include muffled or distant heart tones (difficult to assess in the field); a rapid, weak pulse; hypotension; jugular venous distention; and a narrowing pulse pressure (difference between the systolic and diastolic blood pressures). A pneumothorax is unlikely in this patient; his breath sounds are equal bilaterally. Pericardia! tamponade is a life-threatening emergency that requires prompt treatment. For this patient, you should cover the wound on his chest, administer oxygen, and transport without delay. Shock patients should be transported supine, not on their side. The AED is not indicated for this patient because he is not in cardiac arrest.

Which of the following would be the MOST significant finding in a 2-month-old infant who is dehydrated? A) Dry mucous membranes B) Absence of tear production C) Delayed capillary refill time D) Heart rate of 130 beats/min

C) Delayed capillary refill time Infants and small children can become significantly dehydrated following even a few episodes of vomiting and/or diarrhea. All of the clinical findings listed are consistent with dehydration. Delayed capillary refill time, however, indicates decreased perfusion; therefore, it is the most significant finding of the options listed. Tachycardia is expected in any patient who is dehydrated; 130 beats/min is not excessively fast for a 2-month-old infant.

Which of the following assessment findings is the MOST concerning in a patient with significant burns? A) Severe blisters to both hands B) Closed deformity of the wrist C) Dry cough and a hoarse voice D) Clothes adhered to burned skin

C) Dry cough and a hoarse voice Any condition or injury that involves airway, breathing, or circulation warrants the EMT's most immediate attention. A dry cough and hoarse voice are signs of inhalation injury and airway swelling. The EMT must carefully monitor the patient and be prepared to ventilate him if his breathing becomes inadequate. Transport the patient without delay; if tactically feasible, a paramedic intercept should be requested. If the patient's airway completely closes, more invasive airway management will be needed (ie, cricothyrotomy). Tend to the other injuries listed during transport to the hospital.

With regard to the electrocardiogram (ECG), what is artifact? A) significant abnormality discovered by the paramedic B) The complete absence of electrical activity on the ECG C) Electrical interference that can make diagnosis difficult D) Improper lead placement, resulting in misdiagnosis

C) Electrical interference that can make diagnosis difficult Artifact refers to an ECG tracing that is the result of interference, such as patient movement, deep breathing, or muscle tremors, rather than the heart's electrical activity. Artifact can make diagnosis of the ECG difficu lt or impossible. To minimize artifact on the ECG, make sure the patient is lying supine; if the patient is experiencing respiratory distress, place him or her in a semi-Fowler's position. Ensure that the patient's arms are relaxed by his or her side and his or her feet are uncrossed.

A 40-year-old man was hit in the nose during a fight. He has bruising under his left eye and a nosebleed. What should you do? A) Place a chemical ice pack over his nose. B) Determine if he has any visual disturbances. C) Ensure that he is sitting up and leaning forward. D) Apply direct pressure by pinching his nostrils together.

C) Ensure that he is sitting up and leaning forward. During a nosebleed (epistaxis), much of the blood may pass down the throat into the stomach as the patient swallows; this is especially true if the patient is lying supine. Blood is a gastric irritant; a person who swallows a large amount of blood may become nauseated and vomit, which increases the risk of aspiration. Therefore, your first action should be to ensure that the patient is sitting up and leaning forward. This will prevent blood from draining down the back of the throat. Next, apply direct pressure by pinching the fleshy part of the nostrils together; you or the patient may do this. Placing a chemical ice pack over the nose may further help control the bleeding by constricting the nasal vasculature. After controlling the nosebleed, continue your assessment, which includes assessing for facial deformities and visual disturbances.

Which of the following is a sign of inadequate breathing in an infant? A) Sunken fontanelles B) Abdominal breathing C) Expiratory grunting D) Heart rate of 130 beats/min

C) Expiratory grunting Expiratory grunting in an infant or a child with a respiratory problem is an ominous sign; it indicates impending respiratory arrest. Grunting represents the child's attempt to maintain oxygen reserve in the lungs. Sunken fontanelles, the soft spots on the infant's skull, indicate dehydration. Because infants have a protuberant abdomen and rely heavily on their diaphragm to breathe, their abdomen appears to move more than their chest during breathing; this is a normal finding and is why infants are often referred to as belly breathers. An infant or a child with inadequate breathing may be tachycardic at first; however, as hypoxia becomes more severe, bradycardia often occurs. Bradycardia in an infant or a child with a respiratory problem indicates impending cardiopulmonary arrest.

Which of the fo llowing conditions would MOST likely cause flushed skin? A) Blood loss B) Hypothermia C) Exposure to heat D) Low blood pressure

C) Exposure to heat Whenever the body temperature rises (ie, heat exposure, fever) , the peripheral blood vessels dilate, which draws warm blood to the skin and gives it a flushed (red) appearance. Blood loss, shock, low blood pressure (hypotension), and hypothermia generally cause the skin to become pale; these conditions cause peripheral vasoconstriction, which shunts blood away from the skin.

Which of the following is a common side effect of nitroglycerin? A) ausea B) Anxiety C) Headache D) Hypertension

C) Headache Because nitroglycerin (NTG) causes vasodilation, including the vessels within the brain, cerebral blood flow increases following its administration. This often causes a pounding headache for the patient. As uncomfortable as it is for the patient, headaches are a common and expected side effect of the drug. The vasodilatory effects of nitroglycerin could result in hypotension; therefore, the patient's blood pressure should be carefully monitored. Nausea and anxiety are common symptoms of acute coronary syndrome; they are not common side effects of nitroglycerin.

Which of the fo llowing structures is responsible for regulating body temperature? A) Cerebrum B) Cerebellum C) Hypothalamus D) Medulla oblongata

C) Hypothalamus The hypothalamus, which is located within the brainstem, regulates body temperature by acting as the body's thermostat. During a heat-related emergency, the hypothalamus can "reset" the body's normal temperature to a much higher temperature in response to the environment and the body's inability to eliminate heat.

Which of the following would clearly be detrimental to a patient in cardiac arrest? A) Ventilating just until the chest rises B) Performing CPR before defibrillation C) Interrupting CPR for more than 10 seconds D) Ventilating without supplemental oxygen

C) Interrupting CPR for more than 10 seconds Major emphasis is placed on minimizing interruptions in CPR. Even brief interruptions cause a drop in coronary and cerebral perfusion. If you must interrupt CPR, do not exceed 10 seconds. Every effort should be made to maintain a high chest compression fraction (CCF). CCF is the percentage of time during a resuscitation in which chest compressions are being performed. It is preferable to ventilate a patient with supplemental oxygen, but failing to do so will not be nearly as detrimental as interrupting CPR for extended periods of time. You should ventilate the patient just until the chest visibly rises; ventilations that are too forceful or too fast can cause hyperinflation of the lungs, which may reduce blood return to the heart. When caring for any patient in cardiac arrest, you should immediately begin CPR, and then apply the AED as soon as possible.

Which of the following is MOST indicative of a primary cardiac problem? A) Tachypnea B) Tachycardia C) Irregular pulse D) Sudden fainting

C) Irregular pulse An irregular pulse signifies an abnormality within the electrical conduction system of the heart. Tachycardia, sudden fainting (syncope), and tachypnea (rapid breathing) can indicate many things other than cardiac problems, such as shock, heat-related problems, and diabetic complications. You should always consider the possibility of a cardiac problem in a patient with an irregular pulse.

Which of the following statements regarding the function of insulin is correct? A) It stimulates the liver to release glucose into the bloodstream. B) It promotes the entry of glucose from the cells into the bloodstream. C) It facilitates the uptake of glucose from the bloodstream into the cells. D) It causes the pancreas to produce glucose based on the body's demand.

C) It facilitates the uptake of glucose from the bloodstream into the cells. Insulin is a hormone produced by the beta cells in the islets of Langerhans of the pancreas. It promotes the uptake of glucose from the bloodstream into the cells, where it is used in the production of energy. Glucagon, a hormone produced by the alpha cells in the pancreas, facilitates the conversion of glycogen to glucose (glycogenolysis) in the liver. The liver does not produce glucose; it produces glycogen, a complex sugar that the body cannot utilize until it has been converted to glucose, a simple sugar.

During a soccer game, an 18-year-old woman injured her knee. Her knee is in a flexed position and is obviously deformed. What should you do? A) Assess for the presence of a femoral pulse B) Straighten the knee to facilitate immobilization C) Manually stabilize the leg above and below the knee D) Ask her to flex her knee even more to help with pain

C) Manually stabilize the leg above and below the knee Treatment for any musculoskeletal injury begins by providing manual stabilization above and below the injury (in this case, the distal femur and proximal tibia); this will prevent further injury. Distal circulatory (pulse), sensory, and motor functions should then be assessed; in this case, the dorsalis pedis or posterior tibial pulse (the femoral pulse is proximal to the knee). After manually stabilizing the injury and assessing distal circulatory, sensory, and motor functions, you should appropriately splint the injury. Reassess distal circulatory, sensory, and motor functions after the splint has been applied. Because of the vascularity of the knee, as well as the presence of major nerves in that area, you should not straighten or flex an injured knee. Joint injuries should be immobilized in the position found . If there is no distal pulse and transport will be delayed, medical control may authorize you to make one attempt to gently manipulate the joint to restore a pulse.

Which part of the assessment process would MOST likely provide clues as to the possible underlying cause of a medical patient's chief complaint? A) Vital signs B) Mental status C) Medical history D) Primary assessment

C) Medical history The primary assessment identifies immediate threats to the patient's life, thereby allowing you to intervene in order to prevent further patient deterioration. The patient's vital signs and mental status also provide clues as to the patient's overall hemodynamic status; however, there are multiple causes of abnormal vital signs and mental status changes. The patient's history would provide you with the most information regarding the possible underlying cause of the patient's chief complaint. For example, discovering that an unresponsive patient (many conditions can cause unresponsiveness) has a history of diabetes would increase your index of suspicion for hypo- or hyperglycemia. A patient with chest pain (many conditions can cause chest pain) in a patient with recent coronary stent placement would increase your index of suspicion for cardiac ischemia.

Which of the following will provide you with the MOST information regarding a head-injured patient's condition? A) Pupil size B) Heart rate C) Mental status D) Blood pressure

C) Mental status The patient's mental status provides you with the most information regarding overall perfusion status, especially when assessing a patient with a head injury. Frequent neurologic assessments, which includes assessing the patient's pupils, are critical in determining if the patient's condition is improving or deteriorating. Vital signs should be monitored according to the patient's condition, at least every 5 minutes if he or she is unstable and at least every 15 minutes if he or she is stable.

Which of the following assessment findings is LEAST suggestive of a cardiac problem? A) Rapid, irregular heart rate B) Anxiety and pale, cool skin C) Palpable pain to the chest D) Nausea and epigastric pain

C) Palpable pain to the chest Signs and symptoms of cardiac compromise include nonreproducible pain, pressure, or discomfort in the chest or epigastric region; nausea; pale, cool, clammy (diaphoretic) skin; and an irregular pulse that is either fast or slow. Pain of cardia origin typically is not reproducible by palpation. Palpable pain to the chest suggests a musculoskeletal problem, not a cardiac problem. However, because some patients with a cardiac problem present atypically, you should transport any patient with chest pain, pressure, or discomfort to the hospital.

A patient with a chest injury has a BP of 100/70 mm Hg and a heart rate of 100 beats/min. Reassessment reveals a BP of 90/74 mm Hg, a pulse of 120 beats/min, and the development of jugular venous distention. What should you suspect? A) Massive hemothorax B) Simple pneumothorax C) Pericardia! tamponade D) Traumatic aortic rupture

C) Pericardia! tamponade The EMT should suspect pericardia! tamponade. Signs of pericardia! tamponade include a narrowing pulse pressure and jugular venous distention (JVD). The first pulse pressure recorded was 30 mm Hg; however, it narrowed to 16 mm Hg during reassessment. The pulse pressure narrows in pericardia! tamponade because pressure against the heart prevents full relaxation and filling; as a result, the diastolic BP increases. Because of inadequate ventricular filling, cardiac output decreases; this causes a decrease in the systolic BP. JVD occurs because blood backs up into the systemic venous system. Massive hemothorax and traumatic aortic rupture would not be expected to cause JVD because there is simply not enough blood in the venous system to distend the jugular veins. Simple pneumothorax typically does not present with signs of shock; the patient usually complains of pleuritic chest pain and shortness of breath. However, in some cases, a simple pneumothorax can progress to a tension pneumothorax and cause hemodynamic compromise.

Which of the following clinical presentations is MOST consistent with right heart fajlure? A) Tachycardia, poor skin turgor, and hypotension B) Orthopnea, crackles in the lungs, and tachycardia C) Peripheral edema and jugular venous distention D) Paroxysmal nocturnal dyspnea and hypertension

C) Peripheral edema and jugular venous distention When the right side of the heart fails, blood backs up beyond the right atrium and into the systemic circulation. Right heart failure produces signs such as peripheral edema, abdominal distention and pain, and jugular venous distention. When the left side of the heart fails, blood backs up beyond the left atrium and into the lungs. Left heart failure produces signs such as dyspnea on exertion, positional dyspnea (orthopnea), paroxysmal nocturnal dyspnea, coughing up blood (hemoptysis), and crackles in the lungs. In addition, patients with left heart fa ilure are commonly hypertensive and tachycardic. Tachycardia, poor skin turgor, and hypotension are clinical signs of severe dehydration, not heart failure.

A week after a near-drowning incident, a 6-year-old boy presents with respiratory distress, tachypnea, and fever. What should you suspect? A) Asthma B) Pertussis C) Pneumonia D) Bronchiolitis

C) Pneumonia Pneumonia is a general term that refers to an infection of the lungs. Pneumonia is often a secondary infection; it commonly occurs during or after a preexisting infection such as a cold. It can also be caused by direct lung injuries, such as from an accidental chemical ingestion or a submersion incident (ie, near-drowning). Common signs and symptoms of pneumonia include rapid breathing (tachypnea), breathing with grunting or wheezing sounds, nasal flaring, and hypothermia or fever. Given this child's history of a recent submersion incident, pneumonia should be the suspected cause of his symptoms. A hallmark of asthma is wheezing; however, fever is usually not present. Bronchiolitis could also explain the child's symptoms; however, the recent submersion incident makes pneumonia far more likely. Furthermore, bronchiolitis more commonly affects infants and toddlers. Pertussis (whooping cough) is not likely because the child does not have a cough.

A 49-year-old woman presents with a headache, nausea, and ringing in her ears. She is conscious and alert and states that she has hypertension and type 2 diabetes. Her BP is 202/114 mm Hg, her pulse is 60 beats/min, and her respirations are 16 breaths/min. What should you do? A) Place her in a supine position. B) Give her one tube of oral glucose. C) Prepare for immediate transport. D) Summon an ALS unit to the scene.

C) Prepare for immediate transport. The patient's presentation is indicative of acute hypertensive crisis. Her blood pressure is severely elevated and she is experiencing other symptoms one might expect with such a high blood pressure (ie, headache, nausea, ringing in the ears). You should place her in a position of comfort, which is usually a semisitting position; lying her supine would likely only make her headache worse. Prompt transport to the hospital is essential so her blood pressure can be lowered in a controlled setting. Without knowing her blood glucose level, proceeding with oral glucose would not be appropriate. If it does not delay transport, you could consider summoning an ALS ambulance to the scene. It is far more important, however, to get her to a definitive care facility.

Which of the following structures stimulates the ventricles and causes them to contract? A) Sinus node B) Sinoatrial node C) Purkinje fibers D) Atrioventricular node

C) Purkinje fibers In a normal heart, the sinoatrial {SA) node, also referred to as the sinus node, is the heart's primary pacemaker. Electrical impulses generated by the sinus node travel throughout both atria via special conduction pathways, causing simultaneous atrial contraction. The impulse continues down the conduction pathway to the atrioventricular {AV) node. At the AV node, the impulse is delayed for a brief period of time in order to allow the ventricles to fill with blood. The impulse then exits the AV node and spreads throughout both ventricles via the bundle of His, the right and left bundle branches, and the Purkinje fibers, causing the ventricular muscle cells to contract.

During which part of your assessment would you MOST likely discover a small-caliber gunshot wound to the back with minimal bleeding? A) General impression B) Primary assessment C) Rapid head-to-toe assessment D) Detailed secondary assessment

C) Rapid head-to-toe assessment During both the general impression and the primary assessment, you should assess for major bleeding. If there is no obvious bleeding, you should continue your assessment as usual. It is during the rapid head-to-toe assessment, when log rolling the patient to assess the posterior (back), that you would most likely find a small-caliber gunshot wound, especially if there is little or no bleeding. A secondary assessment should be performed, and focuses primarily on the patient's chief complaint; however, this may not be practical with a critically ill or injured patient. If a secondary assessment is performed on a critically ill or injured patient, it should occur en route to the hospital. All bleeding should have been controlled long before performing a secondary assessment.

Which of the following findings should make the EMT suspect an ectopic pregnancy in a woman who reports abdominal pain? A) Menstrual period began 3 days ago B) Spontaneous abortion 6 months ago C) Recent pelvic inflammatory disease D) Cesarean section scar on the abdomen

C) Recent pelvic inflammatory disease Pelvic inflammatory disease and scarring of the fallop ian tubes are risk factors for ectopic pregnancy. There is no correlation between past Cesarean deliveries or spontaneous abortions and an increased risk of ectopic pregnancy. It is important to remember that acute abdominal pain in any female of childbearing age, especially with a missed menstrual period, is an ectopic pregnancy until proven otherwise.

Which of the following is MOST indicative of compensated shock in an adult? A) Unresponsive, pallor, absent radial pulses, tachypnea B) Confusion, mottling, tachycardia, BP of 88/60 mm Hg C) Restless, diaphoresis, tachypnea, BP of 104/64 mm Hg D) Weak carotid pulse, cool skin, increased respiratory rate

C) Restless, diaphoresis, tachypnea, BP of 104/64 mm Hg In compensated shock, the nervous system is mounting a physiologic response to an underlying illness or injury in an attempt to maintain perfusion to vital organs such as the brain, heart, and kidneys. The patient with compensated shock is restless or anxious, has poor peripheral perfusion (eg, pallor), tachycardia, diaphoresis, and increased respirations (tachypnea). However, his or her blood pressure is maintained, usually above 90 to 100 mm Hg. In decompensated shock, the body's compensatory mechanisms fail, blood pressure begins to fall, and perfusion to vital organs decreases. Other signs of decompensated shock include a decreased level of consciousness, absent peripheral pulses (radial), and weak central pulses (carotid, femoral).

Sudden cardiac arrest in the adult population MOST often is the result of: A) myocardial infarction. B) respiratory failure. C) a cardiac dysrhythmia. D) accidental electrocution.

C) a cardiac dysrhythmia. The most common cause of sudden cardiac arrest (SCA) in the adult population is a cardiac dysrhythmia--usually ventricular fibrillation. This fact underscores the importance of early defibrillation to shock the heart back into a perfusing rhythm. Evidence has shown that SCA-- again, most often the result of an arrhythmia--occurs in up to 40% of patients experiencing an acute myocardial infarction (AMI). The risk of SCA is highest within the first few hours following the onset of an AMI. Respiratory failure is the most common cause of cardiac arrest in children, not adults. Children generally have healthy hearts and rarely experience cardiac arrest due to a primary cardiac event.

A 31-year-old man presents with uncontrolled twitching of his left arm that began shortly after he experienced an odd taste in his mouth. He denies loss of consciousness and states that this has happened to him before. What should you suspect? A) Absence seizure B) Generalized seizure C) Simple partial seizure D) Complex partial seizure

C) Simple partial seizure Seizures are classified as being generalized or partial. Generalized seizures are further classified as being tonic-clonic (formerly called grand mal) or absence (formerly called petit mal) seizures. Generalized tonic-clonic seizures result from abnormal electrical discharges from large areas of the brain, usually involving both hemispheres. They are characterized by unconsciousness and generalized severe twitching of the body's muscles; they last several minutes or longer. Absence seizures are characterized by a brief lapse of consciousness in which the patient seems to stare and does not respond to anyone. Absence seizures do not involve any changes in motor activity. Partial (focal) seizures begin in one part of the brain and are classified as simple and complex. In a simple partial seizure, no change occurs in the patient's level of consciousness. Patients may report numbness, weakness, or dizziness. The senses may also be involved; the patient may report visual changes and unusual smells or tastes. A simple partial seizure may also cause twitching of the extremity muscles that spreads slowly from one part of the body to another. In a complex partial seizure, the patient has an altered mental status and does not interact normally with his or her environment. This type of seizure results from abnormal discharges from the temporal lobe of the brain. Other signs may include lip smacking, eye blinking, and isolated convulsions or jerking of the body or one part of the body, such as the arm. The patient may experience unpleasant smells and visual hallucinations, exhibit uncontrollable fear, or exhibit repetitive physical behavior such as constant sitting or standing.

Which of the following structures is the primary pacemaker, which sets the normal rate for the heart? A) Bundle of His B) Purkinje fibers C) Sinoatrial node D) Atrioventricular node

C) Sinoatrial node Cardiac pacemakers are bundles of nerves that generate electrical impulses and conduct them to the cardiac cells, resulting in contraction of the myocardium (heart muscle). In a normal healthy heart, the sinoatrial (SA) node is the primary pacemaker that sets the inherent rate for the heart. The SA node generates electricity at a rate of 60 to 100 electrical discharges per minute; hence the normal adult heart rate is 60 to 100 beats/min. The atrioventricular (AV) node serves as the heart's secondary pacemaker; if the SA node fails , the AV node resumes the pacing function of the heart, although at a slower rate (40 to 60 beats/min). The bundle of His and Purkinjie fibers, located within the ventricles, may serve as tertiary pacemakers if the SA and AV nodes fail; their inherent pacing rate is 20 to 40 beats/min.

Which of the following signs or symptoms is MOST suggestive of a systemic reaction following ingestion of a poison? A) Nausea and vomiting B) Burns around the mouth C) Tachycardia and hypotension D) Painful or difficult swallowing

C) Tachycardia and hypotension Once an ingested poison gets into the body, it can affect multiple organ systems. Signs that this is occurring include tachycardia or bradycardia, hypotension or hypertension, weakness, restlessness, and an altered level of consciousness, among others. Local effects of an ingested poison include nausea and vomiting as the poison irritates the gastric lining, burns in and around the mouth, and painful or difficult swallowing (dysphagia).

Which of the following would MOST likely cause a rapid drop in a patient's blood glucose level? A) Mild exertion after eating a meal B) Eating a meal after taking insulin C) Taking too much prescribed insulin D) Forgetting to take prescribed insulin

C) Taking too much prescribed insulin Compared to diabetic coma (hyperglycemic ketoacidosis, hyperglycemic crisis), insulin shock (hypoglycemia, hypoglycemic crisis) has a rapid onset. It is commonly caused when a patient accidentally takes too much prescribed insulin. Insulin is a fast -acting drug that rapidly causes glucose to exit the bloodstream and enter the cells. Other common causes of hypoglycemia include taking a regular dose of insulin but not eating or taking insulin and exercising heavily. Eating a meal after taking insulin typically does not cause a significant change in the patient's blood glucose level. If a person fails to take his or her insulin, glucose will not be able to enter the cells and will accumulate in the bloodstream (hyperglycemia).

Which of the following is the MOST significant finding in a patient with a severe headache? A) Pain in both legs B) Chest discomfort C) Unilateral weakness D) Abdominal tenderness

C) Unilateral weakness Unilateral weakness (weakness on one side of the body) is a clinically significant finding in a patient with a headache because it could indicate a stroke (ischemic or hemorrhagic) or space-occupying lesion (such as a brain tumor). Abdominal, chest, and leg pain are not common complaints associated with a headache, although they should be noted and investigated if they are present.

Which of the following signs and symptoms are MOST characteristic of hyperglycemic ketoacidosis? A) Cool, clammy skin and a slow onset B) Cool, clammy skin and a rapid onset C) Warm, dry skin and a slow onset D) Warm, dry skin and a rapid onset

C) Warm, dry skin and a slow onset Hyperglycemic ketoacidosis (diabetic coma) is characterized by a dangerously high blood glucose level (hyperglycemia); slow onset; warm, dry skin (from dehydration); Kussmaul respirations, which are deep and rapid; and breath with a fruity or acetone odor. Insulin shock results from a low blood glucose level (hypoglycemia) and is characterized by a rapid onset; altered mental status; and cool, clammy skin.

A 66-year-old woman presents with an acute onset of confusion, slurred speech, and an inability to move her right arm and leg. Her airway is patent and she is breathing adequately. Which of the following is the MOST important initial information to obtain about this patient? A) Her initial blood pressure reading B) What she was doing when this began C) When she was last seen normal D) Whether or not her pupils are equal

C) When she was last seen normal This patient is experiencing signs of an acute ischemic stroke. She may be a candidate for fibrinolytic therapy (drugs that dissolve blood clots) if her symptoms are of less than 3 hours' duration. Some patients may be candidates for fibrinolytic therapy for up to 4.5 hours after the onset of symptoms. It is vital to determine exactly (or as close to as possible) when the patient was last seen normal, and pass this information along to the receiving facility. The treatment options for stroke are severely limited if more than 3 hours have past since the symptoms began. Even if 3 hours have passed, prompt action on your part is essential. Assessment of the patient's blood pressure, pupils, and events that preceded the symptoms is important; however, identifying the patient as a candidate for an intervention that may reverse the stroke is critical and will afford her the greatest chance for a positive outcome.

Following administration of nitroglycerin to a man with crushing chest pressure, he experiences a significant increase in his heart rate. This is MOST likely the result of: A) a cardiac dysrhythmia. B) preexisting hypertension. C) a drop in blood pressure. D) coronary vasoconstriction.

C) a drop in blood pressure. Nitroglycerin (NTG) is a vasodilator drug. It dilates not only the coronary arteries, but also other arteries in the body. In some patients, NTG may cause a drop in blood pressure, especially if they are taking other medications that cause vasodilation (eg, erectile dysfunction drugs [Viagra, Levitra, Cialis ]). In response to a drop in blood pressure, the nervous system attempts to compensate by increasing the heart rate (tachycardia). Common side effects of NTG include a headache, a burning sensation under the tongue, and a bitter taste in the mouth. Always assess the patient's vital signs both before and after administering nitroglycerin.

A 50-year-old man presents with crushing chest pain that suddenly began about 30 minutes ago. He is diaphoretic and anxious. The EMT should: A) obtain baseline vital signs. B) apply supplemental oxygen. C) administer chewable aspirin. D) perform a complete physical exam.

C) administer chewable aspirin. Aspirin (up to 325 mg) has clearly been shown to reduce mortality and morbidity from acute myocardial infarction (AMI) and should be given as soon as possible to patients with suspected cardiac chest pain (unless they are allergic to aspirin). Not all patients get oxygen, even those experiencing AMI. Give oxygen if the patient is hypoxemic (oxygen saturation less than 94%) or is experiencing respiratory distress. Clearly, it is important to perform a physical exam and obtain vital signs. Of the interventions listed, however, aspirin administration has the highest priority in this patient.

A 48-year-old man became acutely hypoxic, experienced a seizure, and is now postictal. The MOST effective way to prevent another seizure is to: A) place him in the recovery position. B) give him oral glucose if he can swallow. C) administer high-flow supplemental oxygen. D) dim the lights in the back of the ambulance.

C) administer high-flow supplemental oxygen. You should administer high-flow oxygen to all patients who are actively seizing and to patients who experienced a seizure and are postictal. This is especially true if the seizure was caused by hypoxia. Increasing the oxygen content of the blood, which minimizes hypoxia, may prevent another seizure. The recovery position is appropriate for uninjured patients with a decreased level of consciousness and adequate breathing; it will help maintain the airway and facilitate drainage of secretions from the mouth, but will not prevent another seizure. Oral glucose may prevent another seizure if hypoglycemia was the cause of the seizure. You should dim the lights in the back of the ambulance to help prevent any seizure, not just those that are caused by hypoxia.

A 23-year-old man was struck across the face with a baseball bat. His eyes are swollen shut, he has massive facial bruising and deformities, and he has blood in his mouth. Your MOST immediate concern should be: A) spinal trauma. B) intracranial bleeding. C) airway compromise. D) permanent vision loss.

C) airway compromise. Few things will kill a patient more quickly than uncontrolled external hemorrhage and/or a compromised airway. Blood in a patient's mouth must be removed immediately. It may be aspirated into the lungs or, if clotted, obstruct the airway. Spinal trauma, intracranial bleeding, and vision loss are all possible in a patient with blunt trauma to the face; however, airway compromise is the most immediate life threat with this patient. Remember, treat what will kill your patient FIRST.

While transporting an elderly woman who was complaining of nausea, vomiting, and weakness, she suddenly becomes unresponsive. You should: A) analyze her cardiac rhythm with the AED. B) open her airway and ensure that it is clear. C) assess for signs of breathing and a pulse. D) place her on her side in case she vomits.

C) assess for signs of breathing and a pulse. If a patient is found unresponsive or becomes unresponsive in your presence, your first action should be to assess for breathing and a pulse; this assessment can occur simultaneously and should take no more than 10 seconds. If the patient is breathing adequately and has a pulse, position her on her side and administer oxygen if needed. If the patient has a pulse but is not breathing, open the airway and provide rescue breathing. If the patient is not breathing (or has agonal gasps) and does not have a pulse, begin CPR (starting with chest compressions) and apply the AED as soon as possible. If you are transporting a patient who becomes unresponsive, pulseless, and apneic, you should begin CPR and instruct your partner to stop the ambulance and prepare the AED.

You are treating a 60-year-old man in cardiac arrest. After delivering a shock with the AED and performing CPR for 2 minutes, you achieve return of spontaneous circulation. Your next action should be to: A) provide rapid transport to the hospital. B) reanalyze his rhythm for confirmation. C) assess his airway and ventilatory status. D) remove the AED and apply 100% oxygen.

C) assess his airway and ventilatory status. If return of spontaneous circulation (ROSC) occurs (eg, a palpable pulse is restored), your first action should be to reassess the patient's airway and ventilatory status. If the patient remains apneic, continue rescue breathing at a rate of 10 breaths/min (one breath every 6 seconds). If the patient is breathing adequately, administer supplemental oxygen in a concentration that is sufficient to maintain an oxygen saturation that is between 92% and 98%. After reassessing the airway and breathing, and treating the patient accordingly, you should prepare for immediate transport. Because of the high risk that cardiac arrest can recur following resuscitation, you should not remove the AED pads; simply turn the AED off instead. Analysis of the patient's cardiac rhythm is not indicated because he now has a pulse.

A 29-year-old man presents with an acute onset of severe pain to the left upper quadrant of his abdomen. He is conscious and alert and states that he is very thirsty. The EMT should: A) keep him sitting up and give him small sips of water to drink. B) begin the assessment by palpating the left upper quadrant. C) assess his vital signs and prepare for immediate transport. D) position him supine and encourage him to straighten his legs.

C) assess his vital signs and prepare for immediate transport. At the EMT level, treatment for acute abdominal pain involves addressing any airway, breathing, or circulation issues; assessment of vital signs; treatment for shock (if indicated); and prompt transport to the hospital. Most patients with abdominal pain prefer to lie on their side with their knees drawn up into the abdomen; this position relieves pressure on the abdominal musculature and may afford them some relief. Encourage the patient to remain in whichever position is most comfortable for him or her. Do not give anything by mouth; if the patient becomes nauseated and vomits, the airway could be at risk. Furthermore, if the patient requires surgery, the surgeon will not appreciate you putting anything else in the patient's stomach. When assessing the abdomen, begin palpation at the furthest point from the patient's pain.

You are dispatched to a residence for a 4-year-old girl who is sick. Your assessment reveals that she has increased work of breathing and is making a high-pitched sound during inhalation. Her mother tells you that she has been running a high fever for the past 24 hours. Your MOST immediate concern should be: A) preparing to treat her for a febrile seizure. B) taking her temperature to see how high it is. C) assessing the need for ventilation assistance. D) determining if the child has a history of croup.

C) assessing the need for ventilation assistance. The child is clearly experiencing respiratory distress and probably has croup (laryngotracheobronchitis), a viral upper airway infection. The presence of stridor (high-pitched sound heard during inhalation) indicates swelling of the upper airway. Your most immediate concern should be assessing the adequacy of her breathing and determining if ventilation assistance is necessary. If signs of respiratory failure are present (eg, signs of physical exhaustion, bradycardia, bradypnea [slow respirations]), you must begin assisting her ventilations with a bag-mask device; otherwise, she will likely deteriorate and develop cardiac arrest. She may experience a febrile seizure if her fever acutely spikes; although this is a concern, it is not the most immediate concern in a child with an airway or breathing problem.

A patient with severe hypothermia should be actively rewarmed: A) before he or she is moved. B) in the bak of the ambulance. C) at the emergency department. D) as soon as paramedics arrive.

C) at the emergency department. Prehospital treatment for hypothermia depends on the patient's body temperature. For patients who are alert, actively shivering, and responding appropriately (mild hypothermia [90 to 95 degrees Fahrenheit]), you should begin passive rewarming. This includes moving the patient to a warm environment, removing any wet clothing, turning up the heat in the ambulance, and applying warm blankets. Some protocols may call for heat packs or hot water bottles applied to the groin, axillae, and cervical regions; this is a form of active external rewarming. However, if the patient has moderate or severe hypothermia (less than 90 degrees Fahrenheit), active rewarming is best accomplished in the emergency department utilizing aggressive strategies to introduce heat into the body's core. Such therapies might include warm intravenous fluids, lavage with warm fluids, and rewarming blood outside the body before reintroducing it (extracorporeal rewarming). Rewarming too quickly may cause a fatal cardiac dysrhythmia or other significant complications. For this reason, local protocols may dictate the appropriate type of rewarming strategies based on the patient's body temperature.

A middle-aged man was fo und unresponsive by his wife. When you arrive at the scene, you assess the patient and determine that he is apneic and pulseless. You should: A) immediately begin CPR, reassess for a carotid pulse after 60 seconds, and then apply the AED. B) immediately apply the AED, analyze his cardiac rhythm, deliver a shock if indicated, and begin CPR. C) begin CPR starting with chest compressions, apply the AED as soon as possible, and request backup. D) perform CPR with a compression to ventilation ratio of 15:2, apply the AED, and request backup.

C) begin CPR starting with chest compressions, apply the AED as soon as possible, and request backup. As soon as you determine that a patient is unresponsive, pulseless, and apneic, you should begin CPR (starting with chest compressions), apply the AED as soon as possible, and deliver a shock (if indicated). The appropriate compression to ventilation ratio for adult CPR (one- or two-rescuer) is 30:2. A compression to ventilation ratio of 15:2 is used for two-rescuer infant and child CPR. Request a backup ambulance as soon as possible; however, do not interrupt CPR to do so. One EMT should perform CPR while the other radios for assistance. Continue CPR and reanalyze the patient's cardiac rhythm every 2 minutes. If indicated, deliver a single shock and immediately resume CPR, starting with chest compressions. If the AED gives a no shock message, resume CPR, starting with chest compressions. Continue CPR, rhythm analysis every 2 minutes, and defibrillation (if indicated), until backup arrives or the patient starts to move.

A 56-year-old man is found to be pulseless and apneic. His wife states that he collapsed about 5 minutes ago. As your partner gets the AED from the ambulance, you should: A) provide rescue breaths until the AED is ready. B) open the airway and give 2 rescue breaths. C) begin CPR, starting with chest compressions. D) ask the wife if the patient has a living will.

C) begin CPR, starting with chest compressions. When you arrive on scene and dete1mine that a patient is in cardiac arrest, you should immediately begin CPR, starting with chest compressions. Perform 30 chest compressions and then open the airway and deliver 2 rescue breaths. Chest compressions are a crucial part of cardiopulmonary resuscitation and must be started without delay. Apply the AED as soon as it is available. In the interest of this patient, whose arrest interval is short, you should begin resuscitative efforts immediately. In some cases, it is appropriate to inquire about the presence of a living will; however, this should be done after resuscitative efforts have begun.

Naloxone improves ventilation in a patient who overdosed on an opioid because it: A) neutralizes the drug in the bloodstream. B) allows more oxygen to bind to hemoglobin. C) binds to receptor sites and prevents drug uptake. D) dilates the bronchioles and opens the lower airways.

C) binds to receptor sites and prevents drug uptake. Naloxone (Narcan) is an opiate/opioid receptor antagonist. This means that it binds to opiate/opioid receptors in the body and prevents uptake of the drug by the central nervous system. As a result, the drug loses its influence over the respiratory centers and ventilation improves. Naloxone does not neutralize or break down the drug. Beta-2 agonists such as albuterol cause bronchodilation, thus opening the lower airways; naloxone has no effect on the bronchioles. Oxygen-hemoglobin binding increases when the patient's ventilations improve; naloxone does not directly do this.

Following blunt injury to the anterior torso, a patient is coughing up bright red blood. You should suspect: A) intra-abdominal bleeding. B) gastrointestinal bleeding. C) bleeding within the lungs. D) severe myocardial damage.

C) bleeding within the lungs. Hemoptysis (coughing up blood) is a finding that suggests injury to or bleeding within the lungs. Vomiting of bright or dark red blood (hematemesis) suggests gastrointestinal bleeding. Intra-abdominal bleeding following trauma presents with signs of shock as well as abdominal pain, guarding, rigidity, bruising, or distention. Injury to the myocardium may cause cardiac dysrhythmias, but typically does not cause hemoptysis unless it is associated with pulmonary (lung) injury.

A patient who overdosed on an opioid would be expected to present with: A) tachycardia. B) dilated pupils. C) bradypnea. D) hyperpnea.

C) bradypnea. As with all opioids, overdose causes depression of the central nervous system (CNS), resulting in a decreased level of consciousness, bradypnea (slow breathing), shallow depth of breathing (reduced tidal volume), bradycardia, and hypotension. Hyperpnea (deep breathing) would not be present in a patient who overdosed on an opioid. In an opioid overdose, the pupils are typically constricted (miosis). Barbiturates, such as phenobarbital, are also CNS depressants and cause the same symptoms seen with opioid overdose. However, the patient's pupils are typically dilated (mydriasis), not constricted.

You have analyzed a cardiac arrest patient's rhythm three times with the AED, separated by 2-minute cycles of CPR, and have received no shock messages each time. You should: A) consider terminating resuscitation. B) remove the AED and continue CPR. C) continue CPR and transport at once. D) request a paramedic unit at the scene.

C) continue CPR and transport at once. Although protocols vary from system to system, it is generally agreed that if you receive three consecutive no shock messages, separated by 2-minute cycles of CPR, you should continue CPR and transport at once; it is unlikely that the patient will convert to a shockable rhythm (eg, V-Fib, pulseless V-Tach). En route, coordinate a rendezvous with a paramedic unit if possible; waiting at the scene would simply delay further treatment. The decision to terminate resuscitative efforts is made by a physician, and in some cases, a paramedic, after adequately performed BLS and ALS have proved unsuccessful.

During your assessment of a woman in labor, you see the baby's arm protruding from the vagina. The mother tells you that she needs to push. You should: A) gently push the protruding arm back into the vagina. B) encourage the mother to push and give her high-flow oxygen. C) cover the arm with a sterile towel and transport immediately. D) insert your gloved fingers into the vagina and try to turn the baby.

C) cover the arm with a sterile towel and transport immediately. On rare occasions, the presenting part of the fetus is neither the head nor the buttocks, but a single arm or leg. This is called a limb presentation. You cannot successfully deliver such a presentation in the field. These infants usually must be delivered at the hospital. If you encounter a limb presentation, instruct the mother to stop pushing if she is experiencing a contraction; instead, instruct her to pant. Pushing may place pressure on the fetus, potentially causing injury. Cover the protruding limb with a sterile dressing or towel and transport immediately. Never try to push the limb back in, and never pull on it. Place the mother on her back, with head down and pelvis elevated. Because both mother and fetus are likely to be physically stressed in this situation, give the mother high-flow oxygen.

An elderly woman, who was removed from her burning house by firefighters, has full-thickness burns to approximately 50% of her body. Appropriate treatment for this patient would include: A) applying moist, sterile dressings to the burned areas and preventing hypothermia. B) cooling the burns with sterile saline and covering them with dry, sterile burn pads. C) covering the burns with dry, sterile dressings and preventing further loss of body heat. D) peeling burned clothing from the skin and removing all rings, necklaces, and bracelets.

C) covering the burns with dry, sterile dressings and preventing further loss of body heat. After moving the patient to safety, stopping the burning process, and supporting the ABCs, full-thickness burns should be cared for by applying dry, sterile dressings or sterile burn pads and preventing hypothermia. Cooling full-thickness burns (ie, applying moist dressings, pouring saline or water on the burn) should be avoided, as this increases the risks of hypothermia and infection. Rings, necklaces, and other potentially constrictive devices should be removed in the event that severe swelling occurs. If portions of clothing are adhered to the skin, they should be cut around, not peeled from the skin, to prevent further soft-tissue damage.

During your assessment of a patient with a gunshot wound to the chest, you note that his skin is pale. This finding is the result of: A) a critically low blood pressure. B) a significantly elevated heart rate. C) decreased blood flow to the skin. D) peripheral dilation of the vasculature.

C) decreased blood flow to the skin. When the body attempts to compensate for shock, peripheral vasoconstriction shunts blood away from the skin to the more vital organs in the body such as the brain, heart, lungs, and kidneys. When there is minimal or no peripheral blood flow, the skin assumes a pale appearance. By contrast, when peripheral circulation increases (ie, vasodilation), the skin assumes a red (flushed) appearance. Pallor does not necessarily indicate hypotension. Tachycardia is a compensatory response of the nervous system in an attempt to increase cardiac output and maintain adequate perfusion.

After applying the AED to an adult patient in cardiac arrest, you analyze her cardiac rhythm and receive a shock advised message. Emergency medical responders, who arrived at the scene before you, tell you that bystander CPR was not in progress upon their arrival. You should: A) perform CPR for 2 minutes and then defibrillate. B) detach the AED and prepare for immediate transport. C) deliver the shock as indicated followed immediately by CPR. D) notify medical control and request permission to cease resuscitation.

C) deliver the shock as indicated followed immediately by CPR. Information obtained at the scene-whether from emergency medical responders or family members-is not always accurate. If the AED advises you to defibrillate, you should do so and then immediately begin or resume CPR, starting with chest compressions. During CPR, ensure adequate oxygenation and ventilation, minimize interruptions in chest compressions, obtain the patient's medical history if possible, and contact medical control as needed. At no time during resuscitative efforts should you detach the AED from the patient. Reanalyze the patient's cardiac rhythm after 2 minutes of CPR and follow the voice prompts.

During your assessment of a patient who experienced blunt facial trauma, you note the presence of a hyphema. This indicates: A) a fracture of the nasal bone. B) an orbital blowout fracture. C) direct trauma to the eyeball. D) that the pupils are unequal.

C) direct trauma to the eyeball. Some patients with blunt trauma to the eyeball (globe) may present with a hyphema, or bleeding into the anterior chamber of the eye, that obscures a portion of or the entire iris. This condition may seriously impair vision and should be considered a sight- threatening emergency. A fracture of the orbital floor (blowout fracture) is characterized by double vision and an inability of the patient to move his or her eyes above the midline (paralysis of upward gaze) following blunt facial trauma. In an orbital blowout fracture, fragments of fractured bone can entrap some of the muscles that control eye movement. Anisocoria is the term used to describe unequal pupils. Unequal pupils following head trauma indicates increased intracranial pressure.

When treating an unresponsive man who was struck by lightning, you should: A) begin CPR at once if he is apneic and pulseless. B) apply full spinal precautions before moving him. C) ensure that you and the patient are in a safe place. D) manually stabilize his head and open his airway.

C) ensure that you and the patient are in a safe place. Contrary to popular belief, lightning can (and does) strike in the same place twice. After lightning strikes, the ground remains electrically charged for a period of time; this increases the chance of a second strike within a short period of time. You must first ensure that you and the patient are safe by moving to a sheltered area, preferably indoors. After you have ensured the safety of yourself and the patient, begin treatment as dictated by the patient's condition. Do not let the life you save, or attempt to save, TAKE your own!

A 73-year-old man presents with confusion; cool, pale, clammy skin; absent radial pulses; and a blood pressure of70/40 mm Hg. The patient's wife tells you that he has had abdominal pain for a week and began vomiting a coffee-ground substance yesterday. His past medical history includes hypertension and gastric ulcer disease. Your MOST immediate concern should be that: A) his blood glucose level is probably too high. B) he is bleeding from his gastrointestinal tract C) he is in shock and requires prompt transport. D) his condition requires surgery within 2 hours.

C) he is in shock and requires prompt transport. The patient is likely bleeding from his gastrointestinal (GI} tract. Although this is a serious condition, it is not a condition you can treat; internal bleeding cannot be controlled in the field. You can, however, treat his signs and symptoms of shock by administering high-flow oxygen and keeping him warm by covering him with a blanket. Therefore, this should be your most immediate concern. Furthermore, the patient requires prompt transport to the hospital where he can receive definitive care, which may or may not involve surgery. It is highly unlikely that his blood glucose level is too high; he does not have a history of diabetes.

Patients who abuse opioids by injecting them are also at risk for: A) hepatitis A. B) tuberculosis. C) hepatitis C. D) schizophrenia.

C) hepatitis C. Patients who abuse opioids by injecting them often share needles with other people. This significantly increases their risk for contracting bloodborne pathogens such as hepatitis C and human immunodeficiency virus (HIV). Hepatitis A is spread by the fecal-oral route; it is not a bloodborne pathogen. Tuberculosis is spread by the droplet (aerosolization) route; it is not a bloodborne pathogen. Schizophrenia is a spectrum of psychiatric illnesses; there is no correlation between opioid abuse and schizophrenia.

Factors that affect a person's ability to compensate for internal or external blood loss include all of the following, EXCEPT: A) advanced age. B) the rate of blood loss. C) high cholesterol in the blood. D) blood-thinning medications.

C) high cholesterol in the blood. Numerous factors affect a person's ability to compensate for blood loss. The compensatory responses of tachycardia and peripheral vasoconstriction decrease as a person ages; thus, older patients are not able to compensate as effectively as younger patients. Furthermore, older patients commonly take medications to treat high blood pressure, such as beta-blockers; these drugs may blunt the body's release of the catecholamines necessary to increase the heart rate. The ability to compensate for blood loss is also related to how rapidly blood loss occurs. A normal, healthy adult can comfortably donate 1 unit (500 mL) of blood during a period of 10 to 20 minutes and adapts well to this decrease in blood volume. However, if a similar blood loss occurs in a much shorter period, the person's compensatory mechanisms may be overwhelmed, resulting in hypovolemic shock. Patients who take blood-thinning medications (eg, warfarin [Coumadin ]) bleed longer than those not taking such medications; in these patients, bleeding from an otherwise minor injury can be significant. There is no known correlation between high cholesterol and a person's ability to compensate for blood loss.

Patients with significant closed head injuries often have pupillary abnormalities and: A) paralysis. B) paresthesia. C) hypertension. D) tachycardia.

C) hypertension. Closed head injuries can cause a variety of signs and symptoms. In addition to pupillary abnormalities (ie, unequal pupils, sluggishly reactive pupils), a classic finding that indicates a significant increase in intracranial pressure is Cushing's triad. This trio of findings includes hypertension; bradycardia; and abnormal breathing, which can vary from slow and irregular to rapid and deep.

Your patient is a 75-year-old woman who, according to her son, tripped on a throw rug and fell. The patient is conscious and alert, is sitting on the couch, and has a hematoma to her forehead. When you ask her what happened, her son interjects by saying, "I already told you, she tripped and fell!" As you further question her, you find that she is hesitant to answer your questions and keeps looking at her son. You should: A) ask the patient if someone hurt her intentionally. B) notify law enforcement and have the son removed. C) interview the patient and her son separately if possible. D) tell the son that you suspect his mother has been abused.

C) interview the patient and her son separately if possible. There are several indicators that suggest this patient's injury was intentionally inflicted: the son's interjection, her hesitancy in answering your questions, and the fact that she keeps looking at her son. If possible, interview the patient and her son separately; doing so may reveal inconsistencies regarding the event, which would further increase your index of suspicion for abuse. You must not, however, accuse her son of abuse or insinuate such. If you are wrong, you could be liable for slander. At present, the son is not verbally or physically abusive; however, if he becomes abusive, you should notify law enforcement at once. Remember, you are legally obligated to report any suspicions of abuse or neglect to the emergency department physician. You should make every effort to convince the patient to consent to treatment and transport.

A rapid heart rhythm, usually at a rate of150 to 200 beats/min, that originates in the ventricles and can cause hemodynamic compromise is called: A) asystole. B) ventricular fibrillation. C) ventricular tachycardia. D) pulseless electrical activity.

C) ventricular tachycardia. Ventricular tachycardia (V-Tach) is a rapid heart rhythm, usually at a rate of 150 to 200 beats/min, that originates in the ventricle instead of the atrium. VTach usually does not allow adequate time between beats for the left ventricle to fill with blood; therefore, the blood pressure may fall. V-Tach may occur with or without a pulse. Ventricular fibrillation (V-Fib) is a disorganized, chaotic dysrhythmia that does not produce a pulse; it occurs when cardiac cells randomly discharge to the point that regular cardiac contraction is impossible. Asystole (flatline) represents an absence of cardiac electrical and mechanical activity; obviously, asystole does not produce a pulse. Pulseless electrical activity (PEA) is a phenomenon in which organized electrical activity is present on the cardiac monitor, despite the absence of a pulse.

Following delivery of the baby and placenta, the mother continues to experience moderate vaginal bleeding. The EMT should: A) carefully insert several gauze pads into her vagina. B) sit her up and apply direct pressure to her vaginal area. C) keep her supine and massage the fund us of her uterus. D) place her on her side and elevate her lower extremities.

C) keep her supine and massage the fund us of her uterus. Postpartum hemorrhage should be treated by placing the mother supine and firmly massaging the fund us (top) of the uterus. Fundal massage causes vasoconstriction of the uterine blood vessels, which is why it helps control bleeding. Another option is to allow the mother to nurse her baby; doing so causes her pituitary gland to release oxytocin, which also causes vasoconstriction of the uterus. Inserting any type of dressing or gauze into the vagina is clearly inappropriate and will not help control the bleeding. Direct pressure to the vaginal area will accomplish nothing in controlling postpartum hemorrhage. If postpartum hemorrhage is severe enough to produce signs of shock, keep the mother supine, cover her with a warm blanket, administer oxygen, and transport without delay.

When assessing and treating a patient with a gunshot wound, you should routinely: A) apply ice directly to the wound. B) determine why the patient was shot. C) look for the presence of an exit wound. D) evaluate the pulses proximal to the wound.

C) look for the presence of an exit wound. When assessing a patient who sustained a gunshot wound, you should routinely look for an exit wound, which may be difficult to find. Exit wounds can be a source of continued bleeding, both externally and internally. They may or may not follow the same path as the entrance wound. This is why it is important to conduct a thorough examination of the patient. Ice can be applied to the wound, but only after the wound has been covered by a sterile dressing and any bleeding has been controlled. Determining why the patient was shot is the responsibility of law enforcement, not the EMT. If the wound is close to an extremity, pulse, motor, and sensory function should be assessed distal to the wound.

A woman who is 34 weeks' pregnant is in cardiac arrest. During the resuscitation attempt, you should: A) defer defibrillation until arrival at the hospital. B) perform chest compressions at a faster rate. C) manually displace the uterus toward the left. D) allow partial chest recoil in between compressions.

C) manually displace the uterus toward the left. Treatment for a pregnant patient in cardiac arrest is no different than in nonpregnant patients, with a few exceptions. If the woman is in the last month or two of pregnancy, compressions may need to be performed a little higher on the sternum than usual. This is because the pregnant uterus can push the heart higher into the chest cavity. Otherwise, compressions should be at a rate of 100 to 120/min, at a depth of at least 2 inches. Full recoil of the chest is critical in allowing more blood to return to the heart. A person should be assigned to manually displace the uterus to the left; this relieves pressure from the inferior vena cava and facilitates blood return to the heart. If the AED advises a shock, deliver it! Do not defer defibrillation.

The MOST obvious way to reduce heat loss from radiation and convection is to: A) move away from a cold object. B) wear a thick wind-proof jacket. C) move to a warmer environment. D) increase metabolism by shivering.

C) move to a warmer environment. In a cold environment, the body has two ways of staying warm: generating heat (thermogenesis) and reducing heat loss. Radiation is the transfer of heat by radiant energy. The body can lose heat by radiation, such as when a person stands in a cold room. Convection occurs when heat is transferred to circulating air, as when cool air moves across the body's surface. A person standing in windy cold weather, wearing lightweight clothing, is losing heat to the environment mostly by convection. The quickest and most obvious way to decrease heat loss from radiation and convection is to move out of the cold environment and seek shelter from wind. Shivering increases the body's metabolism and is a mechanism for generating heat, not reducing heat loss. Layers of clothing trap air and provide excellent insulation; thus, layered clothing decreases heat loss better than a single, thick jacket. Conduction is the direct transfer of heat from a part of the body to a colder object by direct contact, as when a warm hand touches cold metal or ice. The most obvious way to decrease heat loss by conduction is to remove your hand from the cold object.

The middle, muscular layer of the heart is called the: A) epicardium. B) pericardium. C) myocardium. D) endocardium.

C) myocardium. The heart has four layers. The inner layer is called the endocardium, the middle layer is composed of muscle and is called the myocardium (myo = muscle), and the outer layer of the heart itself is called the epicardium. The pericardium, which is a thin, fibrous membrane, encapsulates the entire heart.

Chest compression effectiveness is MOST effectively assessed by: A) listening for a heartbeat with each compression. B) carefully measuring the depth of each compression. C) palpating for a carotid pulse with each compression. D) measuring the systolic blood pressure during compressions.

C) palpating for a carotid pulse with each compression. When chest compressions are in progress, the most reliable method of determining their effectiveness is to palpate for a carotid or femoral pulse. If compressions are of adequate depth for the patient's age, you should be able to feel a pulsation during each compression. It should be noted that you may not be able to feel a pulse in some patients, despite adequately performed chest compressions. For example, if the patient has lost a significant amount of blood, there is less volume to create the pressure wave against the arterial wall. If the patient has a pericardia! tamponade or tension pneumothorax (examples of obstructive shock), then the forward flow of blood may be obstructed; in these cases, you may not feel a pulse during chest compressions.

When caring for a patient with severe hypothermia who is in cardiac arrest, you should: A) avoid using the AED. B) hyperventilate the patient. C) perform BLS and transport. D) perform rescue breathing only.

C) perform BLS and transport. Patients with severe hypothermia who are in cardiac arrest should be managed with basic life support (chest compressions and ventilations), passive external rewarming (ie, removal of wet clothing, applying warm blankets), and rapid transport to the hospital, where they can be actively rewarmed. Because cold muscle is a poor conductor of electricity, defibrillation, may be less effective until the patient's body temperature has been increased. However, you should not withhold defibrillation if it is indicated. Hyperventilation should be avoided, as this may increase intrathoracic pressure and impair blood flow back to the heart.

Your assessment of a 5-year-old child reveals that he is unresponsive with a respiratory rate of 8 breaths/min and a heart rate of 50 beats/min. Treatment for this child should include: A) high-flow oxygen via nonrebreathing mask and rapid transport. B) oxygen via a nasal cannula at 6 L/min and rapid transport. C) positive pressure ventilation, chest compressions, and rapid transport. D) back slaps and chest thrusts while attempting artificial ventilations.

C) positive pressure ventilation, chest compressions, and rapid transport. A heart rate less than 60 beats/min in an infant or child, especially when accompanied by signs of poor perfusion and inadequate breathing, should be treated with positive pressure ventilation, chest compressions, and rapid transport. Respirations of 8 breaths/min and a heart rate of 50 beats/min will not maintain adequate oxygenation and perfusion in a child. Passive oxygenation (ie, nasal cannula, nonrebreathing mask) is not appropriate for a child with inadequate ventilation, especially when accompanied by bradycardia. Back slaps and chest thrusts are indicated for a responsive infant with a severe foreign body upper airway obstruction.

After the baby's head delivers, it is usually tilted: A) with the face up. B) posteriorly, face down. C) posteriorly, to one side. D) anteriorly, with the chin up.

C) posteriorly, to one side. As the baby's head begins to deliver, it is usually in a posterior, face -down position. After the head delivers completely, however, it usually tilts to the side in preparation for delivery of the shoulders. Remember to check for the presence of a nuchal cord (umbilical cord wrapped around the neck), and to suction the baby's mouth and nose as soon as its head delivers.

The chest pain associated with an acute coronary syndrome is often described as: A) sharp. B) stabbing. C) pressure. D) cramping.

C) pressure. The pain associated with acute coronary syndrome (ie, unstable angina, acute myocardial infarction) is commonly described as a feeling of pressure or heaviness, discomfort, or a crushing pain. Stabbing or sharp (pleuritic) pain is often associated with conditions such as a spontaneous pneumothorax or pulmonary embolism. Do not rule out a cardiac problem just because the patient does not have the "classic" quality of pain, especially if the patient has a history of cardiac problems.

Functions of the liver include: A) production of insulin. B) production of red blood cells. C) production of clotting factors. D) concentration and storage of bile.

C) production of clotting factors. The liver is a solid, very vascular organ that performs many functions. Among these functions are detoxification of harmful substances; the production of glycogen, a complex sugar; the production of bile (bile is concentrated and stored in the gallbladder); and production of many of the body's blood-clotting factors. Insulin is produced by the pancreatic beta cells. Red blood cells are produced by red bone marrow.

You arrive at a residence shortly after a 4-year-old boy experienced an apparent febrile seizure. The child is alert and crying. His skin is flushed, hot, and moist. His mother tells you that the seizure lasted about 2 minutes. You should: A) begin rapid cooling measures at once. B) give him acetaminophen or ibuprofen. C) provide supportive care and transport. D) allow the mother to take her child to the doctor.

C) provide supportive care and transport. Febrile seizures are common in children between the ages of 6 months and 6 years; they occur when the child's body temperature suddenly rises or when an already febrile child experiences an acute fever spike. Treatment for a child who has experienced a febrile seizure involves providing supportive care (eg, monitoring ABCs, administering oxygen as tolerated) and transporting to the hospital. Any infant or child who experienced a seizure should be transported by EMS. Although febrile seizures are typically self-limiting and are rarely life threatening, there are other causes of fever and seizures, such as meningitis. The EMT is generally not authorized to administer medications unless directed by local protocol or direct medical control. Active cooling (eg, cool or cold water baths) should be avoided; a sudden lowering of the child's temperature may cause him to shiver, which may cause a sudden fever spike and induce another seizure.

Freshly oxygenated blood returns to the heart via the: A) aorta. B) venae cavae. C) pulmonary veins. D) pulmonary arteries.

C) pulmonary veins. The pulmonary veins are the only veins that carries oxygen-rich blood. They carry blood from the lungs back to the left atrium. All other veins in the human body, including the venae cavae, carry deoxygenated blood back to the heart. The aorta is the largest artery in the body and branches immediately from the left ventricle, carrying fresh ly oxygenated blood to the rest of the body. The pulmonary arteries carry deoxygenated blood from the right ventricle to the lungs for reoxygenation.

When restraining a violent patient, you should make sure that: A) at least two EMTs restrain the patient. B) the patient is restrained using maximal force. C) someone talks to the patient during the process. D) consent for restraint has been obtained from a family member.

C) someone talks to the patient during the process. If a violent patient needs to be restrained, you must ensure the presence of at least four people (one per extremity). One of the EM Ts should continuously talk to the patient to explain what is happening, even if the patient is not listening. Restraint is a last resort used to protect the EMT as well as the patient. Consent is not needed from a family member prior to restraining the patient. Just enough force to effectively restrain the patient is all that is required to prevent causing unnecessary injury.

Aspirin may be contraindicated in patients with: A) glaucoma. B) diabetes. C) stomach ulcers. D) ibuprofen allergy.

C) stomach ulcers. Aspirin (acetylsalicylic acid [ASA]) inhibits platelet aggregation, thus preventing clots from forming or preventing an existing clot from getting bigger. Aspirin, in a dose of 160 to 325 mg, should be admini tered to patients experiencing acute coronary syndrome (ie, unstable angina, acute myocardial infarction) as soon as possible. Aspirin is absolutely contraindicated for patients who are allergic to salicylates. Because aspirin prolongs bleeding time, it may be contraindicated for patients with stomach ulcers; therefore, you should contact medical control before giving aspirin to such patients. Aspirin is not contraindicated for patients with glaucoma or diabetes. Ibuprofen, the active ingredient in Motrin and Advil, is a nonsteroidal anti-inflammatory drug ( SAID), not a salycilate.

The maximum pressure generated in the arms and legs during contraction of the left ventricle is called: A) afterload. B) pulse pressure. C) systolic blood pressure. D) diastolic blood pressure.

C) systolic blood pressure. Blood pressure is the force of circulating blood against the arterial walls. Systolic blood pressure (SBP) is the maximum pressure generated in the arms and legs during contraction of the left ventricle, during the time period known as systole. As the left ventricle relaxes in the stage known as diastole, the arterial pressure falls. When the left ventricle relaxes, the aortic valve closes and blood flow between the left ventricle and the aorta stops. The diastolic blood pressure (DBP) is the pressure exerted against the arterial walls while the left ventricle is at rest, during the time period known as diastole. Pulse pressure is the numeric difference between the SBP and DBP; it represents the force generated by the heart each time it contracts. If the patient's BP is 120/80 mm Hg, the pulse pressure would be 40 mm Hg. Afterload refers to the amount of resistance the left ventricle must contract against. Conditions such as hypertension cause an increase in afterload; as the arteries narrow, the left ventricle must work harder to overcome the increases resistance to forward blood flow.

In contrast to a patient with a blood glucose level of 25 mg/dL, a patient with a blood glucose level of 800 mg/dL would be expected to present with: A) profuse diaphoresis and pallor. B) an acute alteration in mentation. C) tachycardia and poor skin turgor. D) a rapid, shallow breathing pattern.

C) tachycardia and poor skin turgor. Insulin shock, which results from hypoglycemia, presents with an acutely altered mental status; rapid, shallow breathing; pallor, and diaphoresis. Insulin shock commonly occurs when the patient takes too much of their insulin, or when they take their normal dose of insulin but do not eat. At any rate, the progression to insulin shock is fast. By contrast, the progression to diabetic coma (caused by hyperglycemia and ketoacidosis) is slow, often over a period of days. Failure to take one's prescribed insulin or inadvertently taking too little insulin are common causes. High blood glucose levels result in excess water excretion by the kidneys. This causes dehydration, the signs of which are tachycardia; warm, dry skin; and poor skin turgor. In an effort to eliminate ketoacids from the body, the respiratory rate and depth both increase (Kussmaul respirations), and the acetone breath odor is reflective of ketone removal via the respiratory system.

If a passenger strikes his or her head on the windshield during a motor vehicle crash: A) the posterior portion of the brain will receive the initial impact, resulting in severe intracerebral hemorrhage. B) he or she will likely experience a hyperflexion injury, resulting in fractures of the vertebrae in the cervical spine. C) the anterior part of the brain sustains a compression injury, while the posterior part sustains a stretching injury. D) you will always see a starburst fracture of the windshield at the location where the patient struck his or her head.

C) the anterior part of the brain sustains a compression injury, while the posterior part sustains a stretching injury. Although the presence of a starburst fracture on the windshield is a good indicator that the patient impacted the windshield with his or her head, it is not always present or grossly obvious, especially if the windshield is broken in multiple places. As the passenger's head strikes the windshield, the brain continues its forward movement until it collides with the inside of the skull. Direct injury to the anterior part of the brain results in compression injuries. Indirect injury occurs to the posterior part of the brain due to stretching or tearing. This is an example of a coup-contracoup injury. Although hyperflexion injuries of the neck can occur when the head impacts the windshield, hyperextension injuries are more common.

You receive a call for a 54-year-old woman who is having a seizure. When you arrive at the scene, the patient is actively seizing. Her husband tells you that she has a history of seizures and that she has been seizing continuously for 20 minutes. It is MOST important for you to recognize that: A) her seizure could be the result of a low blood glucose level. B) the patient may have stopped taking her seizure medication. C) the patient is hypoxic and needs oxygen as soon as possible. D) her husband's description is consistent with status epilepticus.

C) the patient is hypoxic and needs oxygen as soon as possible. There are many causes of seizures, including noncompliance with prescribed anticonvulsant medications, hypoglycemia, stroke, head trauma, and poisoning, among others. Regardless of the underlying cause, it is most important to recognize that patients who are actively seizing, especially if the seizure is prolonged, are hypoxic and need supplemental oxygen as soon as possible; in many cases, it may be necessary to assist the patient's ventilations. The husband's description of the seizure (eg, continuous for 20 minutes) is consistent with status epilepticus, which is all the more reason to recognize that she is severely hypoxic. Seizure-related deaths are the result of cerebral hypoxia.

When the vital organs of an 85-year-old patient need additional blood flow, the heart may not be able to meet the increased need because: A) stroke volume increases as a person ages. B) blood pressure decreases as a person ages. C) the reserve capacity of the heart is reduced as a person ages. D) the blood vessels become more elastic as a person ages.

C) the reserve capacity of the heart is reduced as a person ages. Like other body systems, the cardiovascular system undergoes changes as we get older. As the heart's muscle mass and tone decrease, the amount of blood pumped out of the heart per beat (stroke volume) is reduced. The residual (reserve) capacity of the heart is also reduced; therefore, when the vital organs of the body need additional blood flow, the heart is less able to meet that need as effectively as a younger person's heart. The vascular system also is affected by the aging process. Arteriosclerosis (hardening of the arteries) can develop, affecting perfusion of the tissues. In arteriosclerosis, the arteries become less elastic and more narrowed; as a result, blood pressure typically increases with age.

The MAIN reason why small children should ride in the backseat of a vehicle is because: A) they are much less likely to be ejected from the vehicle. B) the back of the front seat will provide a cushion during a crash. C) they can experience severe injury or death if the air bag deploys. D) their legs are highly prone to injury from striking the dashboard.

C) they can experience severe injury or death if the air bag deploys. Children younger than 12 years should ride in the backseat of a vehicle, preferably in the middle, and be restrained in a device that is appropriate for their size. Merely placing the child in the backseat does not reduce the risk of ejection; the child must be properly restrained. Young children, especially those restrained in a child safety seat, may be critically injured or killed by air bags if they are riding in the front passenger seat of a car. This occurs because the child safety seat positions the child too close to the air bag; the force of the deploying air bag may cause severe head and spinal trauma.

If a vehicle strikes a tree at 60 mph, the unrestrained driver would likely experience the MOST severe injuries during the: A) first collision. B) second collision C) third collision. D) fourth collision.

C) third collision. Motor vehicle crashes typically consist of three separate collisions. Understanding the events that occur during each collision will help you remain alert for certain types of injury patterns. During the first collision, the vehicle strikes another object. Damage to the car is perhaps the most dramatic part of the collision, but it does not directly affect patient care. It does, however, provide information about the severity of the collision; thus, it has an indirect effect on patient care. During the second collision, the passenger collides with the interior of the vehicle. Just like the obvious damage to the exterior of the car, the injuries that result are often dramatic and usually apparent during your primary assessment. During the third collision, the occupant's internal organs collide with the solid structures of the body. Although the injuries that occur during the third collision may not be as obvious as those that occur during the second collision, they are often the most life-threatening.

A child typically begins to develop stranger anxiety when he or she is a/an: A) infant. B) neonate. C) toddler. D) preschooler.

C) toddler. A toddler is a child between 1 and 3 years of age. During this period, children begin to walk and explore their environment. They are able to open doors, drawers, boxes, and bottles. Because they are explorers by nature and are not afraid, injuries in this age group increase. Stranger anxiety also develops early in this period. Toddlers often resist separation from caregivers and are afraid to let others come near them.

A 60-year-old man reports a sudden tearing sensation in his abdomen. He is conscious and alert with a blood pressure of 148/88 mm Hg, a pulse of 120 beats/min, and respirations of 22 breaths/min. You should: A) auscultate over his epigastrium to assess for bowel sounds. B) vigorously palpate his abdomen to assess for a pulsating mass. C) transport at once and be prepared to treat him for severe shock. D) perform a rapid head-to-toe assessment and prepare for transport.

C) transport at once and be prepared to treat him for severe shock. Given the onset and nature of the patient's pain (eg, sudden onset, tearing sensation), you should suspect that he has an acute aortic dissection. Prompt transport is essential. Acute aortic dissection can quickly cause an aneurysm, which could rupture and cause profound shock. Therefore, you must carefully monitor him and be prepared to treat him accordingly. Unnecessary or vigorous palpation of his abdomen could cause his aorta to rupture and should be avoided. A rapid head- to-toe assessment is indicated for trauma patients with a significant mechanism of injury and unresponsive medical patients; this patient falls into neither of these categories. Auscultating bowel sounds in the fie ld is time-consuming, will yield little information, and simply delays transport.

A 50-year-old woman reports a rash and itching to her trunk and arms that began a day after she started taking penicillin. Her breath sounds are clear to auscultation, she denies shortness of breath, her oxygen saturation is 96%, and her BP is 132/74 mm Hg. Treatment should include: A) high-flow oxygen and transport. B) epinephrine and rapid transport. C) transport only with close monitoring. D) warm compresses to relieve the itching.

C) transport only with close monitoring. The patient is obviously experiencing an allergic reaction, but she is not in anaphylactic shock. Her breath sounds are clear to auscultation, she is breathing without difficulty, her oxygen saturation is above 94%, and her blood pressure is stable. She requires little more than transport to the hospital with continuous monitoring en route. Epinephrine is not indicated, nor is supplemental oxygen. Warm compresses would make her itching worse, not better. Consider applying cool packs for itch relief.

Following delivery of a newborn, the 21-year-old mother is experiencing mild vaginal bleeding. You note that her heart rate has increased from 90 to 120 beats/min and she is diaphoretic. Treatment should include: A) uterine massage for 5 minutes, and then transport. B) placing her in a left-lateral recumbent position and transport. C) treating for shock and performing uterine massage during transport. D) placing sterile pads into her vagina and treating for shock during transport.

C) treating for shock and performing uterine massage during transport. Blood loss of up to 500 mL within the first 24 hours after delivery is considered normal and usually is well tolerated by the mother. However, any bleeding, regardless of the severity, with accompanying signs of shock, must be treated at once. In this case, you should apply high-flow oxygen, treat for shock (keep her supine and cover her with warm blankets), and provide rapid transport to the hospital while massaging the uterine fundus en route. Placing the mother on her left side is appropriate before she delivers (and if she is not in shock) and prevents supine hypotensive syndrome. Dressings should never be packed into the vagina as this increases the risk for maternal infection.

A baseball player was struck in the center of the chest by a line drive and immediately collapsed. He is unresponsive, apneic, and pulseless. The EMT should recognize that the MOST likely cause of this patient's collapse was: A) rupture of the aorta. B) myocardial contusion. C) ventricular fibrillation. D) shearing of the vena cava.

C) ventricular fibrillation. Commotio cordis is a blunt chest injury caused by a sudden, direct blow to the chest (over the heart) during a critical phase of the person's heartbeat. This can cause immediate cardiac arrest from ventricular fibrillation, which can often be corrected with early defibrillation. Commotio cordis occurs most commonly during sports-related injuries, and has been reported after patients were struck with baseballs, softballs, bats, snowballs, fists, or kicks during kickboxing. The EMT should suspect commotio cordis if a patient presents with cardiac arrest following a sudden, direct blow to the chest. Myocardial contusion can also occur following blunt chest trauma; however, cardiac arrest with this type of injury is uncommon. Great vessel injury (ie, aortic rupture, shearing of the vena cavae) occurs more commonly following rapid deceleration, such as when the chest strikes the steering wheel during a high speed collision or when the patient falls from a significant height.

At what stage during pregnancy does the embryo become a fetus? A) 2 weeks B) 6 weeks C) 8 weeks D) 10 weeks

D) 10 weeks The fertilized egg travels to the uterus where, if implantation occurs, the fertilized egg becomes an embryo (the stage from O to 10 weeks after fertilization) and then a fetus (the stage from 10 weeks until delivery) and grows until the time of delivery at approximately 9 months (40 weeks) of gestation.

You are performing CPR on a 3-year-old child while your partner prepares the AED. What compression to ventilation ratio should you be providing? A) 3 to 1 B) 15 to 2 C) 30 to 1 D) 30 to 2

D) 30 to 2 A universal compression to ventilation ratio of 30:2 is used for all one-rescuer CPR (adult, child, and infant), with the exception of the newborn. A compression to ventilation ratio of 3:1 is used for newborns (one- and two-rescuer). Two-rescuer infant and child CPR is performed at a compression to ventilation ratio of 15:2. In this scenario, you are performing one-rescuer CPR as your partner prepares the AED; therefore, you should give 30 compressions and 2 breaths. However, when you and your partner resume CPR, give 15 compressions and 2 breaths.

A patient with a closed head injury opens his eyes in response to pain, is mumbling words that you cannot understand, and pushes your hand away when you apply a painful stimulus. His Glasgow Coma Scale (GCS) score is: A) 6 B) 7 C) 8 D) 9

D) 9 The Glasgow Coma Scale (GCS) is a valuable tool used when assessing patients with a neurologic injury. It assesses three parameters: eye opening, verbal response, and motor response. The minimum score on the GCS is 3 and the maximum score is 15. A patient who opens his or her eyes in response to pain would receive a score of 2. Mumbling speech, moaning, or incomprehensible words equate to a score of 2 for verbal response. Localization of a painful stimulus, such as pushing your hand away from the source of pain, equates to a score of 5. Therefore, the patient has a GCS score of 9. It is important to note that a patient's GCS score should be reassessed frequently. Review the entire GCS in your EMT text and commit it to memory!

Which of the following statements regarding the automated external defibrillator (AED) is correct? A) The AED should be applied to patients at risk for cardiac arrest. B) AEDs will analyze the patient's rhythm while CPR is in progress. C) The AED should not be used in patients with an implanted defibrilator. D) AEDs can safely be used in infants and children less than 8 years of age.

D) AEDs can safely be used in infants and children less than 8 years of age. According to the 2020 guidelines for CPR and Emergency Cardiac Care (ECC), the AED can safely be used in infants and children less than 8 years of age. Although a manual defibrillator is preferred in infants, an AED can be used. When using the AED in infants and children, you should use pediatric pads and a dose-attenuating system (energy reducer); however, if these are not available, adult AED pads should be used. The AED should be applied only to patients in cardiac arrest; if a patient is at risk for cardiac arrest, have the AED ready but not applied. The AED will not analyze the cardiac rhythm if the patient is moving (ie, CPR is in progress). AEDs can be used in patients with an automated implanted cardioverter/defibrillator (AICD) or implanted pacemaker; ensure that the pads are at least 1 inch away from the implanted device.

A 67-year-old woman presents with acute weakness and nausea. She is conscious and alert and her skin is cool and clammy. Her BP is 140/88 mm Hg, pulse is 70 beats/min and regular, and respirations are 16 breaths/min and unlabored. Her medical history includes hypertension and diabetes, and she advises that she has an implanted pacemaker. What should you suspect? A) Pacemaker malfunction B) Acute hypertensive crisis C) Infection with influenza D) Acute myocardial infarction

D) Acute myocardial infarction Remember this: elderly females with diabetes are the most likely to present with vague, unusual, or nonclassic symptoms of acute myocardial infarction. Acute weakness, nausea, and diaphoresis in this patient population should make you very suspicious for a cardiac event. Pacemaker malfunction is not causing her symptoms; if this was the cause, she would be bradycardic. Her blood pressure is somewhat elevated, but not so high as to cause acute hypertensive crisis. In addition, she is not experiencing any symptoms of acute hypertensive crisis, such as headache, ringing in the ears, or photophobia. Infection with the influenza virus (the flu) is unlikely; the flu does not present acutely, plus you would expect the patient's skin to be warm or hot from fever, not cool.

How can you help maximize cardiac output during CPR? A) Compress the chest at a rate of no more than 100/min B) Ventilate the patient through an advanced airway device C) Deliver rescue breaths until the chest expands widely D) Allow the chest to fully recoil in between compressions

D) Allow the chest to fully recoil in between compressions Cardiac output is the amount of blood ejected from the left ventricle per minute. Bearing in mind that even the best-performed CPR produces only a fraction of what the patient's cardiac output would otherwise be, there are several actions that you must take to help maximize this. Allowing the chest to fully recoil in between compressions will help draw blood back to the heart; if more blood returns to the heart, more blood can be pumped from the heart with chest compressions. Do not lean on the patient's chest between compressions. Delivering each rescue breath over a period of 1 second, just enough to produce visible chest rise, will also help maximize cardiac output. If ventilations are given too fast or too forcefully, intrathoracic pressure will increase, resulting in a decrease in the amount of blood that returns to the heart; as a result, cardiac output will decrease. Ventilations are delivered no differently if an advanced airway device (ie, ET tube, multilumen airway, supraglottic airway) has been inserted. Deliver chest compressions at a rate of 100 to 120 per minute to a depth of at least 2 inches (at least one-third the depth of the chest in infants and children). Chest compression depth in the adult should not exceed 2.4 inches; however, this is not possible to determine without a CPR device that provides immediate feedback.

A middle-aged woman with a history of hypertension and high cholesterol complains of chest discomfort. She asks you to take her to the hospital where her personal physician practices, which is 15 miles away. Her blood pressure is 130/70 mm Hg, pulse is 84 beats/min and regular, and respirations are 18 breaths/min and unlabored. Which of the following actions is clearly OT appropriate for this patient? A) Giving oxygen via nasal cannula B) Taking her to her choice hospital C) Contacting her physician via phone D) Allowing her to walk to the ambulance

D) Allowing her to walk to the ambulance You should NEVER allow a patient with a possible cardiac problem to walk to the ambulance. This causes exertion, which increases cardiac oxygen consumption and demand and could worsen his or her condition. Give the patient oxygen in a concentration sufficient to maintain his or her oxygen saturation equal to or greater than 94%. In general, you should transport patients to the hospital of their choice. However, transport to a closer hospital should be considered if you believe the patient is unstable or is at high risk for becoming unstable. If necessary, consult with the patient's physician via phone to determine if he or she thinks the patient should go to a closer hospital.

Which of the following describes pulseless electrical activity (PEA)? HIDE THIS MENU A) disorganized, chaotic quivering of the heart muscle that does not generate a pulse B) The presence of a palpable pulse in the absence of any electrical activity in the heart C) A rapid cardiac rhythm that does not produce a pulse, but responds to defibrillation D) Any organized cardiac rhythm, slow or fast, that does not produce a palpable pulse

D) Any organized cardiac rhythm, slow or fast, that does not produce a palpable pulse Pulseless electrical activity (PEA) is a condition in which the heart produces organized electrical activity (slow or fast), despite the absence of a palpable pulse. A disorganized, chaotic quivering of the heart muscle that does not generate a pulse is called ventricular fibrillation (V- Fib), and is treated with defibrillation. Defibrillation is not indicated for patients with PEA; it is indicated only for patients with V-Fib or pulseless ventricular tachycardia (V-Tach). If the AED gives a no shock message and the patient is still pulseless, he or she is either in asystole or PEA, neither of which is a shockable rhythm.

Following penetrating trauma to the abdomen, a 50-year-old woman has a large laceration with a loop of protruding bowel. How should you manage this injury? A) Carefully replace the bowel and apply an occlusive dressing. B) Apply a tight pressure dressing to control any external bleeding. C) Apply a dry, sterile dressing covered by an occlusive dressing. D) Apply a moist, sterile dressing covered by a dry, sterile dressing.

D) Apply a moist, sterile dressing covered by a dry, sterile dressing. Management of an open abdominal wound with an eviscerated bowel includes controlling any external bleeding, covering the exposed bowel with a moist, sterile dressing, and covering that with a dry, sterile dressing. Applying a dry dressing directly to the exposed bowel will cause the bowel to dry. You must never replace the exposed bowel into the abdominal cavity or apply pressure to the wound; doing so significantly increases the patient's risk for infection as well as further trauma.

A newborn has cyanosis to the chest and face and a heart rate of 90 beats/min. What should you do? A) Resuction the mouth B) Briskly dry off the infant C) Begin chest compressions D) Begin artificial ventilations

D) Begin artificial ventilations Central cyanosis (cyanosis to the head, face, and trunk) alone initially should be treated with blow-by oxygen; however, when it is accompanied by a heart rate that is less than 100 beats/min, artificial ventilations should be initiated and continued until the heart rate exceeds 100 beats/min. Newborn bradycardia is defined as a heart rate of less than 100 beats/min. Chest compressions are indicated if the newborn's heart rate falls below 60 beats/min, despite 30 seconds of adequate positive pressure ventilation. A newborn should be dried off thoroughly, regardless of its appearance at birth.

Which artery should you palpate when assessing for a pulse in an unresponsive 6-month-old infant? A) Radial B) Carotid C) Femoral D) Brachial

D) Brachial You should assess the brachia! pulse in infants younger than 1 year of age. The carotid or femoral pulse can be assessed in children older than 1 year of age. A carotid pulse is difficult to locate in infants because they have minimal space between their head and shoulders.

A construction worker spilled a bag of powdered chemical, covering both of his arms. He complains of intense pain. What should you do? A) Cover his arms with saline-soaked dressings B) Neutralize the chemical with a vinegar solution C) Irrigate his arms with copious amounts of water D) Brush the chemical away from the skin surface

D) Brush the chemical away from the skin surface When caring for a patient who was exposed to a powdered chemical, it is important to brush the chemical from the skin first, then irrigate the site with copious amounts of water. Irrigating the chemical without brushing it off first may cause further injury; also bear in mind that some dry chemicals may react violently with water. Do not try to neutralize the chemical with vinegar, baking powder, or other similar agents; doing so may only cause further injury.

In which of the following situations would external bleeding be the MOST difficult to control? A) Scalp laceration, BP of 130/70 mm Hg B) Jugular vein laceration, BP of 104/60 mm Hg C) Carotid artery laceration, BP of70/50 mm Hg D) Femoral artery laceration, BP of140/90 mm Hg

D) Femoral artery laceration, BP of140/90 mm Hg In general, the larger the size and type (eg, artery versus vein) of blood vessel injured, and the higher the patient's blood pressure, the more difficult the external bleeding will be to control. Of the choices listed, bleeding from a lacerated femoral artery (large, high-pressure vessel) in a patient with a blood pressure of 140/90 mm Hg (the highest BP listed) would be the most difficult to control. As a patient's blood pressure begins to fall, the driving force of blood in the arteries decreases and the bleeding becomes easier to control. Unfortunately, the patient is usually in decompensated shock at this point. The scalp contains many small blood vessels and tends to bleed heavily; however, direct pressure usually controls the bleeding with relative ease, regardless of the patient's blood pressure.

A 3-year-old child has a sudden onset of respiratory distress. The mother denies any recent illnesses or fever. What should you suspect? A) Croup B) Epiglottitis C) Lower respiratory infection D) Foreign body airway obstruction

D) Foreign body airway obstruction You should suspect a foreign body airway obstruction in any child who presents with an acute onset of respiratory distress in the absence of fever or any other symptoms that could be caused by an infection. Croup, epiglottitis, and lower airway infections (ie, bronchiolitis, bronchitis) commonly present with a fever. If the child is experiencing a mild airway obstruction, in which he or she is moving adequate air, has a normal level of consciousness, and has pink skin, do not attempt to relieve the airway obstruction; doing so may result in a severe airway obstruction. Offer oxygen and transport the child to the hospital without delay. If signs of a severe airway obstruction are present (ie, ineffective cough, decreased level of consciousness, cyanosis), you should perform abdominal thrusts until the object is expelled or the child becomes unresponsive. If the child becomes unresponsive, perform chest compressions.

A woman is 30 weeks pregnant. During your assessment, she tells you that she delivered a stillborn baby at 38 weeks and currently has no living children. How should you document her obstetric history? A) Gravida 1; para 1 B) Gravida 2; para 1 C) Gravida 1; para 2 D) Gravida 2; para 2

D) Gravida 2; para 2 Gravida refers to the number of times a woman has been pregnant, regardless of the length of the pregnancy. Para refers to the number of babies carried beyond 28 weeks, regardless of whether the baby was born dead or alive. The patient in this scenario has been pregnant twice (including her current pregnancy); therefore, she is a gravida 2. Since she has surpassed week 28 of her current pregnancy, and delivered her last baby (albeit, stillborn) past 28 weeks, she is a para 2.

During respiratory distress, a child is trying to keep the alveoli open by increasing pressure in the chest cavity. What sound would you expect to hear? A) Stridor B) Crackles C) Wheezing D) Grunting

D) Grunting A grunting sound heard during exhalation reflects the pediatric patient's attempt to keep the alveoli open by increasing pressure in the chest cavity; it is a sign of inadequate oxygenation. Stridor is a high- pitched sound heard during inhalation; it indicates inflammation or a partial obstruction of the upper airway. A crackling or bubbling sound hear during inhalation indicates fluid in the lungs. Wheezing is a high- or low- pitched whistling sound that is usually heard during exhalation; it indicates a partial lower airway obstruction caused by bronchospasm (ie, asthma, bronchiolitis).

A 56-year-old diabetic man is found unresponsive by his wife. She tells you that he ate breakfast this morning, but is unsure if he took his insulin. His respirations are rapid and shallow, his skin is cool and profusely diaphoretic, and his pulse is rapid and weak. Which of the following statements regarding this patient is correct? A) The fact that he ate breakfast makes hypoglycemia highly unlikely. B) He probably did not take his insulin and has a high blood glucose level. C) You should request an ALS unit so they can give the patient his insulin. D) He needs glucose as soon as possible because he is likely hypoglycemic.

D) He needs glucose as soon as possible because he is likely hypoglycemic. Based on the patient's presentation, you should suspect that he is in insulin shock (hypoglycemic crisis) and requires glucose as soon as possible. Cool, clammy (diaphoretic) skin; rapid, shallow respirations; and a rapid, weak pulse are classic signs of insulin shock. Diabetic coma (hyperglycemic ketoacidosis) typically presents with deep, rapid respirations (Kussmaul respirations); warm, dry skin; and tachycardia. If a diabetic patient has an altered mental status or is unresponsive, you should assume that he or she is hypoglycemic unless there is a specific information to the contrary (ie, failure to take prescribed insulin, hypoglycemia ruled out by glucometer). Even if the patient is in diabetic coma and needs insulin, it is rarely given in the field by ALS personnel. The EMT can quickly rule hypoglycemia in or out by assessing the patient's blood glucose level with a glucometer.

Which of the following conditions would place the patient at greatest risk for complications after receiving epinephrine? A) Asthma B) Bradycardia C) Hypovolemia D) Heart disease

D) Heart disease While there are no contraindications for epinephrine in a life-threatening situation such as anaphylaxis, the EMT should be aware that epinephrine could cause complications in patients with heart disease or acute coronary syndrome. Epinephrine increases the heart rate, which increases cardiac oxygen demand and consumption. This could exacerbate underlying cardiac disease. The EMT should therefore consult with medical control when administering epinephrine to such patients. In addition to increasing the heart rate and causing vasoconstriction, epinephrine also dilates the bronchioles; this would be a desirable effect in patients with asthma. Hypovolemia should be treated initially with IV fluids; however, epinephrine (or other similar drugs) may be required if IV fluids alone do not improve perfusion.

In which of the following situations would nitroglycerin MOST likely be administered? A) Recent use of Cialis B) Systolic BP of 90 mm Hg C) The presence of a head injury D) History of cardiac bypass surgery

D) History of cardiac bypass surgery Nitroglycerin (NTG) is a vasodilator drug used to relieve chest pain in patients with cardiac compromise by dilating the coronary arteries and improving blood flow to the heart. Because of its vasodilator effects, it should not be given to patients who have a systolic BP less than 100 mm Hg or to patients who have recently (within the past 24 to 48 hours) taken erectile dysfunction (ED) drugs (eg, sildenafil [Viagra], vardenafil [Levitra], tadalafil [Cialis]). ED drugs also cause vasodilation and may cause significant hypotension if given together with NTG. You should also avoid NTG in patients with a head injury; dilation of the cerebral blood vessels may worsen intracranial pressure caused by the head injury. By itself, a history of cardiac bypass surgery does not contraindicate the use of NTG.

After several months of using an illegal substance, the patient develops a tolerance for it. What does this mean? A) The drug achieves its effect at the same dose every time it is used B) An overwhelming need to use the drug causes more frequent dosing C) The drug is used more frequently in order to achieve a greater effect D) Increasing amounts of the drug are needed to achieve the same effect

D) Increasing amounts of the drug are needed to achieve the same effect Over time, a person who routinely misuses a substance may need increasing amounts of it to achieve the same effect. This is called developing a tolerance to the substance. A person with an addiction has an overwhelming desire or need to continue using the substance at whatever the cost, with a tendency to increase the dose. This does not only happen with the classic drugs of abuse, such as heroin or cocaine. Almost any substance can be abused, including laxatives, nasal decongestants, vitamins, and food.

When assessing a patient who complains of chest pain, which of the fo llowing questions would you ask to assess the "R" in OPQRST? A) Did the pain begin suddenly or gradually? B) What were you doing when the pain began? C) Is there anything that makes the pain worse? D) Is the pain in one place or does it move around?

D) Is the pain in one place or does it move around? The "R" in OPQRST stands for radiation or referred pain. An appropriate way to determine whether the pain radiates is to ask the patient if the pain remains in one place or if it moves around. When determining if the patient has referred pain, ask the patient if he or she hurts somewhere other than his or her chest. If you use the term "radiating pain," chances are the patient will not understand what you are asking.

A 71-year-old woman presents with an acute onset of confusion, slurred speech, and left-sided weakness. She is conscious and her airway is patent. Her BP is 180/94 mm Hg, her pulse is 70 beats/min and irregular, her respirations are 14 breaths/min and unlabored, and her oxygen saturation is 97% on room air. What should you do? A) Administer up to 325 mg of chewable aspirin. B) Position her supine, and assist her ventilations. C) Give oxygen via nasal cannula at 4 to 6 L/min. D) Keep her sitting up, and transport without delay.

D) Keep her sitting up, and transport without delay. The patient's presentation is obvious for a stroke. Second ONLY to addressing problems with airway, breathing, and circulation, the most important intervention is to transport the patient to a stroke center, where fibrinolytic medications can be given or other interventions can be performed. Keep the patient sitting up in order to protect her airway, and transport without delay. The patient in this scenario is breathing adequately; therefore, assisted ventilation is not indicated. Although her BP is elevated (common with a stroke), this is not treated by the EMT in the field . The patient's oxygen saturation does not indicate hypoxemia and she is not experiencing respiratory distress; therefore, supplemental oxygen is not indicated. Do NOT administer aspirin to a patient with stroke-like symptoms; if the cause of the stroke is hemorrhage in the brain, aspirin could worsen the situation.

In which of the following circumstances would external bleeding be the LEAST difficult to control? A) Lacerated brachia! artery; BP of140/90 mm Hg B) Lacerated jugular vein; BP of 100 /60 mm Hg C) Lacerated carotid artery; BP of 90/50 mm Hg D) Lacerated femoral vein; BP of70/40 mm Hg

D) Lacerated femoral vein; BP of70/40 mm Hg It is generally less difficult to control external bleeding from a lacerated vein rather than an artery. Unlike arteries, veins are under low pressure. Furthermore, the presence of a low blood pressure (hypotension), which causes less pressure against the vascular wall, would make external bleeding that much easier to control. Remember, if direct pressure does not immediately control severe external bleeding from an extremity, apply a proximal tourniquet.

An elderly woman complains of pain to her right groin area after she fell. Her right leg is shortened and externally rotated. Which of the following would be the MOST effective way of stabilizing her injury? A) Bind her legs together and position her on her left side B) Apply a long board splint and secure her to the stretcher C) Apply a traction splint and secure her to a long backboard D) Place her on a scoop stretcher and pad her hip with pillows

D) Place her on a scoop stretcher and pad her hip with pillows The patient likely has a proximal femur fracture , as evidenced by the pain in her groin area and the shortened and externally rotated leg. Of the options listed, the most practical and comfortable for the patient would be to place her onto a scoop stretcher and pad her hip area with pillows. Binding her legs together would more of an effective splint that a long board splint. Keep her supine; placing her onto her side would likely increase her pain. Traction splints are used on patients with mid-shaft femur fractures; they should generally be avoided in patients with proximal or distal femur fractures. The long backboard is perhaps the most uncomfortable device to place a human being on, especially an older patient!

Which of the following assessment findings would indicate primary central nervous system dysfunction in an infant or small child? A) Nasal flaring B) Easily distracted C) Pale extremities D) Poor muscle tone

D) Poor muscle tone Of the findings listed, poor muscle tone (as a primary finding) would indicate dysfunction of the central nervous system. Children should be easily distracted, so an inability to distract the child would be abnormal. Nasal flaring indicates a primary respiratory problem and signals increased work of breathing. Pallor indicates poor peripheral blood flow; therefore, it would indicate a perfusion problem. When assessing an infant or child, the pediatric assessment triangle (PAT) will enable you to determine, by visual inspection, if the problem is neurologic, respiratory or circulatory in origin.

Which of the following clinical signs would you MOST likely observe in an infant or small child with a tension pneumothorax? A) Tracheal deviation B) An irregular pulse C) Distended jugular veins D) Progressive respiratory distress

D) Progressive respiratory distress Tension pneumothorax can be a rapidly fatal condition unless promptly identified and treated. Progressive respiratory distress following a chest injury is a critical finding in any patient and should make you suspect increasing pleural tension with resultant collapsing of the lung. Due to their short neck, the jugular veins can be very difficult, if not impossible, to assess in infants and small children. Tracheal deviation, an extremely late sign, occurs on the lower part of the trachea, near the carina; without a chest x-ray, one cannot appreciate this finding. An irregular pulse is not a common finding in patients with a tension pneumothorax, regardless of age.

Which of the following complications would MOST likely occur if a patient with end-stage renal disease missed several dialysis treatments? A) Severe dehydration B) Potassium depletion C) Bacterial infection D) Pulmonary edema

D) Pulmonary edema Dialysis removes waste products from the body when the patient's kidneys are unable to do so on their own. If dialysis is not performed, everything that should have been removed remains in the body. This can lead to myriad problems for the patient. Potassium levels can rise to dangerous levels (hyperkalemia), and the patient can develop volume overload; this can result in pulmonary edema. Dehydration would be more likely to occur with aggressive dialysis, not missing a treatment. Patients receiving dialysis are at increased risk for infection, not because of the dialysis, but because of the underlying renal disease.

A 40-year-old man reports difficulty swallowing and is vomiting bright red blood. His wife advises that he has diabetes and liver fai lure from years of alcohol abuse. What should you suspect? A) Acute pancreatitis B) Ruptured aortic aneurysm C) Severe gastric ulcer disease D) Ruptured esophageal varices

D) Ruptured esophageal varices Based on the patient's presentation and medical history, you should suspect ruptured esophageal varices. Esophageal varices occur when the amount of pressure within the blood vessels surrounding the esophagus increases, frequently as a result of liver failure. The esophageal blood vessels eventually drain their blood into the liver. If the liver is damaged and blood cannot flow through it easily, blood begins to back up into these portal vessels, dilating the vessels and causing the capillary network of the esophagus to begin leaking. If pressure continues to build, the vessel walls may fail, causing massive upper gastrointestinal bleeding and, quickly afterward, hematemesis (vomiting up blood). Severe gastric ulcer disease can cause hematemesis; however, difficulty swallowing (dysphagia) is not common. Neither acute pancreatitis nor ruptured aortic aneurysm commonly present with hematemesis.

Which of the following is an example of a primary blast injury? A) Depressed skull fracture B) Spinal injury with paralysis C) Stick impaled in the abdomen D) Ruptured tympanic membrane

D) Ruptured tympanic membrane Primary blast injuries are a direct result of the pressure wave that occurs during an explosion. Hollow organs are the most susceptible to the primary blast wave, and ruptured tympanic membranes (eardrums) are a common injury. Secondary blast injuries occur when shrapnel and other debris are propelled away from the explosion; impalement injuries occur during this phase. Blunt traumatic injuries (ie, skull fracture, spinal injury) occur during the tertiary blast phase, when the person is propelled away from the explosion and strikes a solid object.

When you arrive at a residence for a man who is "not acting right," you enter the house and find him sitting on his couch. Which of the following findings would be MOST indicative of an altered mental status? A) Odor of alcohol B) Closed eyes C) Tired appearance D) Slurred speech

D) Slurred speech Often, an altered mental status can be difficult to assess, especially if you do not know how the patient normally acts. However, there are key findings that should increase your index of suspicion. An abnormal speech pattern, such as slurring or incoherent words, can be the result of a diabetic problem, alcohol intoxication, or drug ingestion. All of these can cause an altered mental status. The odor of alcohol suggests intoxication as a potential cause of his problem, but cannot be quantified. Just because the patient's eyes are closed or he has a tired appearance, does not necessarily indicate that he has an altered mental status.

A 62-year-old man with a history of coronary artery disease began experiencing chest pain and nausea while on his daily jog. After sitting down and resting for 5 minutes, his pain and nausea resolve. Which of the following BEST describes this patient's condition? A) Musculoskeletal chest pain B) Acute myocardial infarction C) Unstable angina pectoris D) Stable angina pectoris

D) Stable angina pectoris Angina pectoris is the principle symptom of coronary artery disease (CAD). It occurs when the heart's demand for oxygen exceeds its supply, usually during periods of physical or emotional stress when the heart is working hard. When the increased oxygen demand goes away, the pain typically resolves. Some patients wi th angina may also experience shortness of breath, nausea, or diaphoresis. Angina is classified as being "stable" or "unstable." Stable angina is characterized by cardiac-related chest pain that resolves with rest and/or nitroglycerin. Unstable angina is characterized by cardiac-related chest pain that occurs in response to progressively less physical exertion (for example, during rest). Unstable angina is also referred to as pre-infarction angina because it indicates a more severe degree of CAD and can lead to acute myocardial infarction if untreated. In contrast to stable angina, unstable angina typically does not resolve with rest and/or nitroglycerin. Given the patient's medical history, his chest pain should not be assumed to have simply been of a musculoskeletal origin.

A 4-year-old child has had several generalized seizures over the past 20 minutes, but never woke up in between the seizures. The child's skin is hot and flushed. What should you suspect? A) febrile seizure B) A focal motor seizure C) An absence seizure D) Status epilepticus

D) Status epilepticus Status epilepticus is defined as a prolonged (greater than 20 minutes) seizure or multiple seizures without a return of consciousness in between seizures. A febrile seizure is caused by an abrupt rise in body temperature, usually due to a non-life-threatening infection (eg, middle ear infection). Most febrile seizures last less than 5 minutes, have resolved by the time EMS arrives at the scene, and are not followed by a postictal period. The child in this scenario, although febrile (hot, flushed skin), did not experience a seizure caused by fever alone; you should suspect other causes of fever and seizures, such as meningitis. Absence seizures are characterized by a blank stare and an absence of tonic-clonic motor activity. Like febrile seizures, absence seizures are usually of short duration and are not followed by a postictal period. A focal motor seizure is isolated to one part of the body, such as an extremity, but can progress to a generalized tonic clonic seizure.

Which of the following statements regarding one-rescuer CPR is correct? A) You should assess the patient for a pulse after 3 cycles of CPR. B) A compression to ventilation ratio of 15:2 should be delivered. C) Ventilations should be delivered over a period of 2 to 3 seconds. D) The chest should be allowed to fully recoil after each compression.

D) The chest should be allowed to fully recoil after each compression. When performing CPR on any patient, you should allow the chest to fully recoil after each compression; do not lean on the chest between compressions. Incomplete chest recoil causes increased intrathoracic pressure, which may impair blood return to the heart. Assess the patient's pulse after every 5 cycles (about 2 minutes) of CPR; take no longer than 5 to 10 seconds to do this. A compression to ventilation ratio of 30:2 should be used during all adult and onerescuer CPR (adult, child, and infant), except for newborns. A compression to ventilation ratio of 15:2 is used during two-rescuer infant and child CPR. Ventilations should be delivered over a period of 1 second each, just enough to produce visible chest rise.

In which of the following situations should the lone rescuer call 9-1-1 and retrieve an AED before initiating CPR on a pulseless and apneic child? A) The child is between 1 and 8 years of age. B) The child's past medical history is unknown. C) An opioid overdose is suspected or confirmed. D) The child's collapse was witnessed and sudden.

D) The child's collapse was witnessed and sudden. If you are off duty or otherwise alone and witness the sudden collapse of a child (1 year of age to the onset of puberty [12 to 14 years of age]) who is pulseless and apneic, you should immediately call 9-1-1 and retrieve an AED,just as you would for an adult. The sudden collapse of a child is consistent with a primary cardiac problem, not respiratory failure (the most common cause of cardiac arrest in infants and children). Therefore, early defibrillation is crucial in this patient population. Children with respiratory failure typically get progressively sicker and then develop cardiac arrest; they usually do not collapse suddenly. If the suspected cause of the cardiac arrest is respiratory failure, or if the child's collapse was not witnessed and sudden, you should perform CPR for 2 minutes before calling 9-1-1 and retrieving the AED. In these cases, it should be assumed that respiratory failure is the underlying cause. Cardiac arrest secondary to an opioid overdose is the result of hypoxia; in these cases, CPR should be performed for 2 minutes before calling 9-1-1 and retrieving the AED. The child's medical history is not a factor when determining whether or not to call 9-1-1 first or perform CPR first.

A 32-year-old woman who is 36 weeks' pregnant reports a sudden onset of severe abdominal pain, which she describes as a tearing sensation, and vaginal bleeding. Her medical history includes hypertension and she admits to using cocaine. Which of the following should you suspect? A) The umbilical cord is wrapped around the baby's neck. B) The placenta has implanted over the cervical opening. C) Excessive fetal movement has torn the amniotic sac. D) The placenta has detached from the wall of the uterus.

D) The placenta has detached from the wall of the uterus. Given the patient's presentation and medical history, you should suspect that she is experiencing abruptio placenta. This condition occurs when the placenta acutely detaches from the uterine wall. Signs and symptoms include acute, severe abdominal pain that is often described as a ripping or tearing sensation; vaginal bleeding; and signs of shock. Risk factors include hypertension, abdominal trauma, and cocaine use, among others. Cocaine is a potent vasoconstrictor, and frequent use can cause vascular injury. In pregnant patients, this vascular injury could cause premature placental separation. Placenta previa occurs when part or all of the placenta implants over the cervical opening; vaginal bleeding is common, but abdominal pain is not. Nuchal cord (cord wrapped around the baby's neck) is discovered during delivery; vaginal bleeding and abdominal pain do not occur with this condition. A ruptured or torn amniotic sac would present with clear vaginal discharge; vaginal bleeding and abdominal pain do not occur with this condition.

You receive a call for a 4-month-old infant who suddenly became limp, cyanotic, and apneic. Upon EMS arrival, the mother tells you that she flicked his feet and he woke up. He is breathing adequately. He has good muscle tone and his skin is warm and pink. What should you do? A) Administer oxygen via nonrebreathing mask. B) Allow a parent to transport him to the hospital. C) Contact the infant's pediatrician to seek advice. D) Transport to the hospital and monitor him en route.

D) Transport to the hospital and monitor him en route. Infants who have cyanosis and apnea and are unresponsive when found by a parent sometimes resume normal breathing and normal color with stimulation. This is called an apparent life-threatening event (ALTE), also known as a brief resolved unexplained event (BRUE). In addition to cyanosis, the classic ALTE is characterized by a distinct change in muscle tone (limpness) and choking or gasping. After the ALTE, you may find an infant who appears healthy and shows no signs of illness or distress, such as the infant in this scenario. Even though the infant appears normal now, something happened acutely to cause the initial problem. Infants with this type of event should be transported by EMS to the emergency department for evaluation. Oxygen is not indicated for this particular child; he is breathing normally and has no signs of hypoxemia.

After drying, warming, and suctioning a newborn's mouth and nose, assessment reveals central cyanosis, a weak cry, and a heart rate of 60 beats/min. What should you do? A) Clamp and cut the umbilical cord and transport at once. B) Begin chest compressions and reassess after 30 seconds. C) Resuction the mouth and nose and reassess the heart rate. D) Ventilate with a bag-mask device at 40 to 60 breaths/min.

D) Ventilate with a bag-mask device at 40 to 60 breaths/min. The initial treatment for a cyanotic and bradycardic newborn is positive pressure ventilation (PPV). Ventilate the newborn with a bag-mask device at a rate of 40 to 60 breaths/min and then reassess the heart rate after 30 seconds. If the heart rate is below 60 beats/min after 30 seconds of adequate PPV, you should begin chest compressions. The umbilical cord should not be clamped and cut until it stops pulsating and the newborn is breathing adequately. Unnecessary suctioning can worsen hypoxia and bradycardia and should be avoided.

A 33-year-old man struck a parked car with his motorcycle and was thrown from the motorcycle. He was not wearing a helmet. He is unresponsive and has a depressed area to his forehead, bilaterally deformed femurs, and widespread abrasions with capillary bleeding. Which of the following statements regarding this patient is false? A) You should suspect that the patient has a skull fracture and increased intracranial pressure. B) Femur fractures are a common injury when a rider is thrown from his or her motorcycle. C) Internal hemorrhage cannot be controlled in the field and requires prompt surgical intervention. D) You must stop the bleeding from his abrasions immediately or the patient will die from hypovolemic shock.

D) You must stop the bleeding from his abrasions immediately or the patient will die from hypovolemic shock. The patient's abrasions (road rash) and capillary bleeding are the least of his problems. Capillary bleeding, blood that oozes from the capillary beds, is the least severe type of external bleeding and will not kill your patient. Wasting time at the scene to cover his abrasions, however, will delay definitive care at a trauma center; this may kill him! The patient likely has a depressed skull fracture, and the fact that he is unresponsive indicates a traumatic brain injury with increased intracranial pressure. When a rider is thrown from his or her motorcycle, the femurs typically strike the handlebars, resulting in unilateral or bilateral fractures . You cannot control internal hemorrhage in the field, regardless of your level of training. Internal bleeding requires surgical intervention; therefore, you must transport the patient without delay.

While assessing a patient with chest pain, you note that his pulse is irregular. This indicates: A) acute myocardial infarction or angina pectoris. B) a dysfunction in the left side of the patient's heart C) high blood pressure that is increasing cardiac workload. D) abnormalities in the heart's electrical conduction system.

D) abnormalities in the heart's electrical conduction system. An irregular pulse indicates abnormalities in the electrical conduction system of the heart. The electrical conduction system, beginning with the sinoatrial node as the primary pacemaker, is responsible for initiating the electrical impulses that stimulate the myocardium to contra t. An irregular pulse could indicate a potentially lethal dysrhythmia that could result in cardiac arrest. You should document an irregular pulse and report this important finding to the emergency department.

Which of the following is an expected finding in patients who have a left ventricular assist device (LVAD) with a continuous flow pump? A) cyanotic skin. B) hypertension. C) peripheral edema. D) absence of pulses.

D) absence of pulses. A left ventricular assist device (LVAD) is used to enhance left ventricular pumping. LVADs are commonly implanted in patients with severe heart failure, those who are awaiting a heart transplant, or those who need a temporary boost following acute myocardial infarction. Several types of LVADs exist; the most common ones have an internal pump unit and an external battery pack. These pumps may be pulsatile, meaning they pump in pulsations just like the natural heart, or they may be continuous, in which case the patient will not have any palpable pulses. If pulses are not palpable, then a blood pressure will not be obtainable. You would not expect to encounter cyanosis in any patient with an LVAD, whether the pump is pulsatile or continuous; if you do, an oxygenation problem exists. Peripheral edema is also not expected in patients with an LVAD.

You receive a call to a restaurant where a 34-year-old man is experiencing shortness of breath. When you arrive, you immediately note that the man has urticaria on his face and arms. He is conscious, but restless, and is in obvious respiratory distress. You should: A) ask the patient if he has an epinephrine auto-injector. B) remove the patient's shirt to inspect his chest for urticaria. C) obtain vital signs and a SAMPLE history. D) administer supplemental oxygen at 12 to 15 L/min.

D) administer supplemental oxygen at 12 to 15 L/min. Initial management of a patient with a severe allergic reaction is to ensure a patent airway and administer supplemental oxygen. Positive pressure ventilation may be required if the patient is breathing inadequately. After ensuring a patent airway and adequate ventilation and oxygenation, you should inquirewhether the patient has a prescribed epinephrine auto- injector. If so, you should contact medical control and obtain permission (if required by local protocol) to assist the patient with the auto-injector.

Firefighters have rescued a man from his burning house. He is conscious and alert, but is experiencing significant respiratory distress. He has a brassy cough and singed nasal hairs. The MOST immediate threat to this patient's life is: A) hypothermia. B) severe burns. C) severe infection. D) airway swelling.

D) airway swelling. Because of the patient's signs and symptoms, your most immediate concern should be the potential for swelling and closure of the upper airway; be prepared to assist the patient's ventilations. Signs of upper airway burns include respiratory distress, singed facial and/or nasal hairs, a brassy cough, difficulty breathing, and coughing up sooty sputum. Infection, the burns themselves, and hypothermia should concern you; however, airway problems pose the most immediate life threat.

The signs and symptoms of an allergic reaction to a bee sting are caused by: A) failure of the immune system to produce antibodies that render the bee's venom inactive. B) an inability of the immune system to recognize the bee's venom as being a foreign substance. C) the direct toxic effects of the bee's venom on the cardiovascular and respiratory systems. D) an exaggerated immune system reaction in which chemicals are released in response to the venom.

D) an exaggerated immune system reaction in which chemicals are released in response to the venom. An allergic reaction, an exaggerated immune response to any substance, is not caused directly by an outside stimulus, such as venom released during a bite or sting. Rather, it is a reaction that occurs when the immune system recognizes the stimulus as being foreign; in response, the immune system releases chemicals (histamines and leukotrienes) to combat the stimulus. An allergic reaction is not caused by a failure of the immune system to produce antibodies; rather, it occurs because the immune system produces them. A bee's venom does not have a direct toxic effect on the body; the chemicals released in response to the venom, however, can cause life-threatening conditions such as respiratory failure and shock.

A behavioral crisis is MOST accurately defined as: A) an acute psychiatric emergency characterized by violent behavior, mood swings, and a loss of connection to reality. B) a chronic mental health problem in which the patient experiences frequent thoughts of suicide or other self-destructive behavior. C) a persistent feeling of sadness, despair, or hopelessness that incapacitates the patient and prevents him or her from interacting socially. D) any reaction to an event that interferes with the activities of daily living or has become unacceptable to the patient, family, or community.

D) any reaction to an event that interferes with the activities of daily living or has become unacceptable to the patient, family, or community. The definition of a behavioral crisis or emergency is any reaction to an event that interferes with the activities of daily living (eg, bathing, dressing, eating) or has become unacceptable to the patient, his or her family, or the general community. Some patients react to an event with violent behavior or suicidal thoughts; others react with depression. Regardless of the reaction, it significantly interferes with the patient's life or is unacceptable to his or her family and/or the community.

Despite direct pressure, a large laceration continues to spurt large amounts of bright red blood. You should: A) elevate the extremity and apply a tight pressure dressing. B) apply pressure to the pulse point that is proximal to the injury. C) place additional dressings on the wound until the bleeding stops. D) apply a tourniquet proximal to the injury until the bleeding stops.

D) apply a tourniquet proximal to the injury until the bleeding stops. You must control any and all external bleeding as soon as possible. In the case of arterial bleeding (ie, bright red blood is spurting from the wound), the patient will bleed to death if immediate action is not taken. In most cases, direct pressure will effectively control external bleeding. However, if the wound continues to bleed profusely despite direct pressure, you should apply a tourniquet proximal to the injury and tighten it until the bleeding stops. Packing additional dressings on a severe external hemorrhage will simply cause the patient to continue to bleed externally into the dressings. Locating and applying adequate pressure to a proximal arterial pressure point is often difficult and time-consuming.

A woman has a painful, deformed humerus after she fell. The injured arm is cool and pale, and a radial pulse cannot be palpated. The EMT should: A) allow her to stabilize the arm on her own and transport. B) splint the arm in the position found and transport at once. C) realign the arm to its normal anatomic position and splint it. D) apply gentle traction in order to reestablish distal circulation.

D) apply gentle traction in order to reestablish distal circulation. When treating an orthopaedic injury in which distal circulation is compromised, you should apply gentle traction in an effort to restore distal circulation. Bear in mind, however, that this practice does NOT apply to injuries involvingjoints. Once distal circulation is restored, splint the injury appropriately and transport. lfyou are unable to restore distal circulation, contact medical control; depending on your transport time, he or she may request you to make another effort to restore distal circulation. Fractures, with or without distal circulation, should be splinted in the most comfortable position for the patient; attempting to realign a fractured extremity to its normal anatomic position would cause the patient unnecessary pain.

A man has a large laceration to his right calf after the chainsaw he was using slipped. The wound extends down to the muscle and dark red blood is flowing heavily from the wound. The EMT should: A) apply a tourniquet proximal to the wound. B) position the patient supine and elevate his leg 12 inches. C) check distal circulation and then splint the entire extremity. D) apply manual pressure to the wound with a sterile dressing.

D) apply manual pressure to the wound with a sterile dressing. Immediate treatment for external hemorrhage involves applying direct pressure to the wound. In many cases, this will be all that is required. However, if the wound continues to bleed despite the use of direct pressure, a proximal tourniquet should be applied. The patient with hemorrhage and shock should be positioned supine; do not elevate his or her legs. If direct pressure alone controls the bleeding, it would not be unreasonable to splint the affected extremity; doing so minimizes movement of the extremity and may further assist in bleeding control by facilitating hemostasis.

The two MOST important steps in treating a patient with a contact poisoning are: A) determining when the exposure occurred and irrigating the patient's entire body with copious amounts of water. B) donning a pair of gloves and performing a physical examination to determine if a significant exposure occurred. C) immediately washing any dry chemicals off of the patient's skin and removing his or her clothing as soon as possible. D) avoiding self-contamination and removing the irritating or corrosive substance from the patient as rapidly as possible.

D) avoiding self-contamination and removing the irritating or corrosive substance from the patient as rapidly as possible. When caring for a patient with a contact (skin surface) exposure to a poison, the two most important steps in management are avoiding contaminating yourself and removing the substance from the patient's skin as rapidly as possible. The level of personal protective equipment (PPE) you use depends on the type of chemical the patient was exposed to. After ensuring your own safety, remove all of the patient's clothing that has been contaminated, thoroughly brush off any dry chemicals, and then flush the skin with water. Always brush dry chemicals off the skin before irrigating with water; failing to do so may increase the amount of damage caused by the chemical. A physical exam of the patient should be performed only after he or she has been properly decontaminated.

A prolapsed umbilical cord is dangerous because the: A) cord might pull the placenta from the uterine wall during delivery. B) mother may die of hypoxia due to compromised placental blood flow. C) cord may be wrapped around the baby's neck, causing strangulation. D) baby's head may compress the cord, cutting off its supply of oxygen.

D) baby's head may compress the cord, cutting off its supply of oxygen. A prolapsed umbilical cord, a condition in which a portion of the umbilical cord delivers before the baby, is a dangerous condition; the baby's head may compress the cord, cutting off its own supply of oxygen. Therefore, when a prolapsed umbilical cord is discovered, it is important to take immediate action. Place the mother in a position in which her hips are elevated. It may be necessary to insert your gloved fingers into the vagina and lift the baby's head off of the cord. A nuchal cord occurs when the umbilical cord is wrapped around the baby's neck; it is relatively common and is usually easily treated by simply sliding the cord from around the baby's neck. A nuchal cord and a prolapsed umbilical cord usually do not occur at the same time.

A patient with a spinal injury may still be able to use his or her diaphragm to breathe, but would lose control of the intercostal muscles, if the spinal cord is injured: A) above the C3 level. B) between C1 and C2. C) above the C5 level D) below the C5 level.

D) below the C5 level. The nerves that supply the diaphragm (the phrenic nerves) exit the spinal cord at C3, C4, and C5. A patient whose spinal cord is injured below the C5 level will lose the ability to move his or her intercostal muscles (the muscles in between the ribs), but the diaphragm will still function. The patient may still be able to breathe because the phrenic nerves remain intact. Patients with spinal cord injuries at C3 or above often lose their ability to breathe entirely. Remember this: C3, 4, and 5 keep the diaphragm alive.

Immediately upon delivery of a newborn's head, you should: A) dry the face. B) cover the eyes. C) suction the nose. D) check for a nuchal cord.

D) check for a nuchal cord. As soon as the newborn's head has delivered, you should check for the presence of a nuchal cord (umbilical cord wrapped around the neck). Then you should suction the mouth and nose (mouth before the nose). As the infant is forced through the birth canal, the thoracic cavity is squeezed, which causes the infant to expel amniotic fluid from the lungs. If this fluid is not thoroughly suctioned, it can be aspirated, resulting in inadequate ventilation and hypoxia.

In the patient with diabetes, insulin shock typically presents with: A) dry skin and a slow onset. B) dry skin and a rapid onset. C) clammy skin and a slow onset. D) clammy skin and a rapid onset.

D) clammy skin and a rapid onset. In the patient with diabetes, insulin shock (hypoglycemic crisis) presents with cool, clammy skin and a rapid onset. The brain is critically dependent on glucose and responds quickly when the body is in short supply. Diabetic coma (hyperglycemic crisis) typically presents with warm, dry skin and a slow onset, sometimes occurring over a period of days.

A man was cut on the left side of the neck and is bleeding heavily from the wound. His airway is patent and his breathing is adequate. You should immediately: A) apply high-flow oxygen via a nonrebreathing mask at 15 L/min. B) apply a tight pressure dressing and secure it in place with tape. C) perform a head-to-toe assessment to find and treat other injuries. D) cover the wound with an occlusive dressing and apply direct pressure.

D) cover the wound with an occlusive dressing and apply direct pressure. Neck lacerations are extremely dangerous and can result in severe bleeding and shock, air embolism, or both. If a jugular vein is lacerated, air can be sucked into the wound, enter the circulatory system, and cause a pulmonary embolism. You should immediately apply an occlusive dressing to the wound (prevents entrainment of air), place a bulky dressing over the occlusive dressing, and apply direct pressure. Your patient has a patent airway and is breathing adequately; although high-flow oxygen is important and should be given as soon as possible, it does not take priority over control of life-threatening external hemorrhage. After treating all airway, breathing, and circulation problems, perform a head-to-toe assessment (if indicated) and prepare for rapid transport.

Damaged small blood vessels beneath the skin following blunt trauma cause: A) mottling. B) cyanosis. C) hematoma. D) ecchymosis.

D) ecchymosis. When small blood vessels beneath the skin are damaged, blood seeps into the soft tissues. This manifests as a bruise, also referred to as ecchymosis. A hematoma develops when larger blood vessels are ruptured and the internal bleeding forms a noticeable lump. Cyanosis is a blue or purple discoloration of the skin and signifies a low content of oxygen in the blood. Mottling occurs when the skin takes on a blotched, purple appearance and is a sign of shock (hypoperfusion).

Prevention of cardiac arrest in infants and small children should focus primarily on: A) keeping the child warm. B) avoiding upsetting the child. C) providing immediate transport. D) ensuring adequate ventilation.

D) ensuring adequate ventilation. The most common cause of cardiac arrest in infants and children is failure of the respiratory system. Their hearts generally are healthy, and they rarely go into ventricular fibrillation (V- Fib). The key to preventing cardiac arrest in the majority of infants and children is to ensure adequate ventilation and oxygenation.

Emergency care for a 68-year-old man with partial- and full-thickness bums to his chest and upper extremities includes all of the following, EXCEPT: A) preparing to assist the patient's ventilations. B) covering the bums with dry, sterile dressings. C) avoiding the use of bum ointments or antiseptics. D) flushing the bums with cool water for 10 minutes.

D) flushing the bums with cool water for 10 minutes. Unless the patient is on fire, do not apply water to a full-thickness (third-degree) burn, especially if the patient is already prone to hypothermia and infection (ie, older adults, small children). Cover the burns with dry, sterile dressings or a sterile burn sheet. The use of burn creams, ointments, or antiseptics should be avoided; these increase the risk of infection and will only need to be removed at the hospital. Apply high-flow oxygen, treat any associated injuries, and rapidly transport the patient. If the patient is breathing inadequately (eg, fast or slow rate, shallow breathing [reduced tidal volume]), assist ventilations with a bag-mask device.

When caring for a woman who was sexually assaulted, the EMT should: A) ask the patient if she wishes to change her clothes. B) obtain a concise, detailed account of what happened. C) place any articles of her clothing in a clean plastic bag. D) focus any assessments on life-threatening conditions.

D) focus any assessments on life-threatening conditions. As with any patient, your immediate assessment should focus on finding and treating immediately life-threatening injuries or conditions. Patients who were sexually assaulted may have evidence on (or in) them that could be crucial to bringing the perpetrator to justice. Therefore, you should discourage the patient from showering, urinating, or changing clothes. Again, your assessment should be focused on injuries that are life-threatening. Any articles of clothing that are removed should be placed in a paper bag; condensation can form in plastic bags and potentially destroy evidence. Documentation of the call should include information that is relevant to patient care; concise, detailed information regarding what happened will be gathered by law enforcement personnel.

The purpose of the pediatric assessment triangle is to: A) identify if the child has a medical condition or a traumatic injury. B) detect immediate life threats through a quick hands-on assessment. C) determine whether or not the child requires a hands-on assessment. D) form a general impression of the child without touching him or her.

D) form a general impression of the child without touching him or her. The pediatric assessment triangle (PAT) is a structured assessment tool that allows you to rapidly form a general impression of the infant's or child's condition without touching him or her. The intent is to provide a "first glance" assessment to identify the general category of the child's physiologic problem and to establish urgency for treatment and/or transport. The PAT is a visual assessment of the child before performing a hands-on assessment. It consists of three elements: appearance (muscle tone and mental status), work of breathing, and circulation to the skin. The only equipment required for the PAT are your own eyes and ears. The PAT will help the EMT determine if the child's problem is of a respiratory, circulatory, or neurologic nature.

When insulin levels in the blood remain high: A) a fruity odor can be detected on the patient's breath. B) the cells starve for glucose and begin to metabolize fat. C) the patient urinates excessively and becomes dehydrated. D) glucose is rapidly taken out of the blood to fuel the cells.

D) glucose is rapidly taken out of the blood to fuel the cells. Insulin is a hormone that promotes the cellular uptake of glucose from the bloodstream. If insulin levels remain high, such as when a diabetic inadvertently takes too much insulin, glucose is rapidly taken out of the blood to fuel the cells. This leads to low circulating blood glucose levels (hypoglycemia) and a condition called insulin shock. By contrast, if insulin levels are too low, such as when a diabetic forgets to take his or her insulin, glucose cannot enter the cells and pools in the bloodstream (hyperglycemia). In the absence of glucose, the cells begin to metabolize fat, which produces ketoacids (diabetic ketoacidosis [OKA]). The respiratory system attempts to eliminate ketoacids from the blood with an increased rate and depth of breathing (Kussmaul respirations) that is accompanied by a fruity or acetone breath odor. Hyperglycemia causes the patient to eliminate excess water from the body through urination (diuresis), resulting in dehydration.

Use of the automated external defibrillator is contraindicated in patients who: A) are between 1 and 8 years of age. B) experienced a witnessed cardiac arrest. C) are apneic and have a weak carotid pulse. D) have a nitroglycerin patch applied to the skin.

D) have a nitroglycerin patch applied to the skin. The AED is applied only to patients in cardiac arrest (eg, pulseless and apneic), whether the arrest was witnessed or unwitnessed. According to the 2015 guidelines for CPR and Emergency Cardiac Care (ECC), AED can safely be u ed in infants and children less than 8 years of age in conjunction with a doseattenuating system (energy reducer) and pediatric pads. However, if pediatric pads and an energy reducer are unavailable, adult AED pads should be used. A nitroglycerin patch is not a contraindication to the use of an AED; simply remove the patch (with gloved hands) and apply the AED as usual.

A 29-year-old man with abdominal pain tells the EMT that he has hemophilia A. This means that: A) his red blood cells break down rapidly. B) he will lose blood at a much faster rate. C) his risk for a pulmonary embolism is high. D) he will bleed for a longer period of time.

D) he will bleed for a longer period of time. Hemophilia A is a rare disease caused by a deficiency of Factor VIII, which is an essential component of blood clotting. The body relies upon an intact clotting system in order to spontaneous stop bleeding (hemostasis). Hemophilia impairs the process of hemostasis because the patient's ability to create a clot following an injury is decreased. Hemophilia also increases the risk of spontaneous bleeding, including in the brain. Patients with hemophilia bleed longer, not faster. Patients whose blood clots too quickly (not slowly) are at risk for a pulmonary embolism. Hemophilia does not affect the red blood cells, or how quickly they break down (hemolysis).

The MOST important initial treatment for a patient whose cardiac arrest was witnessed is: A) defibrillation. B) cardiac drug therapy. C) rapid transport. D) high-quality CPR.

D) high-quality CPR. Regardless of whether a patient's cardiac arrest is witnessed or unwitnessed, the single most important initial treatment is high-quality CPR. Delays in performing CPR have been clearly linked to poor patient outcomes. After CPR has been initiated, apply the AED as soon as it is available. Cardiac drug therapy may enhance the patient's chance of survival. There is evidence suggesting that transport to the hospital with CPR in progress does not improve survival; resuscitation at the scene may be more beneficial for the patient; follow your local protocols and current research trends. Minimally interrupted, highquality CPR, however, is clearly linked to good patient outcomes.

Activated charcoal may be indicated following an ingestion of: A) lithium. B) gasoline. C) antifreeze. D) ibuprofen.

D) ibuprofen. Activated charcoal binds (adsorbs) to specific toxins and prevents their absorption by the body. The toxins are then carried out of the body in the stool. Activated charcoal is not indicated, nor will it be effective, for patients who have ingested alkali poisons, cyanide, ethanol (gasoline), iron, lithium, methanol (wood alcohol), mineral acids, organic solvents, and ethylene glycol (antifreeze). Patients who ingested large quantities of ibuprofen, acetaminophen, and aspirin would be typical candidates for activated charcoal, provided that they are conscious and alert and can protect their own airway.

A man was struck in the side of the head with a steel pipe. Blood-tinged fluid is draining from the ear and bruising appears behind the ear. The MOST appropriate treatment for this patient includes: A) elevating the lower extremities and providing immediate transport. B) applying high-flow oxygen and packing the ear with sterile gauze pads. C) controlling the drainage from the ear and immobilizing the entire spine. D) immobilizing the spine, administering oxygen, and monitoring for vomiting.

D) immobilizing the spine, administering oxygen, and monitoring for vomiting. Patients with significant head injury should be treated by applying high-flow oxygen, assisting ventilations as needed, immobilizing the entire spine, and transporting promptly. Closely monitor the patient for vomiting and be prepared to suction the airway. Elevation of the foot of the spine board may cause more blood to engorge the brain and may increase intracranial pressure (ICP). If possible, you should elevate the patient's torso to 30 degrees in an attempt to lower ICP. You should never attempt to control bleeding or fluid drainage from the ears of a patient with a head injury because this, too, may result in increased ICP. If a patient with an isolated head injury begins showing signs of shock (ie, tachycardia, diaphoresis, tachypnea, hypotension), you should assume that he or she has internal bleeding from another injury and treat accordingly (ie, keep the patient warm and supine).

Tachycardia can be detrimental to a patient who is experiencing a cardiac problem because it causes: A) increased cardiac filling in between beats. B) a profound decrease in oxygen consumption. C) an associated increase in breathing difficulty. D) increased cardiac oxygen usage and demand.

D) increased cardiac oxygen usage and demand. Many patients experiencing a cardiac problem are tachycardic (heart rate greater than 100 beats/min); others are bradycardic (heart rate less than 60 beats/min). As the heart beats faster, it consumes, and therefore requires, more oxygen. This can be detrimental to the patient because the heart is already deprived of oxygen (ischemia) and may not be able to accommodate the increased need for oxygen; this may worsen the ischemia, potentially causing further cardiac damage. A very fast heart rate (greater than 150 beats/min) may cause hypotension due to a decrease in cardiac output; it occurs because the ventricles are not adequately filling with blood in between beats. Some patients with a cardiac problem may have difficulty breathing, which may be the result of congestive heart failure and pulmonary edema; it is not caused by the tachycardia itself.

A patient with multiple injuries presents with pallor, diaphoresis, a heart rate of 120 beats/min, and a BP of 112/64 mm Hg. The EMT should recognize that these signs are the result of: A) the shunting of blood to the periphery of the body. B) increased parasympathetic nervous system activity. C) nervous system failure and decompensated shock. D) increased epinephrine and norepinephrine release.

D) increased epinephrine and norepinephrine release. The patient's signs and symptoms are consistent with compensated shock. Decompensated shock occurs when the body's compensatory mechanisms have failed and the patient becomes hypotensive. The body responds to inadequate perfusion in a number of ways, including increasing the respiratory rate and releasing catecholamines (epinephrine and norepinephrine). Catecholamine release causes profuse sweating (diaphoresis), increased heart rate, and central and peripheral vasoconstriction. Because the peripheral blood vessels are constricted as blood is shunted away from the periphery, the skin becomes pale. The net effect of vasoconstriction is to increase blood pressure and maintain perfusion. Increased parasympathetic nervous system activity would cause bradycardia, not tachycardia.

A man's legs were pinned by a steel girder for 5 hours before he was found. He is conscious and alert with stable vital signs but cannot feel his legs. The EMT should anticipate that: A) a surgeon will respond to the scene and amputate the man's legs. B) he will be heavily sedated by paramedics to prevent further pain. C) he will be immediately freed and rapidly transported to the hospital. D) intravenous fluids and certain drugs may be given before he is freed.

D) intravenous fluids and certain drugs may be given before he is freed. Crush syndrome occurs when a part of the body is pinned by a heavy object for more than 4 to 6 hours, although it can occur earlier. Since circulation distal to the crushed part of the body is minimal or absent, chemicals such as potassium and lactic acid can accumulate to dangerous levels. If the body part(s) is/are suddenly freed, these chemicals can be released into the bloodstream, potentially causing a fatal cardiac dysrhythmia. For this reason, the patient may need intravenous fluids and certain medications before being freed in order to prepare the body for the onslaught of these chemicals. In some cases, especially if the patient is hemodynamically unstable, a surgeon may respond to the scene to perform an emergency amputation. However, in this case, the patient is stable. It would not be advisable to heavily sedate the patient while he is still entrapped as this may cause airway compromise. Sedation may be indicated, however, after the patient has been freed.

If a woman is having her first child, the first stage of labor: A) generally does not allow time for you to transport. B) is typically very short and lasts only about 2 hours. C) is shorter than in women who have had other children. D) is usually the longest and lasts an average of 16 hours.

D) is usually the longest and lasts an average of 16 hours. There are three stages of labor: dilation of the cervix, delivery of the baby, and delivery of the placenta. The first stage begins with the onset of contractions and ends when the cervix is fully dilated. Since assessing for cervical dilation is not performed in the prehospital setting, the first stage of labor is said to have ended when crowning occurs. Because the cervix has to be stretched thin by uterine contractions until the opening is large enough for the fetus to pass through into the vagina, the first stage is usually the longest, lasting an average of 16 hours for a first delivery. With subsequent pregnancies, the first stage of labor typically progresses more quickly. You will usually have enough time to transport the mother during the first stage of labor, especially if this is her first pregnancy. It should be noted, however, than some primigravida (pregnant for the first time) women progress through the first stage of labor very quickly.

A young man has a large laceration to his lateral neck, directly over his jugular vein. His airway is patent and his breathing is adequate. Your MOST immediate priority should be to: A) apply high-flow oxygen via a nonrebreathing mask. B) perform a rapid assessment to detect other injuries. C) obtain vital signs to determine if he is hypotensive. D) keep air out of the wound and control the bleeding.

D) keep air out of the wound and control the bleeding. Jugular vein lacerations pose two immediate life threats: entrainment of air into the wound (which may cause a fatal air embolism) and severe external bleeding. The patient's airway is patent and his breathing is adequate; therefore, your most immediate priority is to apply an occlusive dressing directly over the wound, which will keep air from entering the venous circulation, and then cover the occlusive dressing with bulky dressings to control the external bleeding. Apply oxygen if it is clinically indicated (ie, low Sp02). The need to perform a rapid head- to-toe assessment is based on the presence of a significant mechanism of injury (MO!). If a significant MO! is present, the rapid assessment is performed only after problems with the airway, breathing, and circulation have been addressed. Vital signs are typically obtained after the rapid assessment, although they can be obtained by another EMT as you perform the rapid assessment.

Your assessment of the newborn reveals that she has a patent airway, is breathing adequately, and has a heart rate of 130 beats/min. Her face and trunk are pink, but her hands and feet are cyanotic. You have clamped and cut the umbilical cord, but the placenta has not yet delivered. You should: A) massage the lower part of the mother's uterus until the placenta delivers. B) give the newborn high-flow oxygen via a nonrebreathing mask and transport. C) reassess the newborn every 5 minutes and transport after the placenta delivers. D) keep the newborn warm, give oxygen to the mother if needed, and transport.

D) keep the newborn warm, give oxygen to the mother if needed, and transport. The newborn is stable and does not require care beyond providing thermal management and monitoring. Oxygen is indicated for the newborn if it has central cyanosis (cyanosis to the face and trunk), and should be delivered via the blow-by technique with the flowmeter set at 5 L/min. Cyanosis to the periphery of the body (eg, hands and feet), which is called acrocyanosis, is a normal finding in the newborn. You should not wait at the scene for the placenta to deliver; it can take up to 45 minutes for this to occur. Begin transport, keep the newborn warm, and give oxygen to the mother if indicated. Massaging the uterine fundus (top part of the uterus) is indicated for women with postpartum vaginal bleeding (ie, BOTH the baby and placenta have delivered).

A 22-year-old woman fell on her knee and is in severe pain. Her knee is flexed and severely deformed. Her leg is cold to the touch and you are unable to palpate a distal pulse. You should: A) apply gentle longitudinal traction as you straighten her leg and then apply a traction splint. B) place a pillow behind her knee and stabilize the injury by applying padded board splints. C) carefully straighten her leg until you restore a distal pulse and then apply padded board splints. D) manually stabilize her injury and contact medical control for further stabilization instructions.

D) manually stabilize her injury and contact medical control for further stabilization instructions. A dislocated knee occurs when the proximal end of the tibia completely displaces from its juncture with the distal femur. In some cases, the popliteal artery behind the knee may be compressed, resulting in compromised distal blood flow. Signs of this include absent distal pulses and a pale extremity that is cool or cold. Manually stabilize the knee and assess for distal pulses. If distal pulses are absent, contact medical control immediately for further stabilization instructions. Medical control may instruct you to make ONE attempt to realign the knee to reduce compression of the popliteal artery and restore distal circulation. If you are unable to restore distal circulation or medical control advises you not to manipulate the injury, splint the knee in the position it was found and transport promptly. Traction splints are contraindicated in any injury to or near the knee.

To maintain neutral alignment of an 18-month-old child's airway, you should: A) hyperextend the head. B) slightly flex the head. C) place a rolled towel under the back of the head. D) place padding in between the shoulder blades.

D) place padding in between the shoulder blades. Infants and small children have proportionately large heads, specifically the occiput (back of the head). Therefore, it is often necessary to place padding in between the scapulae (shoulder blades) to ensure neutral alignment of the head. Padding behind the head places the child's head in the sniffing position, which is used to facilitate intubation. If the infant's or child's head is hyperextended, the large occiput may push the head forward, resulting in hyperflexion. Flexing the child's head (even slightly) can collapse the trachea, resulting in obstruction of the airway.

You should assist with the delivery of the baby's head by: A) carefully rotating its head to where it is facing up when it delivers. B) placing the palm of your hand firmly against the back of the baby's skull. C) grasping each side of the baby's head and gently pulling to facilitate delivery. D) placing your fingers on the bony part of the skull and applying gentle pressure.

D) placing your fingers on the bony part of the skull and applying gentle pressure. Assist with the delivery of the baby's head by placing the flat parts of your fingers on the bony part of the skull as it emerges from the vagina and then applying gentle pressure to avoid an explosive delivery. Avoid pressing your fingers on the anterior and posterior fontanelles (soft spots). Do not attempt to rotate the baby's head or pull on it to facilitate delivery; these actions clearly increase the risk of injury.

Your assessment of a middle-aged woman with chest pressure reveals that she is confused, is diaphoretic, and has a blood pressure of 70/50 mm Hg. In caring for this patient, it is MOST important for you to: A) request an ALS unit to respond to the scene. B) assist her with her nitroglycerin if she has any. C) reasses her vital signs at least every 5 minutes. D) prepare for immediate transport to the hospital.

D) prepare for immediate transport to the hospital. Your patient is in shock, which is most likely the result of heart failure (cardiogenic shock). Immediate transport to the closest appropriate hospital is critical. Assess for and manage problems with airway, breathing, and circulation, and then get on the road! She needs definitive care that can be provided only at the hospital. Reassess her vital signs at least every 5 minutes, but do it en route! If you can rendezvous with an ALS unit en route, do so. However, you should not remain at the scene to wait for them. Nitroglycerin is clearly contraindicated for this patient; her BP is dangerously low.

After moving a hypothermic patient to a warmer area, your primary focus should be to: A) provide rapid rewarming. B) give warm, humidified oxygen. C) assess his or her body temperature D) prevent further body heat loss.

D) prevent further body heat loss. After moving a hypothermic patient to a warmer area, your primary focus should be to prevent further body heat loss. Remove wet or damp clothing and cover the patient with blankets (passive rewarming). It is especially important to cover the patient's head; most heat is lost around the head and neck. You should not, however, try to actively rewarm the patient (ie, place heat on or into the body). Rewarming too quickly may induce a lethal cardiac dysrhythmia and cardiac arrest. For this reason, active rewarming should be done in the controlled setting of a hospital. In some areas, medical control may call for the administration of warm, humidified oxygen; follow your local protocols. Assessing a hypothermic patient's core body temperature requires a special hypothermia thermometer, which many EMS systems do not carry.

A young man has multiple injuries after he fell approximately 35 feet. He is semiconscious and has an unstable chest wall, numerous long bone fractures, and a large hematoma to his head. He will have the BEST chance for survival if you: A) request an ALS ambulance. B) give him high-flow oxygen early. C) keep him warm and elevate his legs. D) rapidly transport him to a trauma center.

D) rapidly transport him to a trauma center. When caring for a patient with major trauma, rapid transport to a trauma center is essential and will afford the patient the best chance for survival. This is especially true if the patient has trauma to multiple body systems. Definitive care cannot be provided in the fie ld; this requires resources and personnel at the hospital. Oxygen administration and shock treatment (eg, applying warm blankets, keeping the patient supine) may help delay patient deterioration, thus buying some time; although these measures are important, they are not definitive interventions. In certain situations, it would be prudent to request ALS personnel at the scene (eg, lengthy extrication, unavoidable scene delay); however, in the absence of such extenuating circumstances, it is clearly more important to transport without delay. En route to the trauma center, consider an ALS intercept if it is possible and does not delay transport.

You are at the scene where a man panicked while swimming in a small lake. Your initial attempt to rescue him should include: A) throwing a rope to the victim. B) rowing a small raft to the victim. C) swimming to the victim to rescue him. D) reaching for the victim with a long object.

D) reaching for the victim with a long object. General rules to fo llow when attempting to rescue a patient from the water include "reach, throw, row, and then go." In this case, you should attempt to reach the victim by having him grab hold of a long object, such as a stick or pole. If this is not possible or unsuccessful, throw the victim a rope or flotation device (if available). If these are not available, row to the patient in a small raft (if available). Going into the water to retrieve the victim is a last resort. The rescuer must be a strong swimmer because patients who are in danger of drowning are in a state of blind panic and will make every attempt to keep themselves afloat, even if it means forcing the rescuer underwater.

After administering nitroglycerin to a patient with chest discomfort, it is MOST important for you to: A) ask the patient if the discomfort has improved. B) find out how long the discomfort has been present. C) position the patient supine and transport immediately. D) reassess the patient's blood pressure within 5 minutes.

D) reassess the patient's blood pressure within 5 minutes. Nitroglycerin (NTG) relaxes the muscle of blood vessel walls, dilates the coronary arteries, increases blood flow and the supply of oxygen to the heart muscle (myocardium), and decreases the workload of the heart. NTG also dilates blood vessels in other parts of the body, potentially resulting in hypotension. For this reason, you should reassess the patient's blood pressure within 5 minutes after each dose of NTG. If the systolic blood pressure is less than 100 mm Hg, do not give anymore NTG. If significant hypotension occurs, position the patient supine and transport without delay. Asking the patient if his or her chest pain or discomfort has improved following NTG helps you determine if the drug is working and whether additional dosing is needed; however, detecting hypotension is clearly more important. You should determine when the chest pain or discomfort began during the focused history, which is typically performed before assisting a patient with his or her prescribed NTG.

A patient with a core body temperature of 94°F would MOST likely present with: A) decreased muscle activity. B) an altered mental status. C) joint and muscle stiffness. D) shivering and pale skin.

D) shivering and pale skin. Mild hypothermia is defined as a core body temperature (CBT) between 93.2 degrees F and 96.8 degrees F (34 degrees C and 36 degrees C). Mildly hypothermic patients are usually alert and shivering in an attempt to generate heat through muscular activity. Their skin is typically pale due to the body's constriction of blood vessels at the skin to retain heat. In moderate hypothermia (CBT between 86 degrees F and 93.2 degrees F [30 degrees C and 34 degrees C]), shivering stops and muscular activity decreases. As the CBT falls further, all muscle activity stops. In severe hypothermia (CBT 86 degrees F [30 degrees C]), vital functions decrease (eg, level of consciousness, BP, pulse, respirations) and the patient is at risk for life-threatening cardiac dysrhythmias. The muscles become rigid and the patient appears stiff. The patient may appear dead; although a pulse is present, it may not be palpable.

Displaced fractures of the proximal femur are characterized by: A) lengthening and internal rotation of the leg. B) a flexed hip joint and inward thigh rotation. C) hip joint extension and external leg rotation. D) shortening and external rotation of the leg.

D) shortening and external rotation of the leg. Fractures of the proximal (upper) part of the femur are especially common in older people, particularly those with osteoporosis, but may also occur as a result of high-energy trauma in younger patients. Although they are usually called hip fractures, they rarely involve the hip joint. Instead, the break goes through the neck of the femur, the middle region, or across the proximal shaft. Patients with displaced fractures of the proximal femur display a very characteristic deformity. They lie with the leg externally rotated, and the injured leg is usually shorter than the uninjured leg. If the fracture is not displaced, this deformity is not present. A flexed hip joint and internal rotation of the thigh are characteristic of a posterior hip dislocation. With the less common anterior hip dislocation, the limb is in the opposite position, extended straight out, externally rotated, and pointing away from the midline of the body.

A 44-year-old woman was bitten on the ankle by an unidentified snake while working in her garden. She is conscious and alert, has stable vital signs, and denies shortness of breath. Her only complaint is a burning sensation at the wound site. Your assessment reveals two small puncture wounds, redness, and swelling. You should: A) elevate her leg 12 inches and apply an ice pack to reduce pain and swelling. B) apply a constricting band proximal to the bite and use ice to prevent venom spread. C) conclude that envenomation did not occur and allow a friend to take her to the hospital. D) splint her leg to decrease movement and keep her leg below the level of her heart.

D) splint her leg to decrease movement and keep her leg below the level of her heart. Given the fact that the snake was not identified, you should assume that it was poisonous. Furthermore, the presence of puncture wounds, burning, redness, and swelling are suggestive of envenomation. Therefore, you should provide emergency care and transport the patient to the hospital. Treatment for a snake bite involves keeping the patient calm, administering oxygen (if hypoxemic or short of breath), splinting the affected extremity to decrease movement (helps slow the spread of venom), keeping the extremity below the level of the heart, and transporting the patient to the hospital. Do NOT apply ice to a snake bite; it may constrict the blood vessels and force venom further into the bloodstream. The use of a proximal constricting band is controversial; if one is used, it should be loose, not tight. En route to the hospital, monitor the patient's vital signs and mental status, and be alert for vomiting.

When assessing a patient with an apparent behavioral crisis, the MOST important initial observation the EMT should make is whether or not: A) the patient is oriented to person, place, and time. B) the patient's residence is well kept or in disarray. C) there is any drug paraphernalia near the patient. D) the patient seems aggressive or verbally abusive.

D) the patient seems aggressive or verbally abusive. All of the options in this question are important to assess. However, the safety of you and your crew supersedes everything! If ANY patient is aggressive or verbally abusive, that is a direct threat to your safety, and you should request law enforcement personnel. It is common (and wise) for law enforcement to be dispatched to the scene automatically for patients with behavioral crises. If they are not, however, you should request them as soon as possible. Once it is safe for you and your team to do your job, you should gather information specific to the patient's chief complaint or presentation.

Prior to being reoxygenated in the lungs, blood passes through the: A) mitral and tricuspid valves. B) aortic and tricuspid valves. C) aortic and pulmonic valves. D) tricuspid and pulmonic valves.

D) tricuspid and pulmonic valves. There are two valves that blood must pass through before being reoxygenated in the lungs, the tricuspid and the pulmonic. After returning to the body, blood passes from the right atrium, through the tricuspid valve, to the right ventricle. It then passes through the pulmonic valve, into the pulmonary arteries, and to the lungs. The mitral valve is located between the left atrium and left ventricle; blood passes through this valve after being reoxygenated in the lungs. Oxygenated blood moves through the aortic valve, into the aorta, and out to the body.

Treatment for a patient with congestive heart fa ilure and shortness of breath may include: A) prophylactic suctioning of the airway. B) hyperventilation with a bag-mask device. C) supine positioning and elevation of the legs. D) up to three doses of sublingual nitroglycerin.

D) up to three doses of sublingual nitroglycerin. Treatment for patients with congestive heart failure (CHF) includes supplemental oxygen as needed to maintain an oxygen saturation equal to or greater than 94%, continuous positive airway pressure (CPAP), ventilatory assistance with a bag-mask device if needed (do NOT hyperventilate the patient), placing the patient in an upright or sitting position to facilitate breathing (a supine position will clearly make it more difficult to breathe), monitoring the patient's vital signs, and transporting to the hospital without delay. Suction the airway only if there are secretions in the mouth; prophylactic suctioning is not indicated. Nitroglycerin (NTG) may be of value if the patient is not hypotensive and he or she has the medication prescribed to him or her. As a vasodilator, NTG causes systemic venous pooling of blood, which reduces the amount of blood returned to the heart (preload) and, therefore, the amount of blood available to back up in the lungs. Follow your local protocols or contact medical control as needed regarding the use of NTG for patients with CHF.

When an electrical impulse reaches the atrioventricular node, it is briefly delayed so that: A) the aortic valve can open. B) both of the atria can relax. C) the ventricles can contract. D) ventricular filling can occur.

D) ventricular filling can occur. The heart functions as a double pump; when the atria are contracting, the ventri !es are filling, and when the ventricles are contracting, the atria are filling. In order to maintain the double pump effect, the electrical impulse coming from the sinoatrial node must undergo a brief delay at the atrioventricular node; this delay allows the ventricles to fill with blood. Most of the blood that fills the ventricles does so passively, with final filling occurring during atrial contraction; this is called the atrial kick. The brief delay at the AV node does not affect when the aortic valve opens, when the ventricles contract, or when the atria relax.

A 16-year-old, 125-pound man ingested a bottle of aspirin approximately 20 minutes ago. Medical control orders you to administer activated charcoal in a dose of 1 g/kg. How much activated charcoal should you administer? A) 51 g B) 54 g C) 57 g D) 60 g

First, you must determine the patient's weight in kilograms (kg). Either of the following formulae can be used to convert pounds to kilograms: Formula 1: weight (in pounds)/ 2.2 = weight in kg. Formula 2: weight (in pounds)/ 2 minus 10% = weight in kg. On the basis of the above formulae, a 125-pound patient weighs 57 kg. Using formula 1, the equation is as follows: 125 (weight in pounds)/ 2.2 = 56.81 (57 [rounded to the nearest tenth]). Using formula 2, the equation is as follows: 125 (weight in pounds)/ 2 = 62.5 (63 [rounded to the nearest tenth] - 6.3 (10% of 63) = 56.7 (57 [rounded to the nearest tenth]). Since the drug order is for 1 g/kg, you should administer 57 g of activated charcoal to your 125-pound patient.


Ensembles d'études connexes

Postassessment insurance guaranty Association

View Set

The Marketing Mix: Product, Place, Promotion, and Price

View Set